Go Math Grade 3 Answer Key Chapter 3 Understand Multiplication Extra Practice

go-math-grade-3-chapter-4-multiplication-facts-and-strategies-extra-practice-answer-key

Avail the handy resource available for your needs Go Math Grade 3 Answer Key Chapter 3 Understand Multiplication Extra Practice. Check out worked out solutions for all the problems and have an indepth knowledge of the concepts in Go Math Grade 3 Chapter 3. Get acquainted with the kind of questions using the 3rd Grade Go Math Solutions Key Ch 3 Understand Multiplication Extra Practice. Elaborate Explanation is provided in the Go Math Grade 3 Answer Key making it easier for you to understand the concepts underlying.

Go Math Grade 3 Chapter 3 Understand Multiplication Extra Practice Answer Key

The Series of Math Problems provided will help you attain great grades in your exams. Refer our Go Math Grade 3 Answer Key Chapter 3 Understand Multiplication Extra Practice and solve different questions. Practice as many times as possible so that you can attempt the exam with confidence and clear the tests with flying colors.

Common Core – Page No. 63000

Lesson 3.1

Draw equal groups. Skip count to find how many.

Question 1.
2 groups of 4 ______

Answer:
8

Explanation:
Draw 4 counters in each group.
There are 2 equal groups.
skip count by 4 until you say 2 numbers.
There are 2 equal groups with 4 counters in each group.
So, there are 8 counters in all.

Question 2.
4 groups of 3 ______

Answer:
12

Explanation:
Draw 3 counters in each group.
There are 4 equal groups.
skip count by 3 until you say 4 numbers.
There are 4 equal groups with 3 counters in each group.
So, there are 12 counters in all.

Lesson 3.2

Draw a quick picture to show the equal groups. Then write related addition and multiplication sentences.

Question 3.
2 groups of 5
______ + ______ = ______
______ × ______ = ______

Answer:
5 + 5 = 10
2 x 5 = 10

Explanation:
Addition Sentence
Draw 5 counters in each group.
There are a total of 2 groups.
Now, the addition sentence is 5 + 5 = 10.

Multiplication sentence
Draw 5 counters in each circle or group.
Since there is the same number of counters in each group, multiply counters and groups to find how many there are altogether.
2 x 5 = 10.
factor x factor = product

Question 4.
3 groups of 2
______ + ______ + ______ = ______
______ × ______ = ______

Answer:
2 + 2 + 2 = 6
3 x 2 = 6

Explanation:
Addition Sentence
Draw 2 counters in each group.
There are a total of 3 groups.
Now, the addition sentence is 2 + 2 + 2 = 6

Multiplication sentence
Draw 2 counters in each circle or group.
Since there is the same number of counters in each group, multiply groups and counters to find how many there are altogether.
3 x 2 = 6.
factor x factor = product

Lesson 3.3

1. Draw jumps on the number line to show 3 groups of 6.
Find the product.

Question 5.
Go Math Grade 3 Answer Key Chapter 3 Understand Multiplication Extra Practice Common Core img 1
3 × 6 = ______

Answer:
Chapter 12 Common Core image 1 63000
Explanation:
3 x 6 = 18

Write the multiplication sentence the number line shows.

Question 6.
Go Math Grade 3 Answer Key Chapter 3 Understand Multiplication Extra Practice Common Core img 2
______ × ______ = ______

Answer:
4 x 3 = 12

Explanation:
1 jump on the number is considered as 1 group.
There are 3 jumps on a number line. So, there are 3 groups.
The length of each jump is 4.
Begin at 0. Skip count by 3’s.
Multiply 4 x 3 = 12.

Common Core – Page No. 64000

Lesson 3.4

Question 1.
Destiny placed her hair ribbons in 3 groups of 5 on her dresser. How many hair ribbons in all does Destiny have? Draw a diagram to solve.
______ hair ribbons

Answer:
15 hair ribbons

Explanation:
3 groups of 5
Draw 3 counters (hair ribbons) in each group.
There are 5 equal groups (dresser).
We skip count by 3’s until you say 5 numbers (3,6,9,12,15)
There are 15 hair ribbons in all.

Lesson 3.5

Draw an array to find the product.

Question 2.
2 × 7 = ______

Answer:
2 x 7 = 14

Explanation:
Chapter 12 Common Core image 2 64000
2 x 7 = 14

Question 3.
2 × 6 = ______

Answer:
2 x 6 = 12

Explanation:
Chapter 12 Common Core image 4 64000
2 x 6 = 12

Lesson 3.6

Write a multiplication sentence for the model. Then use the Commutative Property of Multiplication to write a related multiplication sentence.

Question 4.
□□□□□
□□□□□
□□□□□
□□□□□
______ × ______ = ______
______ × ______ = ______

Answer:
4 x 5 = 20
5 x 4 = 20

Explanation:
The given image has 4 rows and 5 columns of square boxes. So, multiplication = 4 x 5. Using Commutative Property of Multiplication 4 x 5 = 5 x 4.

Question 5.
______ × ______ = ______
______ × ______ = ______

Answer:

Explanation:

Lesson 3.7

Find the product.

Question 6.
6 × 0 = ______

Answer:
0

Explanation:
The Zero Property of Multiplication states that the product of zero and any number is zero. So, 6 x 0 = 0.

Question 7.
5 × 1 = ______

Answer:
5

Explanation:
Use Commutative Property of Multiplication: 5 x 1 = 1 x 5.
The Identity Property of Multiplication states that the product of any number and 1 is that number. 5 x 1 = 5.

Question 8.
0 × 9 = ______

Answer:
0

Explanation:
The Zero Property of Multiplication states that the product of zero and any number is zero. So, 0 x 9 = 0.

Question 9.
1 × 8 = ______

Answer:
8

Explanation:
The Identity Property of Multiplication states that the product of any number and 1 is that number. 1 x 8 = 8.

Question 10.
1 × 4 = ______

Answer:
4

Explanation:
The Identity Property of Multiplication states that the product of any number and 1 is that number. 1 x 4 = 4.

Question 11.
9 × 1 = ______

Answer:
9

Explanation:
Use Commutative Property of Multiplication: 9 x 1 = 1 x 9.
The Identity Property of Multiplication states that the product of any number and 1 is that number. 9 x 1 = 9.

Question 12.
1 × 0 = ______

Answer:
0

Explanation:
The Zero Property of Multiplication states that the product of zero and any number is zero.
So, 1 x 0 = 0.

Question 13.
7 × 0 = ______

Answer:
0

Explanation:
The Zero Property of Multiplication states that the product of zero and any number is zero.
So, 7 x 0 = 0.

Conclusion

Go Math Grade 3 Answer Key Chapter 3 Understand Multiplication Extra Practice is intended to help the students to increase their solving ability. It is a great opportunity for students to learn the best way to solve questions. Go Math Grade 3 Chapter 3 Understand Multiplication Extra Practice Answer Key is the first priority who wants to achieve their top grades. Download HMH Go Math Grade 3 Answer Key for free.

Go Math Grade 3 Answer Key Chapter 11 Perimeter and Area

go-math-grade-3-chapter-11-perimeter-and-area-answer-key

Go Math Grade 3 Answer Key Chapter 11 Perimeter and Area helps you prepare for the exams. All the Concepts of Area and Perimeter are clearly explained in our Chapter 11 Perimeter and Area Go Math Grade 3 Answer Key. Go Math Grade 3 Perimeter and Area can be of great help during your preparation. You can get the Homework Help needed by referring to the Go Math Grade 3 Answer Key Chapter 11 Perimeter and Area.

Go Math Grade 3 Chapter 11 Perimeter and Area Answer Key

Enhance your Problem-Solving Skills using the HMH Go Math Grade 3 Ch 11 Perimeter and Area Solution Key. Cross Check the Solutions in the 3rd Grade Go Math Ch 11 Perimeter and Area Answer Key and understand where you went wrong. Get to know the Lessons in Chapter 11 with the quick links available below and solve the problems in it.

Lesson 1: Model Perimeter

Lesson 2: Find Perimeter

Lesson 3: Find Unknown Side Lengths

Lesson 4: Understand Area

Lesson 5: Measure Area

Lesson 6: Use Area Models

Mid -Chapter Checkpoint

Lesson 7: Problem Solving Area of Rectangles

Lesson 8: Area of Combined Rectangles

Lesson 9: Same Perimeter, Different Areas

Review/Test

Model Perimeter – Page No. 629

Find the perimeter of the shape. Each unit is 1 centimeter.

Question 1.
Go Math Grade 3 Answer Key Chapter 11 Perimeter and Area Model Perimeter img 1
22 centimeters

Answer:
22 centimeters

Explanation:
Model Perimeter Image 1
Each square in the grid is a 1 by 1-centimeter square. So, we have to do is add up the lengths of the dark segments right over the figure. Start the count from the box where 1 is placed. This parameter is 1, 2, 3, 4, 5, 6, 7, 8, 9, 10, 11, 12, 13, 14, 15, 16, 17, 18, 19, 20, 21, 22 centimeters long. So, it is 22 centimeters.

Question 2.
Go Math Grade 3 Answer Key Chapter 11 Perimeter and Area Model Perimeter img 2
__________ centimeters

Answer:
22 centimeters

Explanation:
Model Perimeter Image 2
Look at the length of each side. Then, add the length of each side to get the perimeter of the given shape. The lengths of the sides are 6 + 5 + 6 + 5 = 22 centimeters.

Question 3.
Go Math Grade 3 Answer Key Chapter 11 Perimeter and Area Model Perimeter img 3
__________ centimeters

Answer:
26 centimeters

Explanation:
Model perimeter Image 3
Given that each unit is 1 centimeter. Count the lengths of each box from number 1. So, this parameter is 26 centimeters long. or 6 + 5 + 2 + 2 + 2 + 2 + 2 + 5 = 26.

Question 4.
Go Math Grade 3 Answer Key Chapter 11 Perimeter and Area Model Perimeter img 4
__________ centimeters

Answer:
30 centimeters

Explanation:
Model Perimeter Image 4
Look at the length of each side. Then, add the length of each side to get the perimeter of the given shape. The lengths of the sides are 2 + 3 + 3+ 3 + 2 + 5 + 7 + 5 = 30 centimeters long.

Problem Solving

Use the drawing for 5–6. Each unit is 1 centimeter.
Go Math Grade 3 Answer Key Chapter 11 Perimeter and Area Model Perimeter img 5

Question 5.
What is the perimeter of Patrick’s shape?
__________ centimeters

Answer:
20 centimeters

Explanation:
Model Perimeter Image 5
The perimeter of Patrick’s shape = 5 + 5 + 5 + 5 =20 centimeters.

Question 6.
How much greater is the perimeter of Jillian’s shape than the perimeter of Patrick’s shape?
__________ centimeters

Answer:
2 centimeters

Explanation:
Model Perimeter Image 6
First, the perimeter of Jillian’s shape = 8 + 1 + 4 + 2 + 4 + 3 = 22 centimeters.
Difference = 22 – 20 = 2 centimeters.
The perimeter of Jillian’s shape is 2 centimeters greater than the perimeter of Patrick’s shape.

Model Perimeter – Page No. 630

Lesson Check

Question 1.
Find the perimeter of the shape.
Each unit is 1 centimeter.
Go Math Grade 3 Answer Key Chapter 11 Perimeter and Area Model Perimeter img 6
Options:
a. 14 centimeters
b. 16 centimeters
c. 18 centimeters
d. 20 centimeters

Answer:
d. 20 centimeters

Explanation:
Model Perimeter Image 7
Given that each unit is 1 centimeter. Count the lengths of each box of the dark lines from number 1. So, this parameter is 20 centimeters long.

Question 2.
Find the perimeter of the shape.
Each unit is 1 centimeter.
Go Math Grade 3 Answer Key Chapter 11 Perimeter and Area Model Perimeter img 7
Options:
a. 19 centimeters
b. 26 centimeters
c. 33 centimeters
d. 55 centimeters

Answer:
b. 26 centimeters

Explanation:
Model Perimeter Image 9
Find the length of each box and add them to find the perimeter of the given shape. The perimeter of the given shape is 8 + 4 +3 + 1 + 4 + 4 + 1 + 1 = 26 centimeters.

Spiral Review

Question 3.
Which lists the fractions in order from least to greatest?
\(\frac{2}{4}, \frac{2}{3}, \frac{2}{6}\)
Options:
a. \(\frac{2}{3}, \frac{2}{4}, \frac{2}{6}\)
b. \(\frac{2}{6}, \frac{2}{4}, \frac{2}{3}\)
c. \(\frac{2}{4}, \frac{2}{3}, \frac{2}{6}\)
d. \(\frac{2}{3}, \frac{2}{6}, \frac{2}{4}\)

Answer:
b. \(\frac{2}{6}, \frac{2}{4}, \frac{2}{3}\)

Explanation:
The numerator of the given factors is the same. So, look at the denominators to compare the size of the pieces. As the denominator gets smaller, the fraction gets larger. In the given problem, 3 is the smaller denominator compared to 4 and 6. Then, 4 is the next smaller denominator. So, the fractions in order from least to greatest are 2/6, 2/4, 2/3.

Question 4.
Kasey’s school starts at the time shown on the clock. What time does Kasey’s school start?
Go Math Grade 3 Answer Key Chapter 11 Perimeter and Area Model Perimeter img 8
Options:
a. 6:40
b. 8:06
c. 8:30
d. 9:30

Answer:
c. 8:30

Explanation:
In the figure, hour-hand is indicating number 8. So, we say it is 8 hours. The minute hand is on 6. To find the minutes, multiply 6 x 5 = 30. So, it indicates 30 minutes. The time is 8 hours 30 minutes.

Question 5.
Michael and Dex are comparing fraction strips. Which statement is NOT correct?
Options:
a. \(\frac{1}{2}<\frac{2}{2}\)
b. \(\frac{2}{3}>\frac{1}{3}\)
c. \(\frac{4}{8}<\frac{3}{8}\)
d. \(\frac{4}{6}>\frac{2}{6}\)

Answer:
c. \(\frac{4}{8}<\frac{3}{8}\)

Explanation:
The denominators of the given fractions are the same. So, look at the numerators to compare the numbers.
1 < 2 is correct.
2 > 1 is correct.
4 < 3 is not correct. So, 4/8 < 3/8 is not correct.

Question 6.
Aiden wants to find the mass of a bowling ball. Which unit should he use?
Options:
a. liter
b. inch
c. gram
d. kilogram

Answer:
d. kilogram

Explanation:
The kilogram is used to find the mass of a bowling ball.

Find Perimeter – Page No. 635

Use a ruler to find the perimeter. (Note: on mobile devices like smart phones or tablets, the measurements will not be accurate.)

Question 1.
Go Math Grade 3 Answer Key Chapter 11 Perimeter and Area Find Perimeter img 9
12 centimeters

Answer:
12 cm

Explanation:
Add the lengths of the sides measured to the perimeter. 4 + 3 + 2 + 3 = 12 cm.

Question 2.
Go Math Grade 3 Answer Key Chapter 11 Perimeter and Area Find Perimeter img 10
_________ centimeters

Answer:
13 centimeters

Explanation:
Use a centimeter ruler to measure the length of each side.
Record and add the lengths of the sides measured to the nearest centimeter.
5 cm + 1 cm + 1 cm + 2 cm + 4 cm = 13 centimeters.

Question 3.
Go Math Grade 3 Answer Key Chapter 11 Perimeter and Area Find Perimeter img 11
_________ inches

Answer:
8 inches

Explanation:
Use an inch ruler to measure the length of each side to the nearest inch.
Record and add the lengths of the sides measured to the nearest inch.
2 in + 2 in + 2 in + 2 in =8 inches.

Question 4.
Go Math Grade 3 Answer Key Chapter 11 Perimeter and Area Find Perimeter img 12
_________ inches

Answer:
6 inches

Explanation:
Use an inch ruler to measure the length of each side.
Record and add the lengths of the sides measured to the nearest inch.
2 in + 2 in + 2 in = 6 inches.

Problem Solving

Draw a picture to solve 5–6.

Question 5.
Evan has a square sticker that measures 5 inches on each side. What is the perimeter of the sticker?
_________ inches

Answer:
20 inches

Explanation:
Chapter 11 - Find Perimeter- image 33
The square has equal sides. Each side has 5 inches. So, the perimeter of the sticker is 5 x 4 = 20 inches.

Question 6.
Sophie draws a shape that has 6 sides. Each side is 3 centimeters. What is the perimeter of the shape?
_________ centimeters

Answer:
18 centimeters

Explanation:
The perimeter of the shape is = Addition of all sides.
Given that Sophie draws a shape that has 6 sides with 3 centimeters. So, the perimeter of the shape = 3 + 3 + 3 + 3 + 3 + 3 = 18 centimeters

Find Perimeter – Page No. 636

Lesson Check

Use an inch ruler for 1–2.

Question 1.
Ty cut a label the size of the shape shown. What is the perimeter, in inches, of Ty’s label?
Go Math Grade 3 Answer Key Chapter 11 Perimeter and Area Find Perimeter img 13
Options:
a. 4 inches
b. 5 inches
c. 6 inches
d. 7 inches

Answer:
c. 6 inches

Explanation:
Chapter 11 - Find perimeter - image 34
By using an inch ruler, Ty can measure the length of each side of the given shape. Now, find the lengths of each side.
1 in + 2 in + 1 in + 2 in = 6 inches.

Question 2.
Julie drew the shape shown below. What is the perimeter, in inches, of the shape?
Go Math Grade 3 Answer Key Chapter 11 Perimeter and Area Find Perimeter img 14
Options:
a. 2 inches
b. 4 inches
c. 6 inches
d. 8 inches

Answer:
d. 8 inches

Explanation:
Chapter 11 - Find perimeter - image 35
Julie can use an inch ruler to measure the length of each side of the shape. Now, find the lengths of each side. Each side has 2 inches. So, the perimeter of the shape is 2 in + 2 in + 2 in + 2 in = 8 inches.

Spiral Review

Question 3.
What is the perimeter of the shape below?
Go Math Grade 3 Answer Key Chapter 11 Perimeter and Area Find Perimeter img 15
Options:
a. 8 units
b. 10 units
c. 20 units
d. 22 units

Answer:
c. 20 units

Explanation:
Model Perimeter Image 10
From the figure, each unit is 1 centimeter. Count the lengths of each box from number 1. So, this parameter is 20 units long.

Question 4.
Vince arrives for his trumpet lesson after school at the time shown on the clock. What time does Vince arrive for his trumpet lesson?
Go Math Grade 3 Answer Key Chapter 11 Perimeter and Area Find Perimeter img 16
Options:
a. 3:26 A.M.
b. 4:26 A.M.
c. 3:26 P.M.
d. 4:26 P.M.

Answer:
c. 3:26 P.M.

Explanation:
Given that Vince arrives for his trumpet lesson after school at the time shown on the clock. As it is mentioned after school time, it is Noon. The times afternoon and before midnight are written with P.M. The hour hand is indicating number 3. So, the answer is 3:26 P.M.

Question 5.
Matthew’s small fish tank holds 12 liters. His large fish tank holds 25 liters. How many more liters does his large fish tank hold?
Options:
a. 12 liters
b. 13 liters
c. 25 liters
d. 37 liters

Answer:
b. 13 liters

Explanation:
To get the more liters, do subtraction of large fish tank liters to small fish tank liters. So, The large fish tank is 25 liters – 12 liters = 13 liters more than Matthew’s small fish tank.

Question 6.
Cecila and Sasha are comparing fraction strips. Which statement is correct?
Options:
a. \(\frac{1}{2}<\frac{1}{3}\)
b. \(\frac{1}{8}>\frac{1}{6}\)
c. \(\frac{1}{4}>\frac{1}{2}\)
d. \(\frac{1}{6}<\frac{1}{4}\)

Answer:
d. \(\frac{1}{6}<\frac{1}{4}\)

Explanation:
Cecila and Sasha are comparing fraction strips. In the given fractions, the numerators are same. So, compare the denominators to find the largest fraction. As the denominator gets smaller, the fraction gets larger.
2 < 3 wrong.
8 > 6 wrong.
4 > 2 wrong.
6 < 4 correct.
So, 1/6 < 1/4 is the correct answer.

Find Unknown Side Lengths – Page No. 641

Find the unknown side lengths.

Question 1.
Perimeter = 33 centimeters
Go Math Grade 3 Answer Key Chapter 11 Perimeter and Area Find Unknown Side Lengths img 17
5 + 8 + 7 + 4 + x = 33
24 + x = 33
x = 9
x = ____ 9 _____ centimeters

Answer:
9 centimeters

Explanation:
If I knew the length x, I would add all the side lengths to find the perimeter. Given Perimeter = 33 centimeters.
Add the lengths of the given sides. 5 + 8 + 7 + 4 + x = 33.
24 + x = 33.
x = 33 – 24 =9.
x = 9 centimeters.

Question 2.
Perimeter = 14 feet
Go Math Grade 3 Answer Key Chapter 11 Perimeter and Area Find Unknown Side Lengths img 18
r = ____ feet

Answer:
3 feet

Explanation:
Given Perimeter = 14 feet.
Add the lengths of the given sides to find r.
4 ft + 4 ft + r + r = 14 feet.
If r =1 -> 4 + 4 + 1 + 1 = 10 not equal to 14.
If r = 2 -> 4 + 4 + 2 + 2 = 12 not equal to 14.
If r = 3 -> 4 + 4 + 3 + 3 = 14 equal to 14.
So, r = 3 feet.

Question 3.
Perimeter = 37 meters
Go Math Grade 3 Answer Key Chapter 11 Perimeter and Area Find Unknown Side Lengths img 19
s = ____ meters

Answer:
11 meters

Explanation:
Given that Perimeter = 37 meters.
Add the lengths of the given sides to find s.
8 + 11 + 5 + 2 + s = 37.
26 + s = 37.
s = 37 – 26 = 11.
s = 11 meters.

Question 4.
Perimeter = 92 inches
Go Math Grade 3 Answer Key Chapter 11 Perimeter and Area Find Unknown Side Lengths img 20
t = ____ inches

Answer:
15 inches

Explanation:
Given that Perimeter = 92 inches.
Add the lengths of the given sides to find t.
7 + 23 + 12 + 12 +23 + t = 92
77 + t = 92.
s = 92 – 77 = 15.
s = 15 inches.

Problem Solving

Question 5.
Steven has a rectangular rug with a perimeter of 16 feet. The width of the rug is 5 feet. What is the length of the rug? ____ feet

Answer:
3 feet

Explanation:
Chapter 11 - FInd unknown side lengths - image 36
Given that Steven has a rectangular rug with a perimeter of 16 feet. Rectangular has 4 sides with two pairs of opposite sides that are equal in length.
Let the length will be x.
If x = 1 feet -> 5 + x + 5 + x = 5 + 1 + 5 + 1 = 12 feet not equal to 16.
If x = 2 feet -> 5 + x + 5 + x = 5 + 2 + 5 + 2 = 14 feet not equal to 16.
If x = 3 feet -> 5 + x + 5 + x = 5 + 3 + 5 + 3 = 16 feet equal to 16.
So, the length of the blanket is 3 feet.

Question 6.
Kerstin has a square tile. The perimeter of the tile is 32 inches. What is the length of each side of the tile?
____ inches

Answer:
8 inches

Explanation:
Chapter 11 - FInd unknown side lengths - image 37
A square has four sides that are equal in length.
So, 4 x s = 32
4 x 8 = 32.
So, the length of each side of the square is 8 inches.

Find Unknown Side Lengths – Page No. 642

Lesson Check

Question 1.
Jesse is putting a ribbon around a square frame. He uses 24 inches of ribbon. How long is each side of the frame? Options:
a. 4 inches
b. 5 inches
c. 6 inches
d. 8 inches

Answer:
c. 6 inches

Explanation:
Jesse is putting a ribbon around a square frame. A square has four sides that are equal in length.
So, 4 x k = 24 inches.
4 x 6 = 24 inches.
Each side of the frame 6 inches.

Question 2.
Davia draws a shape with 5 sides. Two sides are each 5 inches long. Two other sides are each 4 inches long. The perimeter of the shape is 27 inches. What is the length of the fifth side?
Options:
a. 9 inches
b. 13 inches
c. 14 inches
d. 18 inches

Answer:
a. 9 inches

Explanation:
. From the given information, Davia draws a shape with 5 sides.
2 sides = each 5 inches long.
2 sides = each 4 inches long.
Let the other side is k.
The perimeter of the shape = 5 + 5 + 4 + 4 + k = 27 inches.
18 + k = 27 inches.
k = 27 – 18 inches.
k = 9 inches.

Spiral Review

Question 3.
Which of the following represents 7 + 7 + 7 + 7?
Options:
a. 4 × 4
b. 4 × 7
c. 6 × 7
d. 7 × 7

Answer:
b. 4 × 7

Explanation:
7 + 7 + 7 + 7 = 28.
4 x 4 = 16 not equal to 28.
4 x 7 = 28 equal to 28.
7 + 7 + 7 + 7 = 4 × 7.

Question 4.
Bob bought 3 packs of model cars. He gave 4 cars to Ann. Bob has 11 cars left. How many model cars were in each pack?
Options:
a. 18
b. 11
c. 7
d. 5

Answer:
d. 5

Explanation:
If Bob has 11 cars left after he gave 4 to Ann, that means he had 15 cars in total. If Bob had 15 cars in total, and these 15 cars were divided into 3 packs, that means each pack contained 5 cars. The answer is 5 cars in each pack.

Question 5.
Randy looked at his watch when he started and finished reading. How long did Randy read?
Go Math Grade 3 Answer Key Chapter 11 Perimeter and Area Find Unknown Side Lengths img 21
Options:
a. 55 minutes
b. 45 minutes
c. 35 minutes
d. 15 minutes

Answer:
b. 45 minutes

Explanation:
Randy started reading at 4:10.
Randy finished reading at 4:55.
Subtract to find the Randy read time.
4:55 – 4:10 = 45 minutes
So, Randy read 45 minutes.

Question 6.
Which statement does the model represent?
Go Math Grade 3 Answer Key Chapter 11 Perimeter and Area Find Unknown Side Lengths img 22
Options:
a. \(\frac{4}{4}\) = 1
b. \(\frac{3}{4}\) = 1
c. \(\frac{2}{4}\) = 1
d. \(\frac{1}{4}\) = 1

Answer:
a. \(\frac{4}{4}\) = 1

Explanation:
The first image represents the whole circle divided into 4 equal parts. So, each part is \(\frac{1}{4}\).
Together, all 4 parts represent \(\frac{4}{4}\) or one whole circle.
The second image represents one whole circle.
\(\frac{4}{4}\) = 1.

Understand Area – Page No. 647

Count to find the area for the shape.

Question 1.
Go Math Grade 3 Answer Key Chapter 11 Perimeter and Area Understand Area img 23
Area = 6 square units

Answer:
6 square units

Explanation:
chapter 11 - understand area - image 1
The area is the measure of the number of unit squares needed to cover a flat surface. A unit square is a square with a side length of 1 unit. It has an area of 1 square unit (sq un). Count the number of Squares to get the answers. 1, 2, 3, 4, 5, 6.
Area = 6 square units.

Question 2.
Go Math Grade 3 Answer Key Chapter 11 Perimeter and Area Understand Area img 24
Area = ______ square units

Answer:
4 square units

Explanation:
chapter 11 - understand area - image 2
Count the number of Squares to get the answers. 1, 2, 3, 4.
Area = 4 square units.

Question 3.
Go Math Grade 3 Answer Key Chapter 11 Perimeter and Area Understand Area img 25
Area = ______ square units

Answer:
5 square units

Explanation:
chapter 11 - understand area - image 3
Count the number of Squares to get the answers. 1, 2, 3, 4, 5.
Area = 5 square units.

Question 4.
Go Math Grade 3 Answer Key Chapter 11 Perimeter and Area Understand Area img 26
Area = ______ square units

Answer:
7 square units

Explanation:
chapter 11 - understand area - image 4
Count the number of Squares to get the answers. 1, 2, 3, 4, 5, 6, 7.
Area = 7 square units.

Question 5.
Go Math Grade 3 Answer Key Chapter 11 Perimeter and Area Understand Area img 27
Area = ______ square units

Answer:
8 square units

Explanation:
chapter 11 - understand area - image 5
Count the number of Squares to get the answers. 1, 2, 3, 4, 5, 6, 7, 8.
Area = 8 square units.

Question 6.
Go Math Grade 3 Answer Key Chapter 11 Perimeter and Area Understand Area img 28
Area = ______ square units

Answer:
13 square units

Explanation:
chapter 11 - understand area - image 6
Count the number of Squares to get the answers. 1, 2, 3, 4, 5, 6, 7, 8,9, 10, 11, 12, 13.
Area = 13 square units.

Write area or perimeter for each situation.

Question 7.
carpeting a floor
_________

Answer:
Area

Question 8.
fencing a garden
_________

Answer:
2. Perimeter

Problem Solving

Use the diagram for 9–10.
Go Math Grade 3 Answer Key Chapter 11 Perimeter and Area Understand Area img 29

Answer:
12 square units

Explanation:
chapter 11 - understand area - image 7
Count the number of Squares to get the answers. 1, 2, 3, 4, 5, 6, 7, 8,9, 10, 11, 12.
Area = 12 square units.

Question 9.
Rober to is building a platform for his model railroad. What is the area of the platform?
Area = ______ square units

Answer:
12 square units

Explanation:
The area of the platform = Count the total number of Squares to get the answers. 1, 2, 3, 4, 5, 6, 7, 8,9, 10, 11, 12.
Area = 12 square units.

Question 10.
Rober to will put a border around the edges of the platform. How much border will he need?
Border = ______ units

Answer:
16 units

Explanation:
To know the edges of the platform, we need to calculate the perimeter of the given shape. So, we need to add the length of all sides. 2 + 4 + 3 + 2 + 1 + 1 + 2 + 1 = 16 units.

Understand Area – Page No. 648

Lesson Check

Question 1.
Josh used rubber bands to make the shape below on his geoboard. What is the area of the shape?
Go Math Grade 3 Answer Key Chapter 11 Perimeter and Area Understand Area img 30
Options:
a. 3 square units
b. 4 square units
c. 5 square units
d. 6 square units

Answer:
a. 3 square units

Explanation:
chapter 11 - understand area - image 8
If Josh used rubber bands to make the shape below on his geoboard, he can divide the shape into 2 smaller regular shapes.
The shape 1 marked as square = 1 square unit.
The shape 2 marked as rectangle = 2 square units.
The total area = Shape 1 + Shape 2 = 1 + 2 = 3 square units.

Question 2.
Wilma drew the shape below on dot paper. What is the area of the shape she drew?
Go Math Grade 3 Answer Key Chapter 11 Perimeter and Area Understand Area img 31
Options:
a. 4 square units
b. 5 square units
c. 6 square units
d. 7 square units

Answer:
b. 5 square units

Explanation:
chapter 11 - understand area - image 9
Wilma drew the shape below on dot paper.
The area of the shape = number of unite square boxes = 1 + 1 + 1 + 1 + 1 = 5 square units.

Spiral Review

Question 3.
Leonardo knows it is 42 days until summer break. How many weeks is it until Leonardo’s summer break? (Hint: There are 7 days in a week.)
Options:
a. 5 weeks
b. 6 weeks
c. 7 weeks
d. 8 weeks

Answer:
b. 6 weeks

Explanation:
Leonardo knows it is 42 days until summer break. There are 7 days in a week. So, by dividing 42 with 7 Leonardo can find summer break in weeks. 42/7 = 6 weeks.

Question 4.
Nan cut a submarine sandwich into 4 equal parts and ate one part. What fraction represents the part of the sandwich Nan ate?
Options:
a. \(\frac{1}{4}\)
b. \(\frac{1}{3}\)
c. \(\frac{4}{4}\)
d. \(\frac{4}{1}\)

Answer:
a. \(\frac{1}{4}\)

Explanation:
Nan cut a submarine sandwich into 4 equal parts and ate one part. So, remaining parts are 3. The fraction of the sandwich Nan ate = \(\frac{1}{4}\).

Question 5.
Wanda is eating breakfast. Which is a reasonable time for Wanda to be eating breakfast?
Options:
a. 7:45 A.M.
b. 7:45 P.M.
c. 2:15 P.M.
d. 2:15 A.M.

Answer:
a. 7:45 A.M.

Explanation:
The reasonable time for Wanda to eat breakfast is 7:45 A.M. Because breakfast will eat in the morning i.e, A.M.

Question 6.
Dick has 2 bags of dog food. Each bag contains 5 kilograms of food. How many kilograms of food does Dick have in all?
Options:
a. 3 kilograms
b. 5 kilograms
c. 7 kilograms
d. 10 kilograms

Answer:
d. 10 kilograms

Explanation:
Dick has 2 x 5 = 10 kilograms of food in total.

Measure Area – Page No. 653

Count to find the area of the shape. Each unit square is 1 square centimeter.

Question 1.
Go Math Grade 3 Answer Key Chapter 11 Perimeter and Area Measure Area img 32
Area 14 square centimeters.

Answer:
14 square centimeters

Explanation:
Model Perimeter image 11
The number of unit square in the given figure is 14. So, the area of the shape = 14 square centimeters.

Question 2.
Go Math Grade 3 Answer Key Chapter 11 Perimeter and Area Measure Area img 33
Area = ________ square centimeters

Answer:
16 square centimeters

Explanation:
Model Perimeter Image 12
The area of the shape = 16 square centimeters.

Question 3.
Go Math Grade 3 Answer Key Chapter 11 Perimeter and Area Measure Area img 34
Area = ________ square centimeters

Answer:
11 square centimeters

Explanation:
Model Perimeter Image 13
The area of the shape = 11 square centimeters.

Question 4.
Go Math Grade 3 Answer Key Chapter 11 Perimeter and Area Measure Area img 35
Area = ________ square centimeters

Answer:
22 square centimeters

Explanation:
Model Perimeter Image 14
The area of the shape = 22 square centimeters.

Problem Solving

Alan is painting his deck gray. Use the diagram at the right for 5–6. Each unit square is 1 square meter.
Go Math Grade 3 Answer Key Chapter 11 Perimeter and Area Measure Area img 36

Question 5.
What is the area of the deck that Alan has already painted gray?
_______ square meters

Answer:
16 square meters

Explanation:
Model Perimeter Image 17
The area of the deck that Alan has already painted gray is 16 square meters.

Question 6.
What is the area of the deck that Alan has left to paint?
_______ square meters

Answer:
19 square meters

Explanation:
Model Perimeter Image 18
The area of the deck that Alan has left to paint is 19 square meters.

Measure Area – Page No. 654

Lesson Check

Each unit square in the diagram is 1 square foot.
Go Math Grade 3 Answer Key Chapter 11 Perimeter and Area Measure Area img 37

Question 1.
How many square feet are shaded?
Options:
a. 19 square feet
b. 21 square feet
c. 23 square feet
d. 25 square feet

Answer:
c. 23 square feet

Explanation:
Model Perimeter Image 20

Question 2.
What is the area that has NOT been shaded?
Options:
a. 19 square feet
b. 21 square feet
c. 23 square feet
d. 25 square feet

Answer:
a. 19 square feet

Explanation:
Model Perimeter Image 21

Spiral Review

Question 3.
Sonya buys 6 packages of rolls. There are 6 rolls in each package. How many rolls does Sonya buy?
Options:
a. 42
b. 36
c. 24
d. 12

Answer:
b. 36

Explanation:
6 x 6 = 36. Sonya buys 36 rolls.

Question 4.
Charlie mixed 6 liters of juice with 2 liters of soda to make fruit punch. How many liters of fruit punch did Charlie make?
Options:
a. 3 liters
b. 4 liters
c. 8 liters
d. 12 liters

Answer:
c. 8 liters

Explanation:
6 + 2 = 8 liters. Charlie can make 8 liters of fruit punch.

Question 5.
Which drawing shows \(\frac{2}{3}\) of the circle shaded?
Options:
a. Go Math Grade 3 Answer Key Chapter 11 Perimeter and Area Measure Area img 38
b. Go Math Grade 3 Answer Key Chapter 11 Perimeter and Area Measure Area img 39
c. Go Math Grade 3 Answer Key Chapter 11 Perimeter and Area Measure Area img 40
d.Go Math Grade 3 Answer Key Chapter 11 Perimeter and Area Measure Area img 41

Answer:
d. Go Math Grade 3 Answer Key Chapter 11 Perimeter and Area Measure Area img 41

Explanation:
Option d is the correct answer. 2/3 means the whole circle has 3 parts. 2 of the parts are shaded.

Question 6.
Use the models to name a fraction that is equivalent to \(\frac{1}{2}\).
Go Math Grade 3 Answer Key Chapter 11 Perimeter and Area Measure Area img 42
Options:
a. \(\frac{2}{1}\)
b. \(\frac{2}{2}\)
c. \(\frac{2}{4}\)
d. \(\frac{4}{4}\)

Answer:
c. \(\frac{2}{4}\)

Explanation:
\(\frac{2}{4}\) = \(\frac{1}{2}\). So, the answer is \(\frac{2}{4}\)

Use Area Models – Page No. 659

Find the area of each shape. Each unit square is 1 square foot.

Question 1.
Go Math Grade 3 Answer Key Chapter 11 Perimeter and Area Use Area Models img 43

Answer:
24 square feet

Explanation:
Chapter 11- Use area Models - Image 25 (659)
There are 3 rows of 8 unit squares.
3 x 8 = 24.

Question 2.
Go Math Grade 3 Answer Key Chapter 11 Perimeter and Area Use Area Models img 44

_________ square feet

Find the area of each shape.
Each unit square is 1 square meter.

Answer:
16 square feet

Explanation:
Chapter 11 - Use area models - image 26
There are 4 rows of 4 unit squares.
4 x 4 = 16 square feet.

Question 3.
Go Math Grade 3 Answer Key Chapter 11 Perimeter and Area Use Area Models img 45
_________ square meters

Answer:
12 square meters

Explanation:
Chapter 11 - Use area models - image 27
There are 2 rows of 6 unit squares.
2 x 6 = 12 square meters.

Question 4.
Go Math Grade 3 Answer Key Chapter 11 Perimeter and Area Use Area Models img 46
_________ square meters

Answer:
24 square meters

Explanation:
Chapter 11- use area models - Image 28
There are 4 rows of 6 unit squares.
4 x 6 = 24 square meters.

Question 5.
Go Math Grade 3 Answer Key Chapter 11 Perimeter and Area Use Area Models img 47
_________ square meters

Answer:
15 square meters

Explanation:
Chapter 11 - Use area models - image 29
There are 5 rows of 3 unit squares.
5 x 3 = 15 square meters.

Problem Solving

Question 6.
Landon made a rug for the hallway. Each unit square is 1 square foot. What is the area of the rug?
Go Math Grade 3 Answer Key Chapter 11 Perimeter and Area Use Area Models img 48
_________ square feet

Answer:
20 square feet

Explanation:
chapter 11- use area models - Image 30
Count the number of unit square boxes = 20 square feet. Or, there are 2 rows of 10 unit squares.
2 x 10 = 20 square feet.

Question 7.
Eva makes a border at the top of a picture frame. Each unit square is 1 square inch. What is the area of the border? Go Math Grade 3 Answer Key Chapter 11 Perimeter and Area Use Area Models img 49
_________ square feet

Answer:
8 square feet

Explanation:
Chapter 11 - use area models - image 31
Count the number of unit square boxes = 20 square feet. Or, there are 1 rows of 8 unit squares.
1 x 8 = 8 square feet.

Use Area Models – Page No. 660

Lesson Check

Question 1.
The entrance to an office has a tiled floor. Each square tile is 1 square meter. What is the area of the floor?
Go Math Grade 3 Answer Key Chapter 11 Perimeter and Area Use Area Models img 50
Options:
a. 8 square meters
b. 9 square meters
c. 10 square meters
d. 12 square meters

Answer:
b. 9 square meters

Explanation:
Chapter 11- Use area Models-Image 23(660)
There are 9 square boxes available. So, the area of the floor = 9 square meters.

Question 2.
Ms. Burns buys a new rug. Each unit square is 1 square foot. What is the area of the rug?
Go Math Grade 3 Answer Key Chapter 11 Perimeter and Area Use Area Models img 51
Options:
a. 5 square feet
b. 7 square feet
c. 10 square feet
d. 12 square feet

Answer:
c. 10 square feet

Explanation:
Chapter 11- Use area models-Image 24(660(2))
The area of the rug = 10 square feet.

Spiral Review

Question 3.
Ann and Bill are comparing fraction strips. Which statement is correct?
Options:
a. \(\frac{3}{8}>\frac{5}{8}\)
b. \(\frac{3}{4}<\frac{1}{4}\)
c. \(\frac{3}{6}>\frac{4}{6}\)
d. \(\frac{1}{3}<\frac{2}{3}\)

Answer:
d. \(\frac{1}{3}<\frac{2}{3}\)

Explanation:
The denominator of the given fractions are same. So, compare the numerators to find the correct answer.
3 > 5 wrong.
3 < 1 wrong.
3 > 4 wrong.
1 < 2 correct.
The answer is \(\frac{1}{3}<\frac{2}{3}\).

Question 4.
Claire bought 6 packs of baseball cards. Each pack had the same number of cards. If Claire bought 48 baseball cards in all, how many cards were in each pack?
Options:
a. 54
b. 42
c. 8
d. 6

Answer:
c. 8

Explanation:
6 x k = 48. So, k = 48/6 = 8.
8 cards were in each pack.

Question 5.
Austin left for school at 7:35 A.M.. He arrived at school 15 minutes later. What time did Austin arrive at school?
Options:
a. 7:40 A.M.
b. 7:50 A.M.
c. 7:55 A.M.
d. 8:00 A.M.

Answer:
b. 7:50 A.M.

Explanation:
7:35 A.M. + 15 minutes = 7:50 A.M

Question 6.
Wyatt’s room is a rectangle with a perimeter of 40 feet. The width of the room is 8 feet. What is the length of the room?
Options:
a. 5 feet
b. 12 feet
c. 16 feet
d. 32 feet

Answer:
b. 12 feet

Explanation:
The perimeter of rectangle = 40 feet.
The width of the room is 8 feet.
8 + k + 8 + k = 40.
If k = 10 -> 8 + 10 + 8 + 10 = 36.
If k = 11 -> 8 + 11 + 8 + 11 = 38.
If k = 12 -> 8 + 12 + 8 + 12 = 40.
So, the length of the room = 12 feet.

Mid -Chapter Checkpoint – Page No. 661

Vocabulary

Choose the best term from the box.
Go Math Grade 3 Answer Key Chapter 11 Perimeter and Area Mid -Chapter Checkpoin img 52

Question 1.
The distance around a figure is the _____________ .
_____________

Answer:
Perimeter

Question 2.
The measure of the number of unit squares needed to cover a figure with no gaps or overlaps is the _____________ .
_____________

Answer:
Area

Concepts and Skills

Find the perimeter of the figure. Each unit is 1 centimeter.

Question 3.
Go Math Grade 3 Answer Key Chapter 11 Perimeter and Area Mid -Chapter Checkpoin img 53
_______ centimeters

Answer:
16 centimeters

Explanation:
Chapter 11 - Concepts and Skills -image 32. jpg (2)
5 + 2 + 2 + 1 + 2 + 1 + 1 + 2 = 16 centimeters.

Question 4.
Go Math Grade 3 Answer Key Chapter 11 Perimeter and Area Mid -Chapter Checkpoin img 54
_______ centimeters

Answer:
14 cm

Explanation:
Chapter 11 - Concepts and Skills -image 33. jpg
4 + 3 + 4 + 3 = 14 cm.

Find the unknown side lengths.

Question 5.
Perimeter = 33 centimeters
Go Math Grade 3 Answer Key Chapter 11 Perimeter and Area Mid -Chapter Checkpoin img 55
g = ______ centimeters

Answer:
3 centimeters

Explanation:
Given that Perimeter = 33 centimeters.
10 + 6 + 10 + g + 4 = 33 centimeters.
30 + g = 33 centimeters.
g= 33 – 30 centimeters.
g = 3 centimeters.

Question 6.
Perimeter = 32 feet
Go Math Grade 3 Answer Key Chapter 11 Perimeter and Area Mid -Chapter Checkpoin img 56
k = ______ feet

Answer:
4 feet

Explanation:
Perimeter = 32 feet.
12 ft + k + 12 ft + k = 32 feet.
If k = 1 then 12 + 1 + 12 + 1 = 26 feet not equal to 32 feet.
If k = 2 then 12 + 2 + 12 + 2 = 28 feet not equal to 32 feet.
If k = 3 then 12 + 3 + 12 + 3 = 30 feet not equal to 32 feet.
If k = 4 then 12 + 4 + 12 + 4 = 32 feet equal to 32 feet.
So, k = 4 feet.

Find the area of the figure. Each unit square is 1 square meter.

Question 7.
Go Math Grade 3 Answer Key Chapter 11 Perimeter and Area Mid -Chapter Checkpoin img 57
______ square meters

Answer:
14 square meters

Explanation:
Chapter 11 - Concepts and Skills -image 32

Question 8.
Go Math Grade 3 Answer Key Chapter 11 Perimeter and Area Mid -Chapter Checkpoin img 58
______ square meters

Answer:
30 square meters

Explanation:
Chapter 11 - Concepts and Skills -image 33 (2)
10 x 3 = 30 square meters.

Mid -Chapter Checkpoint – Page No. 662

Question 9.
Ramona is making a lid for her rectangular jewelry box. The jewelry box has side lengths of 6 centimeters and 4 centimeters. What is the area of the lid Ramona is making?
Go Math Grade 3 Answer Key Chapter 11 Perimeter and Area Mid -Chapter Checkpoint img 59
______ square meters

Answer:
24 square meters.

Explanation:
The area of the lid = 4 cm x 6 cm = 24 cm.
Ramona making 24 cm lid for her rectangular jewelry box.

Question 10.
Adrienne is decorating a square picture frame. She glued 36 inches of ribbon around the edge of the frame. What is the length of each side of the picture frame?
Go Math Grade 3 Answer Key Chapter 11 Perimeter and Area Mid -Chapter Checkpoint img 60
a = ______ inches

Answer:
9 inches

Explanation:
A square has the same lengths.
The length of each side 4 x a = 36 inches.
a = 36/4 = 9 inches.

Question 11.
Margo will sweep a room. A diagram of the floor that she needs to sweep is shown at the right. What is the area of the floor?
Go Math Grade 3 Answer Key Chapter 11 Perimeter and Area Mid -Chapter Checkpoint img 61
______ square units

Answer:
27 square units

Explanation:
Chapter 11 - Mid chapter checkpoint - image 38 662
27 square units

Question 12.
Jeff is making a poster for a car wash for the Campout Club. What is the perimeter of the poster?
Go Math Grade 3 Answer Key Chapter 11 Perimeter and Area Mid -Chapter Checkpoint img 62
______ feet

Answer:
8 ft

Explanation:
Perimeter = 3 + 1 + 3 + 1 = 8 ft.

Question 13.
A rectangle has two side lengths of 8 inches and two side lengths of 10 inches. What is the perimeter of the rectangle? What is the area of the rectangle?
Perimeter ______ inches
Area = ______ square inches

Answer:
Perimeter = 36 inches.
Area = 80 square inches.

Explanation:
Chapter 11 - Mid chapter checkpoint - image 39
The perimeter of the rectangle = 8 + 10 + 8 + 10 = 36.
Area = 10 x 8 = 80 square inches.

Problem Solving Area of Rectangles – Page No. 667

Use the information for 1–3.

An artist makes rectangular murals in different sizes. Below are the available sizes. Each unit square is 1 square meter.
Go Math Grade 3 Answer Key Chapter 11 Perimeter and Area Problem Solving Area of Rectangles img 63

Chapter 11 - problem solving area of rectangles-images 39

Question 1.
Complete the table to find the area of each mural.

Mural Length (in meters) Width (in meters) Area (in square meters)
A 2 1 2
B 2 2 4
C 2 __4_______ __8_______
D 2 ____8_____ ___16______

Question 2.
Find and describe a pattern of how the length changes and how the width changes for murals A through D.
Type below:
_____________

Answer:
For each mural, the width doubles and the length stays the same.

Question 3.
How do the areas of the murals change when the width changes?
Type below:
_____________

Answer:
For each mural, the area doubles.

Question 4.
Dan built a deck that is 5 feet long and 5 feet wide. He built another deck that is 5 feet long and 7 feet wide. He built a third deck that is 5 feet long and 9 feet wide. How do the areas change?
Type below:
_____________

Answer:
The area of each deck is increased by 10 square feet.

Explanation:
1st deck area = 5 x 5 = 25 feet.
2nd deck area = 5 x 7 = 35 feet.
3rd deck area = 5 x 9 = 45 feet.
The area of each deck is increased by 10 square feet.

Problem Solving Area of Rectangles – Page No. 668

Lesson Check

Question 1.
Lauren drew the designs below. Each unit square is 1 square centimeter. If the pattern continues, what will be the area of the fourth shape?
Go Math Grade 3 Answer Key Chapter 11 Perimeter and Area Problem Solving Area of Rectangles img 64
Options:
a. 10 square centimeters
b. 12 square centimeters
c. 14 square centimeters
d. 16 square centimeters

Answer:
b. 12 square centimeters

Explanation:
Chapter 11 - problem solving area of rectangles-images 40
4 x 3 = 12 cm. The area of the fourth shape is 12 square centimeters.

Question 2.
Henry built one garden that is 3 feet wide and 3 feet long. He also built a garden that is 3 feet wide and 6 feet long, and a garden that is 3 feet wide and 9 feet long. How do the areas change?
Options:
a. The areas do not change.
b. The areas double.
c. The areas increase by 3 square feet.
d. The areas increase by 9 square feet.

Answer:
d. The areas increase by 9 square feet.

Explanation:
1st garden = 3 x 3 = 9 feet.
2nd garden = 3 x 6 = 18 feet.
3rd garden = 3 x 9 = 27 feet.
The areas increase by 9 square feet.

Spiral Review

Question 3.
Joe, Jim, and Jack share 27 football cards equally. How many cards does each boy get?
Options:
a. 7
b. 8
c. 9
d. 10

Answer:
c. 9

Explanation:
x + x + x = 27.
3x = 27.
x = 27/3 = 9.
Each boy gets 9 football cards.

Question 4.
Nita uses \(\frac{1}{3}\) of a carton of 12 eggs. How many eggs does she use?
Go Math Grade 3 Answer Key Chapter 11 Perimeter and Area Problem Solving Area of Rectangles img 65
Options:
a. 3
b. 4
c. 6
d. 9

Answer:
b. 4

Explanation:
\(\frac{1}{3}\) of a carton of 12 eggs = \(\frac{1}{3}\) x 12 = 4.
Nita uses 4 eggs.

Question 5.
Brenda made 8 necklaces. Each necklace has 10 large beads. How many large beads did Brenda use to make the necklaces?
Options:
a. 80
b. 85
c. 90
d. 100

Answer:
a. 80

Explanation:
8 x 10 = 80 beads.
Brenda uses 80 beads to make the necklaces

Question 6.
Neal is tiling his kitchen floor. Each square tile is 1 square foot. Neal uses 6 rows of tiles with 9 tiles in each row. What is the area of the floor?
Options:
a. 15 square feet
b. 52 square feet
c. 54 square feet
d. 57 square feet

Answer:
c. 54 square feet

Explanation:
The area of the floor = 6 x 9 = 54 square feet.

Area of Combined Rectangles – Page No. 673

Use the Distributive Property to find the area.
Show your multiplication and addition equations.

Question 1.
Go Math Grade 3 Answer Key Chapter 11 Perimeter and Area Area of Combined Rectangles img 66

Answer:
28 Square Units

Explanation:
Chapter 11 - area of combined rectangles - image 40

Question 2.
Go Math Grade 3 Answer Key Chapter 11 Perimeter and Area Area of Combined Rectangles img 67
_______ square units

Answer:
27 square units

Explanation:
Chapter 11 - area of combined rectangles - image 41
3 x 3 = 9; 3 x 6 = 18.
9 + 18 = 27
27 square units.

Draw a line to break apart the shape into rectangles. Find the area of the shape.

Question 3.
Go Math Grade 3 Answer Key Chapter 11 Perimeter and Area Area of Combined Rectangles img 68
Type below:
_____________

Answer:
31 square units

Explanation:
Chapter 11 - area of combined rectangles - image 42
5 x 5 = 25; 2 x 3 = 6
25 + 6 = 31
31 square units

Question 4.
Go Math Grade 3 Answer Key Chapter 11 Perimeter and Area Area of Combined Rectangles img 69
Type below:
_____________

Answer:
32 square units

Explanation:
Chapter 11 - area of combined rectangles - image 43
4 x 4 = 16; 2 x 8 = 16.
16 + 16 = 32.
32 square units

Problem Solving

A diagram of Frank’s room is at right. Each unit square is 1 square foot.
Go Math Grade 3 Answer Key Chapter 11 Perimeter and Area Area of Combined Rectangles img 70

Question 5.
Draw a line to divide the shape of Frank’s room into rectangles.

Answer:
Chapter 11 - area of combined rectangles - image 45 (2)

Question 6.
What is the total area of Frank’s room?
_______ square feet

Answer:
75 square feet

Explanation:
Chapter 11 - area of combined rectangles - image 44
6 x 5 = 30; 5 x 9 = 45
30 + 45 = 75.
75 square feet

Area of Combined Rectangles – Page No. 674

Lesson Check

Question 1.
The diagram shows Ben’s backyard. Each unit square is 1 square yard. What is the area of Ben’s backyard?
Go Math Grade 3 Answer Key Chapter 11 Perimeter and Area Area of Combined Rectangles img 71
Options:
a. 12 square yards
b. 16 square yards
c. 18 square yards
d. 24 square yards

Answer:
b. 16 square yards

Explanation:
Chapter 11 - area of combined rectangles - image 45
6 x 3 = 18 square yards

Question 2.
The diagram shows a room in an art gallery. Each unit square is 1 square meter. What is the area of the room?
Go Math Grade 3 Answer Key Chapter 11 Perimeter and Area Area of Combined Rectangles img 72
Options:
a. 24 square meters
b. 30 square meters
c. 36 square meters
d. 40 square meters

Answer:
b. 30 square meters

Explanation:
Chapter 11 - area of combined rectangles - image 46
3 x 5 = 15; 3 x 5 = 15
15 + 15 = 30 square meters

Spiral Review

Question 3.
Naomi needs to solve 28 ÷ 7 = ■. What related multiplication fact can she use to find the unknown number?
Options:
a. 3 × 7 = 21
b. 4 × 7 = 28
c. 5 × 7 = 35
d. 6 × 7 = 42

Answer:
b. 4 × 7 = 28

Explanation:
28 ÷ 7 = 4. So, 4 x 7 = 28 is the answer.

Question 4.
Karen drew a triangle with side lengths 3 centimeters, 4 centimeters, and 5 centimeters. What is the perimeter of the triangle?
Options:
a. 7 centimeters
b. 9 centimeters
c. 11 centimeters
d. 12 centimeters

Answer:
d. 12 centimeters

Explanation:
Perimeter = 3 centimeters + 4 centimeters + 5 centimeters = 12 centimeters.

Question 5.
The rectangle is divided into equal parts. What is the name of the equal parts?
Go Math Grade 3 Answer Key Chapter 11 Perimeter and Area Area of Combined Rectangles img 73
Options:
a. half
b. third
c. fourth
d. sixth

Answer:
c. fourth

Question 6.
Use an inch ruler. To the nearest half inch, how long is this line segment?
Go Math Grade 3 Answer Key Chapter 11 Perimeter and Area Area of Combined Rectangles img 74
Options:
a. 1
b. 1 \(\frac{1}{2}\)
c. 2
d. 2 \(\frac{1}{2}\)

Answer:
c. 2

Explanation:
The line segment = 2 inches.

Same Perimeter, Different Areas – Page No. 679

Find the perimeter and the area.
Tell which rectangle has a greater area.

Question 1.
Go Math Grade 3 Answer Key Chapter 11 Perimeter and Area Same Perimeter, Different Areas img 75
A: Perimeter = 12 ;
Area = 9 square units

B: Perimeter = ______ units;
Area = ______ square units;
Rectangle ______ has a greater area.

Answer:
Rectangle A has a greater area.

Explanation:
Chapter 11 - same perimeter, different areas - image 1
A: Perimeter = 12 ;
Area = 9 square units
B: Perimeter = 2 + 4 + 2 + 4 = 12
Area = 2 x 4 = 8 square units
Rectangle A has a greater area.

Question 2.
Go Math Grade 3 Answer Key Chapter 11 Perimeter and Area Same Perimeter, Different Areas img 76
A: Perimeter = ______ units;
Area = ______ square units;

B: Perimeter = ______ units;
Area = ______ square units;
Rectangle ______ has a greater area.

Answer:
Rectangle A has a greater area.

Explanation:
Chapter 11 - same perimeter, different areas - image 2

A: Perimeter = 4 + 1 + 4 + 1 = 10
Area = 4 x 1 = 4 square units
B: Perimeter = 3 + 2 + 3 + 2 = 10
Area = 3 x 2 = 6 square units
Rectangle B has a greater area.

Problem Solving

Question 3.
Tara’s and Jody’s bedrooms are shaped like rectangles. Tara’s bedroom is 9 feet long and 8 feet wide. Jody’s bedroom is 7 feet long and 10 feet wide. Whose bedroom has the greater area? Explain.
_________ ‘s bedrooms is greater.

Answer:
Tara’s bedroom has a greater area than Jody’s bedroom area.

Explanation:
Tara’s bedroom area = 9 x 8 = 72 feet.
Jody’s bedroom area = 7 x 10 = 70 feet.
72 feet > 70 feet.
Tara’s bedroom has a greater area than Jody’s bedroom area.

Question 4.
Mr. Sanchez has 16 feet of fencing to put around a rectangular garden. He wants the garden to have the greatest possible area. How long should the sides of the garden be?
Width: ______ Length: ______ feet long

Answer:
All four sides should be 4 feet long.

Explanation:
Mr. Sanchez has 16 feet of fencing to put around a rectangular garden.
Perimeter = 1 + 7 + 1 + 7 = 16 feet. Area = 1 x 7 = 7 feet.
Perimeter = 2 + 6 + 2 + 6 = 16 feet. Area = 2 x 6 = 12 feet.
Perimeter = 3 + 5 + 3 + 5 = 16 feet. Area = 3 x 5 = 15 feet.
perimeter = 4 + 4 + 4 + 4 = 16 feet. Area = 4 x 4 = 16 feet.
The area is maximum when all the four sides are 4 feet long.

Same Perimeter, Different Areas – Page No. 680

Question 1.
Which shape has a perimeter of 12 units and an area of 8 square units?
Options:
a. Go Math Grade 3 Answer Key Chapter 11 Perimeter and Area Same Perimeter, Different Areas img 77
b. Go Math Grade 3 Answer Key Chapter 11 Perimeter and Area Same Perimeter, Different Areas img 78
c. Go Math Grade 3 Answer Key Chapter 11 Perimeter and Area Same Perimeter, Different Areas img 79
d. Go Math Grade 3 Answer Key Chapter 11 Perimeter and Area Same Perimeter, Different Areas img 80

Answer:
b

Explanation:
Perimeter = 2 + 4 + 2 + 4 = 12 units.
Area = 2 x 4 = 8 square units

Question 2.
All four rectangles below have the same perimeter. Which rectangle has the greatest area?
Options:
a. Go Math Grade 3 Answer Key Chapter 11 Perimeter and Area Same Perimeter, Different Areas img 81
b. Go Math Grade 3 Answer Key Chapter 11 Perimeter and Area Same Perimeter, Different Areas img 82
c. Go Math Grade 3 Answer Key Chapter 11 Perimeter and Area Same Perimeter, Different Areas img 83
d. Go Math Grade 3 Answer Key Chapter 11 Perimeter and Area Same Perimeter, Different Areas img 84

Answer:
d

Explanation:
a. Area = 2 x 10 = 20.
b. Area = 3 x 9 = 27.
c. Area = 4 x 8 = 32.
d. Area = 5 x 7 = 35.
d has the greatest area.

Spiral Review

Question 3.
Kerrie covers a table with 8 rows of square tiles. There are 7 tiles in each row. What is the area that Kerrie covers in square units?
Options:
a. 15 square units
b. 35 square units
c. 42 square units
d. 56 square units

Answer:
d. 56 square units

Explanation:
Area = 8 x 7 = 56.
Kerrie covers 56 square units area.

Question 4.
Von has a rectangular workroom with a perimeter of 26 feet. The length of the workroom is 6 feet. What is the width of Von’s workroom?
Options:
a. 7 feet
b. 13 feet
c. 20 feet
d. 26 feet

Answer:
a. 7 feet

Explanation:
Perimeter = 26 feet.
Length = 6 feet.
Given Von has a rectangular workroom. So, two lengths and two widths are the same for a rectangle.
Perimeter of a rectangle = Length + Width + Length + Width = 6 + W + 6 + W = 26 feet.
12 + 2W = 26 feet.
2W = 26 – 12 = 14.
W = 14/2 = 7 feet.

Same Perimeter, Different Areas – Page No. 685

Find the perimeter and the area. Tell which rectangle has a greater perimeter.

Question 1.
Go Math Grade 3 Answer Key Chapter 11 Perimeter and Area Same Area, Different Perimeters img 85
A: Area = 8 square units ;
Perimeter = 18 units ;

B: Area = _______ square units;
Perimeter = _______ units;
Rectangle _______ has a greater perimeter.

Answer:
Rectangle A has a greater perimeter

Explanation:
Chapter 11 - same perimeter, different areas - image 5
A: Area = 8 square units ;
Perimeter = 18 units ;
B: Area = ___8____ square units;
Perimeter = ____12___ units;
Rectangle ___A____ has a greater perimeter.

Question 2.
Go Math Grade 3 Answer Key Chapter 11 Perimeter and Area Same Area, Different Perimeters img 86
A: Area = _______ square units;
Perimeter = _______ units;

Question 2.
Go Math Grade 3 Answer Key Chapter 11 Perimeter and Area Same Area, Different Perimeters img 87
B: Area = _______ square units;
Perimeter = _______ units;
Rectangle _______ has a greater perimeter.

Answer:
Rectangle B has a greater perimeter

Explanation:
Chapter 11 - same perimeter, different areas - image 6
Chapter 11 - same perimeter, different areas - image 7
A: Area = 12 square units ;
Perimeter = 14 units ;
B: Area = ___12____ square units;
Perimeter = ____16___ units;
Rectangle ___B____ has a greater perimeter.

Question 3.
Go Math Grade 3 Answer Key Chapter 11 Perimeter and Area Same Area, Different Perimeters img 88
A: Area = _______ square units;
Perimeter = _______ units;

Question 3.
Go Math Grade 3 Answer Key Chapter 11 Perimeter and Area Same Area, Different Perimeters img 89
B: Area = _______ square units;
Perimeter = _______ units;
Rectangle _______ has a greater perimeter.

Answer:
Rectangle B has a greater perimeter

Explanation:
Chapter 11 - same perimeter, different areas - image 8
Chapter 11 - same perimeter, different areas - image 9
A: Area = 16 square units ;
Perimeter = 16 units ;
B: Area = ___16____ square units;
Perimeter = ____20___ units;
Rectangle ___B____ has a greater perimeter.

Problem Solving

Use the tile designs for 4–5.
Go Math Grade 3 Answer Key Chapter 11 Perimeter and Area Same Area, Different Perimeters img 90

Chapter 11 - same perimeter, different areas - image 10

Question 4.
Compare the areas of Design A and Design B.
The area of Design A ________ The area of Design B

Answer:
The area of Rectangle A and Rectangle B are equal.

Explanation:
A: Area = 20 square units ;
B: Area = ___20____ square units;

Question 5.
Compare the perimeters. Which design has the greater perimeter?
The perimeter of A ________ The perimeter of B

Answer:
The perimeter of A Greater than the perimeter of B

Explanation:
A: Perimeter = 24 units ;
B: Perimeter = ____18___ units;

Same Perimeter, Different Areas – Page No. 686

Lesson Check

Question 1.
Jake drew two rectangles. Which statement is true?
Go Math Grade 3 Answer Key Chapter 11 Perimeter and Area Same Area, Different Perimeters img 91
Options:
a. The perimeters are the same.
b. The area of A is greater.
c. The perimeter of A is greater.
d. The perimeter of B is greater

Answer:
d. The perimeter of B is greater

Explanation:
Chapter 11 - same perimeter, different areas - image 11
A: Area = 6 square units ;
Perimeter = 10 units ;
B: Area = ___6____ square units;
Perimeter = ____14___ units;
The perimeter of B is greater.

Question 2.
Alyssa drew two rectangles. Which statement is true?
Go Math Grade 3 Answer Key Chapter 11 Perimeter and Area Same Area, Different Perimeters img 92
Options:
a. The perimeter of B is greater.
b. The perimeter of A is greater.
c. The area of B is greater.
d. The perimeters are the same.

Answer:
b. The perimeter of A is greater.

Explanation:
Chapter 11 - same perimeter, different areas - image 12
A: Area = 18 square units ;
Perimeter = 22 units ;
B: Area = ___18____ square units;
Perimeter = ____18___ units;
The perimeter of A is greater.

Spiral Review

Question 3.
Marsha was asked to find the value of 8 – 3 x 2. She wrote a wrong answer. Which is the correct answer?
Options:
a. 22
b. 10
c. 4
d. 2

Answer:
d. 2

Explanation:
8 – (3 x 2) = 8 – 6 = 2

Question 4.
What fraction names the point on the number line?
Go Math Grade 3 Answer Key Chapter 11 Perimeter and Area Same Area, Different Perimeters img 93
Options:
a. \(\frac{1}{4}\)
b. \(\frac{2}{3}\)
c. \(\frac{3}{4}\)
d. \(\frac{3}{1}\)

Answer:
c. \(\frac{3}{4}\)

Explanation:
From the given figure, we can write the fraction name the point on the number line = \(\frac{3}{4}\)

Question 5.
Kyle drew three line segments with these lengths: \(\frac{2}{4}\) inch, \(\frac{2}{3}\) inch, and \(\frac{2}{6}\) inch. Which list orders the fractions from least to greatest?
Options:
a. \(\frac{2}{6}, \frac{2}{4}, \frac{2}{3}\)
b. \(\frac{2}{3}, \frac{2}{4}, \frac{2}{6}\)
c. \(\frac{2}{4}, \frac{2}{3}, \frac{2}{6}\)
d. \(\frac{2}{6}, \frac{2}{3}, \frac{2}{4}\)

Answer:
c. \(\frac{3}{4}\)

Explanation:
Given fractions are \(\frac{2}{4}\) inch, \(\frac{2}{3}\) inch, and \(\frac{2}{6}\) inch.
All the fractions have the same numerator. The denominators should compare to find the answer. The smaller denominator represents the larger number. So, \(\frac{2}{6}, \frac{2}{4}, \frac{2}{3}\) is the answer.

Question 6.
On Monday, \(\frac{3}{8}\) inch of snow fell. On Tuesday, \(\frac{5}{8}\) inch of snow fell. Which statement correctly compares the snow amounts?
Options:
a. \(\frac{3}{8}=\frac{5}{8}\)
b. \(\frac{3}{8}<\frac{5}{8}\)
c. \(\frac{5}{8}<\frac{3}{8}\)
d. \(\frac{3}{8}>\frac{5}{8}\)

Answer:
b. \(\frac{3}{8}<\frac{5}{8}\)

Explanation:
Given fractions are \(\frac{3}{8}\) and \(\frac{5}{8}\).
The denominators are equal for given fractions. So, compare numerators to find the correct equation.
3 < 5. So, \(\frac{3}{8}<\frac{5}{8}\) is the correct answer.

Review/Test – Page No. 687

Question 1.
Find the perimeter of each figure on the grid. Identify the figure that have a perimeter of 14 units. Mark all that apply.
Options:
a. Go Math Grade 3 Answer Key Chapter 11 Perimeter and Area Review/Test img 94
b. Go Math Grade 3 Answer Key Chapter 11 Perimeter and Area Review/Test img 95
c. Go Math Grade 3 Answer Key Chapter 11 Perimeter and Area Review/Test img 96
d. Go Math Grade 3 Answer Key Chapter 11 Perimeter and Area Review/Test img 97

Answer:
a and c have a perimeter of 14 units.

Explanation:
a. The Perimeter of A = 14 units.
b. The perimeter of B = 16 units.
c. The perimeter of C = 14 units.
d. The perimeter of D = 16 units

Question 2.
Kim wants to put trim around a picture she drew. How many centimeters of trim does Kim need for the perimeter of the picture?
Go Math Grade 3 Answer Key Chapter 11 Perimeter and Area Review/Test img 98
_________ centimeters

Answer:
24 centimeters

Explanation:
Perimeter = 6 + 6 + 6+ 6 = 24 centimeters.

Question 3.
Sophia drew this rectangle on dot paper. What is the area of the rectangle?
Go Math Grade 3 Answer Key Chapter 11 Perimeter and Area Review/Test img 99
_________ square units

Answer:
8 square units

Explanation:
Chapter 11 - review & test - image 1. jpg
8 square units

Review/Test – Page No. 688

Question 4.
The drawing shows Seth’s plan for a fort in his backyard. Each unit square is 1 square foot.
Go Math Grade 3 Answer Key Chapter 11 Perimeter and Area Review/Test img 100
Which equations can Seth use to find the area of the fort?
Mark all that apply.
Options:
a. 4 + 4 + 4 + 4 = 16
b. 7 + 4 + 7 + 4 = 22
c. 7 + 7 + 7 + 7 = 28
d. 4 × 4 = 16
e. 7 × 7 = 49
f. 4 × 7 = 28

Answer:
b and f are correct.

Explanation:
Chapter 11 - review & test - image 2. jpg
Area = 4 × 7 = 28
Perimeter = 7 + 4 + 7 + 4 = 22.

Question 5.
Which rectangle has a number of square units for its area equal to the number of units of its perimeter?
Options:
a. Go Math Grade 3 Answer Key Chapter 11 Perimeter and Area Review/Test img 101
b. Go Math Grade 3 Answer Key Chapter 11 Perimeter and Area Review/Test img 102
c. Go Math Grade 3 Answer Key Chapter 11 Perimeter and Area Review/Test img 103
d. Go Math Grade 3 Answer Key Chapter 11 Perimeter and Area Review/Test img 104

Answer:
B rectangle has a number of square units for its area equal to the number of units of its perimeter

Explanation:
a. Area = 1 x 7 = 7 units.
Perimeter = 1 + 7 + 1 + 7 = 16 units.
b. a. Area = 4 x 4 = 16 units.
Perimeter = 4 + 4 + 4 + 4 = 16 units.
c. a. Area = 2 x 6 = 12 units.
Perimeter = 2 + 6 + 2 + 6 = 16 units.
d. a. Area = 3 x 5 = 7 units.
Perimeter = 3 + 5 + 3 + 5 = 16 units.

Question 6.
Vanessa uses a ruler to draw a square. The perimeter of the square is 12 centimeters. Select a number to complete the sentence.
Go Math Grade 3 Answer Key Chapter 11 Perimeter and Area Review/Test img 105

The square has a side length of Go Math Grade 3 Answer Key Chapter 11 Perimeter and Area Review/Test img 106 centimeters.
_________ centimeters

Answer:
3 centimeters

Explanation:
Perimeter = s + s + s + s = 12 centimeters.
4s = 12 centimeters.
s = 3 centimeters.

Review/Test – Page No. 689

Question 7.
Tomas drew two rectangles on grid paper.
Circle the words that make the sentence true.
Go Math Grade 3 Answer Key Chapter 11 Perimeter and Area Review/Test img 107
Rectangle A has an area that is Go Math Grade 3 Answer Key Chapter 11 Perimeter and Area Review/Test img 108 the area of Rectangle B, and a perimeter that is Go Math Grade 3 Answer Key Chapter 11 Perimeter and Area Review/Test img 109 the perimeter of Rectangle B.
Type below:
_____________

Answer:
Rectangle A has an area that is the same as the area of Rectangle B.
A perimeter that is less than the perimeter of Rectangle B.

Explanation:
Chapter 11 - review & test - image 3. jpg
A: Area = 3 x 4 = 12 units.
Perimeter = 3 + 4 + 3 + 4 = 14 units.
B: Area = 2 x 6 = 12 units.
Perimeter = 2 + 6 + 2 + 6 = 16 units.

Question 8.
Yuji drew this figure on grid paper. What is the perimeter of the figure?
Go Math Grade 3 Answer Key Chapter 11 Perimeter and Area Review/Test img 110
________ units

Answer:
18 units

Explanation:
chapter 11 Review 689 image 1

Question 9.
What is the area of the figure shown? Each unit square is 1 square meter.
Go Math Grade 3 Answer Key Chapter 11 Perimeter and Area Review/Test img 111
________ square meters

Answer:
13 square meters

Explanation:
Chapter 11 - review & test - image 5. jpg

Review/Test – Page No. 690

Question 10.
Shawn drew a rectangle that was 2 units wide and 6 units long. Draw a different rectangle that has the same perimeter but a different area.
Go Math Grade 3 Answer Key Chapter 11 Perimeter and Area Review/Test img 112
Type below:
_____________

Answer:
Chapter 11 Review image 1 690
Area = 2 x 6 = 12 Square Units.
Area = 4 x 4 = 16 Square Units.

Question 11.
Mrs. Rios put a wallpaper border around the room shown below. She used 72 feet of wallpaper border. What is the unknown side length? Show your work.
Go Math Grade 3 Answer Key Chapter 11 Perimeter and Area Review/Test img 113
a = ________ ft

Answer:
16 feet

Explanation:
20 + a + 6 + 8 + 14 + 8 = 72 feet.
a + 56 = 72 feet.
a = 72 – 56 feet.
a = 16 feet.

Question 12.
Elizabeth has two gardens in her yard. The first garden is 8 feet long and 6 feet wide. The second garden is half the length of the first garden. The area of the second garden is twice the area of the first garden. For numbers 12a–12d, select True or False.

First garden = 8 x 6 = 48 feet.
Second garden length = Half the length of the first garden = 4 feet.
Second Garden Area = Twice the area of the first garden = 2 x 48 = 96.
Second Garden Area Width = 4 x s = 96.
s = 96/4 = 24

a. The area of the first garden is 48 square feet.
i. True
ii. False

Answer:
i. True

Question 12.
b. The area of the second garden is 24 square feet.
i. True
ii. False

Answer:
ii. False

Question 12.
c. The width of the second garden is 12 feet.
i. True
ii. False

Answer:
ii. False

Explanation:

Question 12.
d. The width of the second garden is 24 feet.
i. True
ii. False

Answer:
i. True

Review/Test – Page No. 691

Question 13.
Marcus bought some postcards. Each postcard had a perimeter of 16 inches. Which could be one of the postcards Marcus bought? Mark all that apply.
Options:
a. Go Math Grade 3 Answer Key Chapter 11 Perimeter and Area Review/Test img 114
b. Go Math Grade 3 Answer Key Chapter 11 Perimeter and Area Review/Test img 115
c. Go Math Grade 3 Answer Key Chapter 11 Perimeter and Area Review/Test img 116
d. Go Math Grade 3 Answer Key Chapter 11 Perimeter and Area Review/Test img 117

Answer:
a and d options are correct.

Explanation:
3 + 5 + 3 + 5 = 16 inches
4 + 6 + 4 + 6 = 20 inches.
5 + 10 + 5 + 10 = 30 inches.
4 + 4 + 4 + 4 = 16 inches.

Question 14.
Anthony wants to make two different rectangular flowerbeds, each with an area of 24 square feet. He will build a wooden frame around each flowerbed. The flowerbeds will have side lengths that are whole numbers.
Part A
Each unit square on the grid below is 1 square foot. Draw two possible flowerbeds. Label each with a letter.
Go Math Grade 3 Answer Key Chapter 11 Perimeter and Area Review/Test img 118
Type below:
__________

Answer:

Chapter 11 - review & test - image 8. jpg

Question 14.
Part B
Which of the flowerbeds will take more wood to the frame?
Explain how you know.
Type below:
__________

Answer:
A Perimeter is greater than B Perimeter.

Explanation:
Area = 24.
3 x 8 = 24; 4 x 6 = 24.
A Perimeter = 8 + 3 + 8 + 3 = 22 feet.
B Perimeter = 4 + 6 + 4 + 6 = 20 feet.
A Perimeter is greater than B Perimeter.

Review/Test – Page No. 692

Question 15.
Keisha draws a sketch of her living room on grid paper. Each unit square is 1 square meter. Write and solve a multiplication equation that can be used to find the area of the living room in square meters.
Go Math Grade 3 Answer Key Chapter 11 Perimeter and Area Review/Test img 119
Type below:
__________

Answer:
40 meter.

Explanation:
Chapter 11 - review & test - image 6. jpg
4 x 10 = 40 m.

Question 16.
Mr. Wicks designs houses. He uses grid paper to plan a new house design. The kitchen will have an area between 70 square feet and 85 square feet. The pantry will have an area between 4 square feet and 15 square feet. Draw and label a diagram to show what Mr. Wicks could design. Explain how to find the total area.
Go Math Grade 3 Answer Key Chapter 11 Perimeter and Area Review/Test img 120
Type below:
__________

Answer:
78 square feet

Explanation:Chapter 11 - review & test - image 7. jpg
Are of Kitchen = 8 x 9 = 72 square feet. (70 < 72 < 85)
Area of Pantry = 3 x 2 = 6 square feet. (4 < 6 < 15)
Total area of kitchen and pantry = 72 + 6 = 78 square feet.

Conclusion

Go Math Grade 3 Answer Key Chapter 11 Perimeter and Area has step-by-step solutions for all the problems making it easy for you to understand the concepts. Practice problems from the Go Math Grade 3 Answer Key Chapter 11 Perimeter and Area Extra Practice and test your preparation standards.

Go Math Grade 6 Answer Key Chapter 7 Exponents

go-math-grade-6-chapter-7-exponents-answer-key

The solutions of Grade 6 Go Math Answer Key for Chapter 7 Exponents are available in simple PDFs here. With the help off the HMH Go Math Grade 6 Chapter 7 Exponents Answer Ley can be easily downloaded by the students by using the provided links. You can understand the concept of the standard form in this article. So, Download a free pdf of Go Math Grade 6 Answer Key Chapter 7 Exponents.

Go Math Grade 6 Answer Key Chapter 7 Exponents

Our main aim is to provide a brief explanation of all the questions. We have provided the table of contents of chapter 7 Exponents in the below section. So, once go through the topics before you start your preparation. This will help you to know in which topic you are lagging. Hence make use of the resources provided on this page and try to score good marks in the exams. After your preparation we suggest the students to test your skills by solving the questions in the mid-chapter checkpoint and review test.

Lesson 1: Exponents

Lesson 2: Evaluate Expressions Involving Exponents

Lesson 3: Write Algebraic Expressions

Lesson 4: Identify Parts of Expressions

Lesson 5: Evaluate Algebraic Expressions and Formulas

Mid-Chapter Checkpoint

Lesson 6: Use Algebraic Expressions

Lesson 7: Problem Solving • Combine Like Terms

Lesson 8: Generate Equivalent Expressions

Lesson 9: Identify Equivalent Expressions

Chapter 7 Review/Test

Share and Show – Page No. 359

Question 1.
Write 24 by using repeated multiplication. Then find the value of 24.
___________

Answer: 16

Explanation:
The repeated factor is 2
The number 2 is repeated 4 times.
The repeated multiplication of 24 is 2 × 2 × 2 × 2 = 16
Thus the value of 24 is 16.

Use one or more exponents to write the expression.

Question 2.
7 × 7 × 7 × 7
Type below:
_____________

Answer: 74

Explanation:
The repeated factor is 7.
7 is repeated four times.
The exponent of the repeated multiplication 7 × 7 × 7 × 7 is 74

Question 3.
5 × 5 × 5 × 5 × 5
Type below:
_____________

Answer: 55

Explanation:
The repeated factor is 5. The number 5 is repeated five times.
The exponent of the repeated multiplication 5 × 5 × 5 × 5 × 5 is 55

Question 4.
3 × 3 × 4 × 4
Type below:
_____________

Answer: 32 × 42

Explanation:
The exponent of the repeated multiplication 3 × 3 is 32
The exponent of the repeated multiplication 4 × 4 is 42
Thus the exponent for 3 × 3 × 4 × 4 is 32 × 42

On Your Own

Find the value.

Question 5.
202
______

Answer: 20 × 20 = 400

Explanation:
The repeated factor is 20
Write the factor 2 times.
20 × 20 = 400
The value of 202 = 400

Question 6.
821
______

Answer: 82

Explanation:
The repeated factor is 82
Write the factor 1 time.
The value of 821 is 82

Question 7.
35

Answer: 3 × 3 × 3 × 3 × 3 = 243

Explanation:
The repeated factor is 3
Write the factor 5 times.
The value of 35 is 343

Question 8.
Write 32 as a number with an exponent by using 2 as the base.
Type below:
_____________

Answer: 25

Explanation:
The exponent of 32 by using the base 2 is 2 × 2 × 2 × 2 × 2  = 25

Complete the statement with the correct exponent.

Question 9.
5? = 125
______

Answer: 53

Explanation:
The exponential form of 125 is 5 × 5 × 5 = 53
5? = 125
5? = 53
When bases are equal powers should be equated.
Thus the exponent is 3

Question 10.
16? = 16
______

Answer: 1

Explanation:
The exponential form of 16 is 161
16? = 161
When bases are equal powers should be equated.
Thus the exponent is 1.

Question 11.
30? = 900
______

Answer: 2

Explanation:
The exponential form of 900 is 30 × 30 = 302
30? = 302
When bases are equal powers should be equated.
Thus the exponent is 2.

Question 12.
Use Repeated Reasoning Find the values of 41, 42, 43, 44, and 45. Look for a pattern in your results and use it to predict the ones digit in the value of 46.
Type below:
_____________

Answer:
The value of 41 is 4.
The value of 42 is 4 × 4 = 16
The value of 43 is 4 × 4 × 4 = 64
The value of 44 is 4 × 4 × 4 × 4 = 256
The value of 45 is 4 × 4 × 4 × 4 × 4 = 1024
The value of 46 is 4 × 4 × 4 × 4 × 4 × 4 = 4096

Question 13.
Select the expressions that are equivalent to 32. Mark all that apply.
Options:
a. 25
b. 84
c. 23 × 4
d. 2 × 4 × 4

Answer: 25

Explanation:
The exponent of 32 by using the base 2 is 2 × 2 × 2 × 2 × 2  = 25
32 = 25
Thus the correct answer is option A.

Bacterial Growth – Page No. 360

Bacteria are tiny, one-celled organisms that live almost everywhere on Earth. Although some bacteria cause disease, other bacteria are helpful to humans, other animals, and plants. For example, bacteria are needed to make yogurt and many types of cheese.

Under ideal conditions, a certain type of bacterium cell grows larger and then splits into 2 “daughter” cells. After 20 minutes, the daughter cells split, resulting in 4 cells. This splitting can happen again and again as long as conditions remain ideal.

Complete the table.
Go Math Grade 6 Answer Key Chapter 7 Exponents img 1
Extend the pattern in the table above to answer 14 and 15.

Question 14.
What power of 2 shows the number of cells after 3 hours? How many cells are there after 3 hours?
Type below:
_____________

Answer: 29

Explanation:
So, each cell doubles every 20 mins. After 20 minutes, you have 1(2) = 2 cells. After 40 minutes, you have 2(2) = 4 cells, etc.
1 hour = 60 minutes
3 hours = 3 × 60 minutes = 180 minutes
180/20 = 9 divisions
Thus 29 cells are there after 3 hours.

Question 15.
How many minutes would it take to have a total of 4,096 cells?
_______ minutes

Answer: 240 minutes

Explanation:
First, convert the cells into the exponential form.
The exponential form of 4096 is 2 × 2 × 2 × 2 × 2 × 2× 2 × 2× 2 × 2× 2 × 2 = 212
Multiply the power with 20
12 × 20 = 240
Thus it would take 240 minutes to have a total of 4,096 cells

Exponents – Page No. 361

Use one or more exponents to write the expression.

Question 1.
6 × 6
Type below:
_____________

Answer:
The number 6 is used as a repeated factor.
6 is used as a factor 2 times.
Now write the base and exponent for 6 × 6 = 62

Question 2.
11 × 11 × 11 × 11
Type below:
_____________

Answer:
The number 11 is used as a repeated factor.
11 is used as a factor 4 times.
Now write the base and exponent for 11 × 11 × 11 × 11 = 114

Question 3.
9 × 9 × 9 × 9 × 7 × 7
Type below:
_____________

Answer:
The number 9 and 7 is used as a repeated factor.
9 is used as a factor 4 times and 7 is used 2 times.
Now write the base and exponent for 9 × 9 × 9 × 9 × 7 × 7 = 94 × 72

Question 4.
64
_______

Answer:
The repeated factor is 6.
Write the factor 4 times.
The value of 64 is 6 × 6 × 6 × 6 = 1296

Question 5.
16
_______

Answer:
The repeated factor is 1.
Write the factor 6 times.
The value of 16 is 1 × 1 × 1 × 1 × 1 × 1 = 1

Question 6.
105
_______

Answer:
The repeated factor is 10.
Write the factor 5 times.
The value of 105 is 10 × 10 × 10 × 10 × 10 = 1,00,000

Question 7.
Write 144 with an exponent by using 12 as the base.
Type below:
_____________

Answer: 12 × 12 = 122
The exponential form of 144 is 12 × 12 = 122

Question 8.
Write 343 with an exponent by using 7 as the base.
Type below:
_____________

Answer: The exponential form of 343 is 7 × 7 × 7 = 73

Question 9.
Each day Sheila doubles the number of push-ups she did the day before. On the fifth day, she does 2 × 2 × 2 × 2 × 2 push-ups. Use an exponent to write the number of push-ups Shelia does on the fifth day.
Type below:
_____________

Answer:
The number 2 is the repeated factor.
2 is repeated 5 times.
The exponential form of 2 × 2 × 2 × 2 × 2 is 25

Question 10.
The city of Beijing has a population of more than 107 people. Write 107 without using an exponent.
_______

Answer:
The repeated factor is 10.
Write the factor 7 times.
The value of 107 is 10 × 10 × 10 × 10 × 10 × 10 × 10 = 10,000,000

Question 11.
Explain what the expression 45 means and how to find its value.
Type below:
_____________

Answer:
The repeated factor is 4.
Write the factor 5 times.
The value of 45 is 4 × 4 × 4 × 4 × 4 = 1024

Lesson Check – Page No. 362

Question 1.
The number of games in the first round of a chess tournament is equal to 2 × 2 × 2 × 2 × 2 × 2. Write the number of games using an exponent.
Type below:
_____________

Answer: 26

Explanation:

The number 2 is the repeated factor.
2 is repeated 6 times.
2 × 2 × 2 × 2 × 2 × 2 = 26

Question 2.
The number of gallons of water in a tank at an aquarium is equal to 83. How many gallons of water are in the tank?
_______ gallons

Answer: 512 gallons

Explanation:
The repeated factor is 8.
Write the factor 3 times.
The value of 83 is 8 × 8 × 8 = 512 gallons
Therefore there are 512 gallons of water in the tank.

Spiral Review

Question 3.
The table shows the amounts of strawberry juice and lemonade needed to make different amounts of strawberry lemonade. Name another ratio of strawberry juice to lemonade that is equivalent to the ratios in the table.
Go Math Grade 6 Answer Key Chapter 7 Exponents img 2
Type below:
_____________

Answer: 5 : 15

Explanation:
By using the above table we can find the ratio of strawberry juice to lemonade.
2 : 6 = 1 : 3
The ratio of strawberry juice to lemonade next to 4 : 12 is 5 : 15

Question 4.
Which percent is equivalent to the fraction \(\frac{37}{50}\)?
_______ %

Answer: 74%

Explanation:
\(\frac{37}{50}\) × 100
0.74 × 100 = 74
Thus 74% is equivalent to the fraction \(\frac{37}{50}\)

Question 5.
How many milliliters are equivalent to 2.7 liters?
_______ milliliters

Answer: 2700 milliliters

Explanation:
Convert from liters to milliliters.
1 liter = 1000 milliliters
2.7 liters = 2.7 × 1000 milliliters = 2700 milliliters
2.7 liters is equivalent to 2700 milliliters.

Question 6.
Use the formula d = rt to find the distance traveled by a car driving at an average speed of 50 miles per hour for 4.5 hours.
_______ miles

Answer: 225 miles

Explanation:
Given,
r = 50 miles/hour
t = 4.5 hours
Use the formula d = rt
d = 50 × 4.5 = 225 miles
Thus the distance traveled by a car driving at an average speed of 50 miles per hour for 4.5 hours is 225 miles.

Share and Show – Page No. 365

Question 1.
Evaluate the expression 9 + (52 − 10)
_______

Answer: 24

Explanation:
First write the square for 52
52 is 25
Now simplify the expression 9 + (25 – 10)
9 + 15 = 24
So, 9 + (52 − 10) = 24

Evaluate the expression.

Question 2.
6 + 33 ÷ 9
_______

Answer: 9

Explanation:
6 + 33 ÷ 9
6 + (33 ÷ 9)
Write the factor for 33
33 = 3 × 3 × 3 = 27
6 + (27 ÷ 9)
27 ÷ 9 = 3
6 + 3 = 9
Thus 6 + 33 ÷ 9 = 9

Question 3.
(15 − 3)2 ÷ 9
_______

Answer: 16

Explanation:
First subtract 15 – 3 = 12
(12)2 ÷ 9
(12)2 = 12 × 12 = 144
144 ÷ 9
9 divides 144 16 times.
144 ÷ 9 = 16
Thus (15 − 3)2 ÷ 9 = 16

Question 4.
(8 + 92) − 4 × 10
_______

Answer: 49

Explanation:
First multiply 9 × 9 = 81
(8 + 81) – (4 × 10)
Multiply 4 and 10.
4 × 10 = 40
(8 + 81) – (40)
89 – 40 = 49
(8 + 92) − 4 × 10 = 49

On Your Own

Evaluate the expression

Question 5.
10 + 62 × 2 ÷ 9
_______

Answer: 18

Explanation:
10 + (62 × 2) ÷ 9
Multipley 6 × 6 = 36
10 + (36 × 2) ÷ 9
Multiply 36 and 2 and then divide by 9.
10 + (72 ÷ 9)
10 + 8 = 18
So, 10 + 62 × 2 ÷ 9 = 18

Question 6.
62 − (23 + 5)
_______

Answer: 23

Explanation:

The value of 62 is 6 × 6 = 36
The value of 23 is 2 × 2 × 2 = 8
36 – (8 + 5)
36 – 13 = 23
Thus the answer for the expression for 62 − (23 + 5) is 23.

Question 7.
16 + 18 ÷ 9 + 34
_______

Answer: 99

Explanation:
16 + (18 ÷ 9) + 34
First divide 18 by 9
16 + 2 + 34
18 + 34
The value of 34 is 3 × 3 × 3 × 3 = 81
18 + 81 = 99
Thus the answer for the expression 16 + (18 ÷ 9) + 34 is 99.

Place parentheses in the expression so that it equals the given value.

Question 8.
102 − 50 ÷ 5
value: 10
Type below:
_____________

Answer: 10

Explanation:
102 − 50 ÷ 5
The factor of 102 is 10 × 10 = 100
(102 − 50) ÷ 5
50 ÷ 5 = 10
102 − 50 ÷ 5 = 10
The value of 102 − 50 ÷ 5 = 10

Question 9.
20 + 2 × 5 + 41
value: 38
Type below:
_____________

Answer: 38

Explanation:
20 + 2 × 5 + 41
The value of 41 is 4.
20 + 2 × (5 + 4)
20 + 2 × 9
Now multiply 2 and 9.
20 + 18 = 38
The value of 20 + 2 × 5 + 41 = 38

Question 10.
28 ÷ 22 + 3
value: 4
Type below:
_____________

Answer: 4

Explanation:
28 ÷ 22 + 3
28 ÷ (22 + 3)
The value of 22 is 4
28 ÷ (4 + 3)
28 ÷ 7 = 4
The value of 28 ÷ 22 + 3 is 4.

Problem Solving + Applications – Page No. 366

Use the table for 11–13.
Go Math Grade 6 Answer Key Chapter 7 Exponents img 3

Question 11.
Write an Expression To find the cost of a window, multiply its area in square feet by the price per square foot. Write and evaluate an expression to find the cost of a knot window
$ _______

Answer: 108

Explanation:
To find the cost of the knot window multiply the area with the price per square foot.
Area per square feet is 22
Price per square foot is $27
Cost = 22 × 27 = 4 × 27 = 108
Thus the cost of a knot window is $108

Question 12.
A builder installs 2 rose windows and 2 tulip windows. Write and evaluate an expression to find the combined area of the windows.
_______ square feet

Answer: 50

Explanation:
The area of rose window is 32
The area of tulip windows is 22
Combines area of rose and tulip window is 32 + 22 = 52
52 × 2 = 25 × 2 = 50
Thus the area of the combined windows is 50 square feet.

Question 13.
DeShawn bought a tulip window. Emma bought a rose window. Write and evaluate an expression to determine how much more DeShawn paid for his window than Emma paid for hers.
$ _______

Answer: 258

Explanation:
Given that, DeShawn bought a tulip window.
DeShawn bought it for 42 × $33 = 16 × $33 = 528
Emma bought a rose window
Emma bought it for 32 × 30 = 9 × 30 = 270
$528 – $270 = $258
DeShawn paid $258 for his window than Emma paid for hers.

Question 14.
What’s the Error? Darius wrote 17 − 22 = 225. Explain his error.
Type below:
_____________

Answer: 17 – 4 is actually 13 but not 225.

Question 15.
Ms. Hall wrote the expression 2 × (3 + 5)2÷ 4 on the board. Shyann said the first step is to evaluate 52. Explain Shyann’s mistake. Then evaluate the expression
_______

Answer: 32

Explanation:
2 × (3 + 5)2÷ 4
First, add 3 and 5.
2 × (8)2÷ 4
The square of 8 × 8 is 64.
2 × (64 ÷ 4) = 2 × 16 = 32

Evaluate Expressions Involving Exponents – Page No. 367

Evaluate the expression.

Question 1.
5 + 17 − 102 ÷ 5
_______

Answer: 2

Explanation:
5 + 17 – (100 ÷ 5)
Divide 100 by 5
(5 + 17) – 20
22 – 20 = 2
So, the value for the expression 5 + 17 − 102 ÷ 5 = 2

Question 2.
72 − 32 × 4
_______

Answer: 13

Explanation:
72 − 32 × 4
72 − (32 × 4)
72 − (9 × 4)
49 – 36 = 13
Thus, 72 − 32 × 4 = 13

Question 3.
24 ÷ (7 − 5)
_______

Answer: 8

Explanation:
24 ÷ (7 − 5)
24 ÷ 2
24 = 2 × 2 × 2 × 2 = 16
16 ÷ 2 = 8
24 ÷ (7 − 5) = 8

Question 4.
(82 + 36) ÷ (4 × 52)
_______

Answer: 1

Explanation:
(82 + 36) ÷ (4 × 52)
82 = 8 × 8 = 64
52 = 5 × 5 = 25
(64 + 36) ÷ (4 × 25)
100 ÷ 100 = 1
So, (82 + 36) ÷ (4 × 52) = 1

Question 5.
12 + 21 ÷ 3 + (22 × 0)
_______

Answer: 19

Explanation:
12 + 21 ÷ 3 + 0
12 + (21 ÷ 3)
12 + 7 = 19
12 + 21 ÷ 3 + (22 × 0) = 19

Question 6.
(12 − 8)3 − 24 × 2
_______

Answer: 16

Explanation:
(12 − 8)3 − 24 × 2 = (4)3 − 24 × 2
64 – (24 × 2)
= 64 – 48 = 16
(12 − 8)3 − 24 × 2 = 16

Place parentheses in the expression so that it equals the given value.

Question 7.
12 × 2 + 23
value: 120
Type below:
_____________

Answer:
12 × (2 + 23)
12 × (2 + 8)
12 × 10 = 120
12 × 2 + 23 = 120

Question 8.
72 + 1 − 5 × 3
value: 135
Type below:
_____________

Answer:
(72 + 1 − 5) × 3
(49 + 1 – 5) × 3
(50 – 5) × 3
45 × 3 = 135
72 + 1 − 5 × 3 = 135

Problem Solving

Question 9.
Hugo is saving for a new baseball glove. He saves $10 the first week, and $6 each week for the next 6 weeks. The expression 10 + 62 represents the total amount in dollars he has saved. What is the total amount Hugo has saved?
$ _______

Answer: $46

Explanation:
Hugo is saving for a new baseball glove.
He saves $10 the first week, and $6 each week for the next 6 weeks.
The expression 10 + 62 represents the total amount in dollars he has saved.
10 + 62 = 10 + 36 = 46
total amount Hugo has saved is $46

Question 10.
A scientist placed 5 fish eggs in a tank. Each day, twice the number of eggs from the previous day hatch. The expression 5 × 26 represents the number of eggs that hatch on the seventh day. How many eggs hatch on the seventh day?
_______ eggs

Answer: 320 eggs

Explanation:
A scientist placed 5 fish eggs in a tank.
Each day, twice the number of eggs from the previous day hatch.
The expression 5 × 26 represents the number of eggs that hatch on the seventh day.
5 × 26 = 5 × 64 = 320 eggs
Therefore 320 eggs hatch on the seventh day.

Question 11.
Explain how you could determine whether a calculator correctly performs the order of operations.
Type below:
_____________

Answer: Create a problem that must use the order of operations and isn’t solved by just left to right. Solve it going left to right. Then solve it using the order of operations. Solve it on the calculator. Your answer on the calculator will match the one using the order of operations.

Lesson Check – Page No. 368

Question 1.
Ritchie wants to paint his bedroom ceiling and four walls. The ceiling and each of the walls are 8 feet by 8 feet. A gallon of paint covers 40 square feet. Write an expression that can be used to find the number of gallons of paint Ritchie needs to buy.
Type below:
_____________

Answer:
Ritchie wants to paint his bedroom ceiling and four walls.
The ceiling and each of the walls are 8 feet by 8 feet.
A gallon of paint covers 40 square feet.
8 × 8 × (4 + 1) ÷ 40
82 (4 + 1) ÷ 40
Thus the expression that can be used to find the number of gallons of paint Ritchie needs to buy is 82 (4 + 1) ÷ 40

Question 2.
A Chinese restaurant uses about 225 pairs of chopsticks each day. The manager wants to order a 30-day supply of chopsticks. The chopsticks come in boxes of 750 pairs. How many boxes should the manager order?
_______ boxes

Answer: 9 boxes

Explanation:
A Chinese restaurant uses about 225 pairs of chopsticks each day.
The manager wants to order a 30-day supply of chopsticks.
Multiply the number of pairs with the number of days
225 × 30 = 6750
The chopsticks come in boxes of 750 pairs.
Now divide the number of chopsticks by the number of pairs.
6750 ÷ 750 = 9 boxes.

Spiral Review

Question 3.
Annabelle spent $5 to buy 4 raffle tickets. How many tickets can she buy for $20?
_______ tickets

Answer: 16 tickets

Explanation:
Annabelle spent $5 to buy 4 raffle tickets.
To find the number of tickets she can buy for $20.
($20 ÷ $5) × 4
4 × 4 = 16 tickets
That means she can buy 16 tickets for $20.

Question 4.
Gavin has 460 baseball players in his collection of baseball cards, and 15% of the players are pitchers. How many pitchers are in Gavin’s collection?
_______ pitchers

Answer: 69 pitchers

Explanation:
Gavin has 460 baseball players in his collection of baseball cards, and 15% of the players are pitchers.
The decimal form of 15% is 0.15
Now multiply 460 with 0.15
460 × 0.15 = 69.00
Thus there are 69 pitchers in Gavin’s collection.

Question 5.
How many pounds are equivalent to 40 ounces?
_______ pounds

Answer: 2.5 pounds

Explanation:
Convert from ounces to pounds.
1 pound = 16 ounces
1 ounce = 1/16 pound
40 ounces = 40 × 1/16 pound
40 ounces = 2.5 pounds
Thus, 2.5 pounds are equivalent to 40 ounces

Question 6.
List the expressions in order from least to greatest.
15 33 42 81
Type below:
_____________

Answer:
15 33 42 81
15 = 1 × 1 × 1 × 1 × 1 = 1
33 = 3 × 3 × 3 = 27
42 = 4 × 4 = 16
81 = 8
Thus the order from least to greatest.
15 81 42 33

Share and Show – Page No. 371

Question 1.
Write an algebraic expression for the product of 6 and p.
What operation does the word “product” indicate?
Type below:
_____________

Answer: 6 × p
Explanation:
The word product indicates multiplication.
Multiply 6 with p.
The algebraic expression for the product of 6 and p is 6 × p.

Write an algebraic expression for the word expression.

Question 2.
11 more than e
Type below:
_____________

Answer: 11 + e

Explanation:
The word more than indicates addition operation.
So, the algebraic expression is 11 + e

Question 3.
9 less than the quotient of n and 5
Type below:
_____________

Answer: 9 – (n ÷ 5)

Explanation:
The word “less than” indicates subtraction and the “quotient” indicates division.
So, the expression is 9 – (n ÷ 5)

On Your Own

Write an algebraic expression for the word expression.

Question 4.
20 divided by c
Type below:
_____________

Answer: 20 ÷ c

Explanation:
Here we have to divide 20 by c.
The expression is 20 ÷ c

Question 5.
8 times the product of 5 and t
Type below:
_____________

Answer: 8 × (5t)

Explanation:
The word times indicate multiplication and the product indicates multiplication.
Here we have to multiply 8 with 5 and t.
Thus the expression is 8 × 5 × t = 8 × 5t

Question 6.
There are 12 eggs in a dozen. Write an algebraic expression for the number of eggs in d dozen.
Type below:
_____________

Answer: 12d

Explanation:
Given,
There are 12 eggs in a dozen.
d represents number of eggs in dozen
So, we have to multiply 12 with d.
Thus the algebraic expression is 12d.

Question 7.
A state park charges a $6.00 entry fee plus $7.50 per night of camping. Write an algebraic expression for the cost in dollars of entering the park and camping for n nights.
Type below:
_____________

Answer: $6.00 + $7.50 n

Explanation:
Given that, A state park charges a $6.00 entry fee plus $7.50 per night of camping.
Find the camping for n nights. The product of $7.50 camping for n nights.
$7.50 × n
Now add park charges to the camping nights.
$6.00 + $7.50 n
Thus the algebraic expression for the cost in dollars of entering the park and camping for n nights is $6.00 + $7.50 n

Question 8.
Look for Structure At a bookstore, the expression 2c + 8g gives the cost in dollars of c comic books and g graphic novels. Next month, the store’s owner plans to increase the price of each graphic novel by $3. Write an expression that will give the cost of c comic books and g graphic novels next month.
Type below:
_____________

Answer: 2c + 11g

Explanation:
Look for Structure At a bookstore, the expression 2c + 8g gives the cost in dollars of c comic books and g graphic novels.
Next month, the store’s owner plans to increase the price of each graphic novel by $3.
Here we have to add $3 to 8 g = 3g + 8g = 11g
Sum of cost of c comic books and g graphic novels
Thus the expression is 2c + 11g

Unlock the Problem – Page No. 372

Question 9.
Martina signs up for the cell phone plan described at the right. Write an expression that gives the total cost of the plan in dollars if Martina uses it for m months.
Go Math Grade 6 Answer Key Chapter 7 Exponents img 4
a. What information do you know about the cell phone plan?
Type below:
_____________

Answer: Pay a low monthly fee of $50. Receive $10 off your first month’s fee.

Question 9.
b. Write an expression for the monthly fee in dollars for m months.
Type below:
_____________

Answer:
M is the number of months.
50 × m
Given that $10 off on first-month fee.
50m + (50-10)
50m + $40

Question 9.
c. What operation can you use to show the discount of $10 for the first month?
Type below:
_____________

Answer: We have to use subtraction operations to show a discount of $10 for the first month.

Question 9.
d. Write an expression for the total cost of the plan in dollars for m months
Type below:
_____________

Answer: 50m + 40

Question 10.
A group of n friends evenly share the cost of dinner. The dinner costs $74. After dinner, each friend pays $11 for a movie. Write an expression to represent what each friend paid for dinner and the movie.
Type below:
_____________

Answer: 74 ÷ n + 11n

Explanation:
Given,
A group of n friends evenly share the cost of dinner.
The dinner costs $74. After dinner, each friend pays $11 for a movie.
The word share represents the division operation.
That means we have to divide 74 by n.
74 ÷ n
After that n friends paid $11 for movie
Multiply 11 with n.
Thus the expression to represent what each friend paid for dinner and the movie is 74 ÷ n + 11n

Question 11.
A cell phone company charges $40 per month plus $0.05 for each text message sent. Select the expressions that represent the cost in dollars for one month of cell phone usage and sending m text messages. Mark all that apply.
Options:
a. 40m + 0.05
b. 40 + 0.05m
c. 40 more than the product of 0.05 and m
d. the product of 40 and m plus 0.05

Answer: 40 + 0.05m

Explanation:
A cell phone company charges $40 per month plus $0.05 for each text message sent.
Let m represent the messages sent.
40m + 0.05m
Thus the answer is option B.

Write Algebraic Expressions – Page No. 373

Write an algebraic expression for the word expression.

Question 1.
13 less than p
Type below:
_____________

Answer: 13 – p

Explanation:
Less than is nothing but subtraction.
So the expression for 13 less than p is 13 – p

Question 2.
the sum of x and 9
Type below:
_____________

Answer: x + 9

Explanation:
The sum is nothing but an addition.
Thus the expression for the sum of x and 9 is x + 9.

Question 3.
6 more than the difference of b and 5
Type below:
_____________

Answer: 6 + (b – 5)

Explanation:
More than is nothing but addition and difference means subtraction.
The expression for 6 more than the difference of b and 5 is 6 + (b – 5)

Question 4.
the sum of 15 and the product of 5 and v
Type below:
_____________

Answer: 15 + 5v

Explanation:
Product is nothing but multiplication and sum is nothing but an addition.
So, the expression for the sum of 15 and the product of 5 and v is 15 + 5 × v

Question 5.
the difference of 2 and the product of 3 and k
Type below:
_____________

Answer: 2 – 3k

Explanation:
The difference means subtraction and Product is nothing but the multiplication
So, the difference of 2 and the product of 3 and k is 2 – 3 × k
2 – 3k

Question 6.
12 divided by the sum of h and 2
Type below:
_____________

Answer: 12 ÷ h + 2

Explanation:
12 divided by the sum of h and 2
Divide 12 by sum of h and 2.
12 ÷ (h + 2)

Question 7.
the quotient of m and 7
Type below:
_____________

Answer: m ÷ 7

Explanation:
Given the quotient of m and 7
That means we have to divide m by 7.
Thus the answer is m ÷ 7

Question 8.
9 more than 2 multiplied by f
Type below:
_____________

Answer: 9 + 2f

Explanation:
9 more than 2 multiplied by f
We have to add 9 to 2 × f
So, the expression is 9 + 2f

Question 9.
6 minus the difference of x and 3
Type below:
_____________

Answer: 6 – (x – 3)

Explanation:
First, subtract 3 from x
The expression for 6 minus the difference of x and 3 is 6 – (x – 3)

Question 10.
10 less than the quotient of g and 3
Type below:
_____________

Answer: 10 – (g ÷ 3)

Explanation:
The quotient of g and 3 is nothing but divide g by 3
g ÷ 3
Now subtract g ÷ 3 from 10.
So, the expression for 10 less than the quotient of g and 3 is 10 – (g ÷ 3)

Question 11.
the sum of 4 multiplied by a and 5 multiplied by b
Type below:
_____________

Answer: 4a + 5b

Explanation:
First, multiply 4 with a and then multiply 5 with b
After that add both the expressions.
4a + 5b
So, the sum of 4 multiplied by a and 5 multiplied by b is 4a + 5b

Question 12.
14 more than the difference of r and s
Type below:
_____________

Answer: 14 + (r – s)

Explanation:
Subtract r and s
And then add 14 to that r -s
14 + (r – s)

Problem Solving

Question 13.
Let h represent Mark’s height in inches. Suzanne is 7 inches shorter than Mark. Write an algebraic expression that represents Suzanne’s height in inches.
Type below:
_____________

Answer: h – 7

Explanation:
Let h represent Mark’s height in inches. Suzanne is 7 inches shorter than Mark.
That means we have to subtract 7 from h.
i.e., h – 7
Thus Suzanne’s height is h – 7 inches.

Question 14.
A company rents bicycles for a fee of $10 plus $4 per hour of use. Write an algebraic expression for the total cost in dollars for renting a bicycle for h hours.
Type below:
_____________

Answer: 10 + 4h

Explanation:
A company rents bicycles for a fee of $10 plus $4 per hour of use.
Multiply 4 with hours
And then 10 to 4h
10 + 4h
Thus the total cost in dollars for renting a bicycle for h hours is 10 + 4h

Question 15.
Give an example of a real-world situation involving two unknown quantities. Then write an algebraic expression to represent the situation.
Type below:
_____________

Answer:
Cooper bikes so many miles per day and does it for 7 months.
The expression for the question is 6m × 7

Lesson Check – Page No. 374

Question 1.
The female lion at a zoo weighs 190 pounds more than the female cheetah. Let c represent the weight in pounds of the cheetah. Write an expression that gives the weight in pounds of the lion.
Type below:
_____________

Answer: c + 190

Explanation:
Given that, The female lion at a zoo weighs 190 pounds more than the female cheetah.
Let c represent the weight in pounds of the cheetah.
We have to add 190 to the weight in pounds of the cheetah.
That means c + 190
Thus the expression that gives the weight in pounds of the lion is c + 190.

Question 2.
Tickets to a play cost $8 each. Write an expression that gives the ticket cost in dollars for a group of g girls and b-boys.
Type below:
_____________

Answer: 8 × (g + b)

Explanation:
First add girls group and boys group.
g + b
And then multiply 8 with the group of girls and boys.
8 × (g + b)
So, the expression that gives the ticket cost in dollars for a group of g girls and b-boys is 8 × (g + b).

Spiral Review

Question 3.
A bottle of cranberry juice contains 32 fluid ounces and costs $2.56. What is the unit rate?
$ _______ per fluid ounce

Answer: 0.08

Explanation:
A bottle of cranberry juice contains 32 fluid ounces and costs $2.56.
Divide the number of fluid ounces by the cost.
32 ÷ $2.56
32/2.56 = 0.08
The unit rate is 0.08 per fluid ounce.

Question 4.
There are 32 peanuts in a bag. Elliott takes 25% of the peanuts from the bag. Then Zaire takes 50% of the remaining peanuts. How many peanuts are left in the bag?
_______ peanuts

Answer: 12

Explanation:
First, we have to find 25% of 32.
25% of 32 its 0.25 × 32=8
Now we have to subtract 32 and 8
32 – 8=24
Now we have to find 50% of 24
50% of 24 = 12
24-12=12.
Thus 12 peanuts are left in the bag.

Question 5.
Hank earns $12 per hour for babysitting. How much does he earn for 15 hours of babysitting?
$ _______

Answer: 180

Explanation:
Hank earns $12 per hour for babysitting.
Multiply $12 with 15
12 × 15 = $180
He earned $180 for 15 hours of babysitting.

Question 6.
Write an expression using exponents that represent the area of the figure in square centimeters
Go Math Grade 6 Answer Key Chapter 7 Exponents img 5
Type below:
_____________

Answer: 72 – 22

Explanation:
The area of the square is 7 cm × 7 cm = 72
The area of the square is 2 cm × 2 cm = 22
Now subtract a small square from the large square.
The expression that represents the area of the figure is 72 – 22

Share and Show – Page No. 377

Identify the parts of the expression. Then, write a word expression for the numerical or algebraic expression.

Question 1.
7 × (9 ÷ 3)
Type below:
_____________

Answer:
The quotient of 9 and 3 and then multiply with 7.
Word expression: Product of 7 with the quotient of 9 and 3.

Question 2.
5m + 2n
Type below:
_____________

Answer:
Product of 5 and m and product of 2 and n
Now add both the product of 5 and m and 2 and n.
Word Expression: Product of 5 and m plus the product of 2 and n.

On Your Own

Practice: Copy and Solve Identify the parts of the expression. Then write a word expression for the numerical or algebraic expression.

Question 3.
8 + (10 − 7)
Type below:
_____________

Answer:
Subtraction is the difference between 10 and 7. Addition to the subtraction of 10 and 7.
Word expression: Add 8 to the difference between 10 and 7.

Question 4.
1.5 × 6 + 8.3
Type below:
_____________

Answer:
The addition is the sum of 6 and 8.3 and then multiply the sum to 1.5.
Word expression: 1.5 times the sum of 6 and 8.3

Question 5.
b + 12x
Type below:
_____________

Answer:
Product of 12 and x. Add b to the product of 12 and x.
Word expression: Sum of b to the product of 12 and x.

Question 6.
4a ÷ 6
Type below:
_____________

Answer:
The division is the quotient of 4a and 6. Multiply 4 and a. The expression is the product of 4 and a divided by 6.
Word expression: The quotient of the product 4 and a and 6.

Identify the terms of the expression. Then, give the coefficient of each term.

Question 7.
k − \(\frac{1}{3}\)d
Type below:
_____________

Answer:
The terms of the expression are k and \(\frac{1}{3}\)d
Coefficients – 1 and \(\frac{1}{3}\)

Question 8.
0.5x + 2.5y
Type below:
_____________

Answer:
The terms of the expression are 0.5x and 2.5y
Coefficients – 0.5 and 2.5

Question 9.
Connect Symbols and Words Ava said she wrote an expression with three terms. She said the first term has the coefficient 7, the second term has the coefficient 1, and the third term has the coefficient 0.1. Each term involves a different variable. Write an expression that could be the expression Ava wrote
Type below:
_____________

Answer:
Connect Symbols and Words Ava said she wrote an expression with three terms.
She said the first term has the coefficient 7, the second term has the coefficient 1, and the third term has the coefficient 0.1.
The expression for the first term is 7x
The expression for the second term is 1y
The expression for the third term is 0.1z
7x + y + 0.1z

Problem Solving + Applications – Page No. 378

Use the table for 10–12.
Go Math Grade 6 Answer Key Chapter 7 Exponents img 6

Question 10.
A football team scored 2 touchdowns and 2 extra points. Their opponent scored 1 touchdown and 2 field goals. Write a numerical expression for the points scored in the game.
Type below:
_____________

Answer:
A football team scored 2 touchdowns and 2 extra points.
2 touchdowns = 2 × 6
2 extra points = 2 × 1
Their opponent scored 1 touchdown and 2 field goals.
1 touchdown = 1 × 6
2 field goals = 2 × 3
Thue the numerical expression is 12 + 2 + 6 + 6
14 + 12
The numerical expression for the points scored in the game is 14 + 12.

Question 11.
Write an algebraic expression for the number of points scored by a football team that makes t touchdowns, f field goals, and e extra points
Type below:
_____________

Answer: 6t + 3f + e

Explanation:
The number of points scored by a football team that makes t touchdowns, f field goals, and e extra points.
The table shows that touchdown has 6 points, field goal has 3 points and extra point has 1 point.
So we need to add all the points to make the expressions
That means 6t + 3f + e

Question 12.
Identify the parts of the expression you wrote in Exercise 11.
Type below:
_____________

Question 13.
Give an example of an expression involving multiplication in which one of the factors is a sum. Explain why you do or do not need parentheses in your expression
Type below:
_____________

Answer: 6 × 2 + 3
In this expression, there is no need for parentheses because there are no exponents or multiple operations.

Question 14.
Kennedy bought a pounds of almonds at $5 per pound and p pounds of peanuts at $2 per pound. Write an algebraic expression for the cost of Kennedy’s purchase.
Type below:
_____________

Answer: 5 + 2p = x

Explanation:
Kennedy bought a pounds of almonds at $5 per pound and p pounds of peanuts at $2 per pound.
We have to multiply p with $2 per pound.
The algebraic expression for the cost of Kennedy’s purchase is the sum of 5 and the product of p and 2
Thus the expression is 5 + 2p = x

Identify Parts of Expressions – Page No. 379

Identify the parts of the expression. Then write a word expression for the numerical or algebraic expression.

Question 1.
(16 − 7) ÷ 3
Type below:
_____________

Answer:
Subtraction is the difference between 16 and 7. The division is the quotient of the difference and 3
Word expression: the quotient of the difference 16 and 7 and 3.

Question 2.
8 + 6q + q
Type below:
_____________

Answer:
Sum of 8 and the product of 6 and q added to q.
Addition – Sum of 8 plus the product of 6 and q plus q.
Addition – 6 times q and the sum of q.
Multiply – the product of 6 and q.

Identify the terms of the expression. Then give the coefficient of each term.

Question 3.
11r + 7s
Type below:
_____________

Answer:
The terms of the expression are 11r and 7s
The coefficient of each term is 11 and 7.

Question 4.
6g − h
Type below:
_____________

Answer:
The terms of the expression are 6g and h
The coefficient of each term is 6 and 1.

Problem Solving

Question 5.
Adam bought granola bars at the store. The expression 6p + 5n gives the number of bars in p boxes of plain granola bars and n boxes of granola bars with nuts. What are the terms of the expression?
Type below:
_____________

Answer:
Adam bought granola bars at the store.
The expression 6p + 5n gives the number of bars in p boxes of plain granola bars and n boxes of granola bars with nuts.
The terms of the expression are 6p and 5n.

Question 6.
In the sixth grade, each student will get 4 new books. There is one class of 15 students and one class of 20 students. The expression 4 × (15 + 20) gives the total number of new books. Write a word expression for the numerical expression.
Type below:
_____________

Answer:
In the sixth grade, each student will get 4 new books.
There is one class of 15 students and one class of 20 students.
The expression 4 × (15 + 20) gives the total number of new books.
The product of 4 the sum of 15 and 20.

Question 7.
Explain how knowing the order of operations helps you write a word expression for a numerical or algebraic expression.
Type below:
_____________

Answer: Because if you don’t know and use the order of operations you can get an entirely different answer.

Lesson Check – Page No. 380

Question 1.
A fabric store sells pieces of material for $5 each. Ali bought 2 white pieces and 8 blue pieces. She also bought a pack of buttons for $3. The expression 5 × (2 + 8) + 3 gives the cost in dollars of Ali’s purchase. How can you describe the term (2 + 8) in words?
Type below:
_____________

Answer: the sum of 2 and 8

Explanation:
A fabric store sells pieces of material for $5 each.
Ali bought 2 white pieces and 8 blue pieces.
She also bought a pack of buttons for $3.
The expression 5 × (2 + 8) + 3 gives the cost in dollars of Ali’s purchase.
The word expression for the term 2 + 8 is the sum of 2 and 8.

Question 2.
A hotel offers two different types of rooms. The expression k + 2f gives the number of beds in the hotel where k is the number of rooms with a king-size bed and f is the number of rooms with 2 full-size beds. What are the terms of the expression?
Type below:
_____________

Answer: k and 2f

Explanation:
The terms for the expression k + 2f is k and 2f.

Spiral Review

Question 3.
Meg paid $9 for 2 tuna sandwiches. At the same rate, how much does Meg pay for 8 tuna sandwiches?
$ _______

Answer: 36

Explanation:
Meg paid $9 for 2 tuna sandwiches.
To find how much does Meg pay for 8 tuna sandwiches
2 – $9
8 -?
$9 × 8/2 = 72/2 = 36
Thus Meg pays $36 for 8 tuna sandwiches.

Question 4.
Jan is saving for a skateboard. She has saved $30 already, which is 20% of the total price. How much does the skateboard cost?
$ _______

Answer: 150

Explanation:
Jan is saving for a skateboard. She has saved $30 already, which is 20% of the total price.
Divide $30 by 20%
30 ÷ 20%
30 ÷ 20 × 1/100
30 ÷ 1/5
30 × 5 = 150
Thus the cost of the skateboard is $150.

Question 5.
It took Eduardo 8 hours to drive from Buffalo, NY, to New York City, a distance of about 400 miles. Find his average speed.
_______ miles per hour

Answer: 50

Explanation:
Given,
It took Eduardo 8 hours to drive from Buffalo, NY, to New York City, a distance of about 400 miles.
We can use the formula d = rt
r = d/t
r = 400 miles/8 hours
r = 50 miles per hour

Question 6.
Write an expression that represents the value, in cents, of n nickels.
Type below:
_____________

Answer: 0.05n

Explanation:
An expression does not have an equal sign.
Since the value of a nickel is 5 cents and you want to find out the value of n nickels (which means if you had any number of nickels) the expression would be
.05n

Share and Show – Page No. 383

Question 1.
Evaluate 5k + 6 for k = 4.
_______

Answer: 26

Explanation:
The expression is 5k + 6
Substitute the value k = 4
5(4) + 6 = 20 + 6 = 26
5k + 6 = 26

Evaluate the expression for the given value of the variable.

Question 2.
m − 9 for m = 13
_______

Answer: 4

Explanation:
m – 9
Substitute the value of m in the expression
13 – 9 = 4
Thus m – 9 = 4

Question 3.
16 − 3b for b = 4
_______

Answer: 4

Explanation:
Given the expression 16 – 3b
Now substitute the value of b in the expression.
16 – 3b = 16 – 3(4) = 16 – 12 = 4
16 – 3b = 4

Question 4.
p2 + 4 for p = 6
_______

Answer: 40

Explanation:
Given the expression p2 + 4
Substitute the value of p in the expression
62 + 4 = 36 + 4 = 40
Thus the value of p2 + 4 is 40.

Question 5.
The formula A = lw gives the area A of a rectangle with length l and width w. What is the area in square feet of a United States flag with a length of 12 feet and a width of 8 feet?
_______ square feet

Answer: 96 square feet

Explanation:
Use the formula A = lw
Length = 12 feet
Width = 8 feet
A = lw
A = 12 feet × 8 feet = 96 square feet
Thus the area of the United States flag is 96 square feet.

On Your Own

Practice: Copy and Solve Evaluate the expression for the given value of the variable.

Question 6.
7s + 5 for s = 3
_______

Answer: 26

Explanation:
Given the expression 7s + 5
Substitute  the value of S in the above expression
7(3) + 5 = 21 + 5 = 26

Question 7.
21 − 4d for d = 5
_______

Answer: 1

Explanation:
Given the expression 21 – 4d
Substitute  the value d = 5 in the above expression
21 – 4(5) = 21 – 20 = 1

Question 8.
(t − 6)2 for t = 11
_______

Answer: 25

Explanation:
Given the expression (t − 6)2
Substitute the value t = 11
Thus (t − 6)2 = (11 − 6)2 = 5 × 5 = 25

9.6 × (2v − 3) for v = 5
_______

Answer: 42

Explanation:
Given the expression 6 × (2v – 3)
Substitute the value of v in the above expression.
6 × (2v – 3) = 6 × (2 × 5 – 3)
6 × (10 – 3)
6 × 7 = 42
Thus the value of 6 × (2v – 3) = 42

Question 10.
2 × (k2 − 2) for k = 6
_______

Answer: 68

Explanation:
Given the expression 2 × (k2 − 2)
Substitute the value of k in the above expression
2 × (k2 − 2) = 2 × (62 − 2)
2 × (36 – 2) = 2 × 34 = 68
Thus the value of 2 × (k2 − 2) is 68

Question 11.
5 × (f − 32) ÷ 9 for f = 95
_______

Answer: 35

Explanation:
The expression is 5 × (f – 32) ÷ 9
Substitute  the value f = 95
5 × (f – 32) ÷ 9 = 5 × (95 – 32) ÷ 9
5 × (63 ÷ 9) = 5 × 7 = 35
The value of 5 × (63 ÷ 9) = 35

Question 12.
The formula P = 4s gives the perimeter P of a square with side length s. How much greater is the perimeter of a square with a side length of 5 \(\frac{1}{2}\) inches than a square with a side length of 5 inches?
_______ inches

Answer: 2 inches

Explanation:
We have to use the formula P = 4s to find the perimeter of the square.
4 × 5 \(\frac{1}{2}\)
Convert the mixed fraction to the improper fraction.
4 × 11/2 = 2 × 11 = 22 inches
4 × 5 inches = 20 inches
To find which has the greater  perimeter  we have to subtract 20 inches from 22 inches
22 inches – 20 inches = 2 inches
Thus the perimeter of a square with 5 \(\frac{1}{2}\) inches is 2 inches greater than a square with a side length of 5 inches.

Problem Solving + Applications – Page No. 384

The table shows how much a company charges for skateboard wheels. Each pack of 8 wheels costs $50. Shipping costs $7 for any order. Use the table for 13−15.
Go Math Grade 6 Answer Key Chapter 7 Exponents img 7

Question 13.
Complete the table.
Type below:
_____________

Answer:
Go-Math-Grade-6-Answer-Key-Chapter-7-Exponents-img-7

Question 14.
A skateboard club has $200 to spend on new wheels this year. What is the greatest number of packs of wheels the club can order?
_______ packs

Answer: 3 packs

Explanation:
A skateboard club has $200 to spend on new wheels this year.
From the above table, we can say that the club can order 3 packs of wheels.

Question 15.
Make Sense of Problems A sporting goods store placed an order for 12 packs of wheels on the first day of each month last year. How much did the sporting goods store spend on these orders last year?
$ _______

Answer: 7284

Explanation:
Make Sense of Problems A sporting goods store placed an order for 12 packs of wheels on the first day of each month last year.
Substitute n = 7 in the expression 50 × n + 7
We get, 50 × 12 + 7
600 + 7 = 607
Now multiply 607 with 12
607 × 12 = 7284
Therefore the sporting goods store spent $7284 on these orders last year.

Question 16.
What’s the Error? Bob used these steps to evaluate 3m − 3 ÷ 3 for m = 8. Explain his error.
3 × 8 − 3 ÷ 3 = 24 − 3 ÷ 3
= 21 ÷ 3
= 7
Type below:
_____________

Answer:
First, he has to subtract 8 and 3. But he first multiplied and then subtracted 24 and 3.
3 × 8 − 3 ÷ 3 = 3 × (8 − 3) ÷ 3
3 × 5 ÷ 3
15 ÷ 3 = 5

Question 17.
The surface area of a cube can be found by using the formula 6s2, where s represents the length of the side of the cube.
The surface area of a cube that has a side length of 3 meters is _____ meters squared.
The surface area of a cube that has a side length                       meters
of 3 meters is _____________ squared

Answer: 5

Explanation:
The surface area of a cube can be found by using the formula 6s2
he surface area of a cube that has a side length of 3 meters
s2 = 32 = 9
6 × 9 = 54 square meters

Evaluate Algebraic Expressions and Formulas – Page No. 385

Evaluate the expression for the given values of the variables.

Question 1.
w + 6 for w = 11
_______

Answer: 17

Explanation:
Given the expression w + 6
Substitute the value w = 6 in the expression
w + 6 = 11 + 6 = 17

Question 2.
17 − 2c for c = 7
_______

Answer: 3

Explanation:
Substitute  the value c = 7 in the given expression
17 – 2(7) = 17 – 14 = 3
Thus the value for 17 – 2c is 3.

Question 3.
b2 − 4 for b = 5
_______

Answer: 21

Explanation:
Substitute the value b = 5 in the expression
b2 − 4 = 52 − 4 = 25 – 4 = 21
Thus the value for the expression b2 − 4 is 21.

Question 4.
(h − 3)2 for h = 5
_______

Answer: 4

Explanation:
We have to substitute the value h = 5
(h − 3)2 = (5 − 3)2
= (2)2 = 4
Therefore the value of (h − 3)2 is 4.

Question 5.
m + 2m + 3 for m = 12
_______

Answer: 39

Explanation:
Given the expression m + 2m + 3
Now substitute the value m = 12 in the above expression.
12 + 2(12) + 3 = 12 + 24 + 3 = 39.
The value for m + 2m + 3 = 39.

Question 6.
4 × (21 − 3h) for h = 5
_______

Answer: 24

Explanation:
Substitute h = 5 in the given expression.
4 × (21 – 3h) = 4 × (21 – 3(5))
4 × (21 – 15) = 4 × 6 = 24
Therefore the value for 4 × (21 – 3h) is 24.

Question 7.
7m − 9n for m = 7 and n = 5
_______

Answer: 4

Explanation:
Substitute the values m = 7 and n = 5 in the above expression.
7m – 9n = 7 × 7 – 9 × 5
= 49 – 45 = 4
Thus 7m – 9n = 4.

Question 8.
d2 − 9k + 3 for d = 10 and k = 9
_______

Answer: 22

Explanation:
Given the expression d2 − 9k + 3
Now substitute d = 10 and k = 9 in the expression.
d2 − 9k + 3 = 102 − 9(9) + 3
100 – 81 + 3 = 22
Thus the value for the expression d2 − 9k + 3 is 22.

Question 9.
3x + 4y ÷ 2 for x = 7 and y = 10
_______

Answer: 41

Explanation:
Substitute the values x = 7 and y = 10 in the expression.
3x + 4y ÷ 2 = 3(7) + 4(10) ÷ 2
21 + 40 ÷ 2 = 21 + 20 = 41
Thus the value for 3x + 4y ÷ 2 is 41.

Problem Solving

Question 10.
The formula P = 2l + 2w gives the perimeter P of a rectangular room with length l and width w. A rectangular living room is 26 feet long and 21 feet wide. What is the perimeter of the room?
_______ feet

Answer: 94 feet

Explanation:
Use the formula  of the perimeter  of a rectangle P = 2l + 2w
L = 26 feet
W = 21 feet
P = 2(26) + 2(21)
P = 52 feet + 42 feet
P = 94 feet
Therefore the perimeter of a room is 94 feet.

Question 11.
The formula C = 5(F − 32) ÷ 9 gives the Celsius temperature in C degrees for a Fahrenheit temperature of F degrees. What is the Celsius temperature for a Fahrenheit temperature of 122 degrees?
_______ degrees Celsius

Answer: 50

Explanation:
C = 5(F – 32) ÷ 9
We know that F = 122 degrees
Substitute the value of F in the formula
C = 5(122 – 32) ÷ 9
C = 5(90) ÷ 9
C = 450 ÷ 9 = 50
Thus the answer is 50 degrees Celsius.

Question 12.
Explain how the terms variable, algebraic expression, and evaluate are related.
Type below:
_____________

Answer: To evaluate an algebraic expression, you have to substitute a number for each variable and perform the arthematic operations. If we know the variables, we can replace the variables with their values and then evaluate the expression.

Lesson Check – Page No. 386

Question 1.
When Debbie baby-sits, she charges $5 to go to the house plus $8 for every hour she is there. The expression 5 + 8h gives the amount in dollars she charges. How much will she charge to baby-sit for 5 hours?
$ _______

Answer: 45

Explanation:
When Debbie baby-sits, she charges $5 to go to the house plus $8 for every hour she is there. The expression 5 + 8h gives the amount in dollars she charges.
If h = 5 hours
Substitute the value h in the above expression.
5 + 8h = 5 + 8(5) = 5 + 40 = 45
Thus she charges $45 to baby-sit for 5 hours.

Question 2.
The formula to find the cost C in dollars of a square sheet of glass is C = 25s2 where s represents the length of a side in feet. How much will Ricardo pay for a square sheet of glass that is 3 feet on each side?
$ _______

Answer: $225

Explanation:
Use the formula C = 25s2
s represents the length of a side in feet.
s = 3 feet
Substitute the value s in the above formula.
C = 25s2
C = 25(32)
C = 25(9) = 225
Ricardo pays $225 for a square sheet of glass that is 3 feet on each side.

Spiral Review

Question 3.
Evaluate using the order of operations.
\(\frac{3}{4}+\frac{5}{6} \div \frac{2}{3}\)
_______

Answer: 2

Explanation:
\(\frac{3}{4}\) + [/latex]\frac{5}{6}[/latex] ÷ [/latex]\frac{2}{3}[/latex]
[/latex]\frac{5}{6}[/latex] ÷ [/latex]\frac{2}{3}[/latex]
= [/latex]\frac{5}{6}[/latex] × [/latex]\frac{3}{2}[/latex] = [/latex]\frac{15}{12}[/latex] = [/latex]\frac{5}{4}[/latex]
Now convert the improper fraction to the mixed fraction.
[/latex]\frac{5}{4}[/latex] = 1 [/latex]\frac{1}{4}[/latex]
1 [/latex]\frac{1}{4}[/latex] + \(\frac{3}{4}\)
1 + [/latex]\frac{1}{4}[/latex] + \(\frac{3}{4}\) = 1 + 1 = 2
\(\frac{3}{4}+\frac{5}{6} \div \frac{2}{3}\) = 2

Question 4.
Patricia scored 80% on a math test. She missed 4 problems. How many problems were on the test?
_______ problems

Answer: 20

Explanation:
Patricia scored 80% on a math test. She missed 4 problems.
4 ÷ 80%
4 × [/latex]\frac{100}{80}[/latex] = 4 × 5 = 20
Therefore there are 20 questions in the test.

Question 5.
What is the value of 73?
_______

Answer: 343

Explanation:
73 = 7 × 7 × 7 = 49 × 7 = 343
Thus the value of 73 is 343.

Question 6.
James and his friends ordered b hamburgers that cost $4 each and f fruit cups that cost $3 each. Write an algebraic expression for the total cost in dollars of their purchases.
Type below:
_____________

Answer: 4b + 3f

Explanation:
Given that, James and his friends ordered b hamburgers that cost $4 each and f fruit cups that cost $3 each.
Multiply b with $4 and multiply $3 with f
Add 4b and 3f
Thus the expression is 4b + 3f.

Vocabulary – Page No. 387

Choose the best term from the box to complete the sentence.
Go Math Grade 6 Answer Key Chapter 7 Exponents img 8

Question 1.
A(n) _____ tells how many times a base is used as a factor.
Type below:
_____________

Answer: Exponent
An Exponent tells how many times a base is used as a factor.

Question 2.
The mathematical phrase 5+2×18 is an example of a(n) _____.
Type below:
_____________

Answer: Numerical expression
The mathematical phrase 5+2×18 is an example of Numerical expression.

Concepts and Skills

Find the value.

Question 3.
54
________

Answer: 5 × 5 × 5 × 5 = 625

Explanation:
The number 5 is the repeated factor.
5 is used 4 times.
Multiply 5 four times.
5 × 5 × 5 × 5 = 625

Question 4.
212
________

Answer: 21 × 21 = 441

Explanation:
The number 21 is the repeated factor.
21 is used 2 times.
Multiply 21 two times.
21 × 21 = 441

Question 5.
83
________

Answer: 8 × 8 × 8 = 512

Explanation:
The number 8 is the repeated factor.
8 is used 3 times.
8 × 8 × 8 = 512

Evaluate the expression.

Question 6.
92 × 2 − 42
________

Answer: 146

Explanation:
92 × 2 − 42
92 = 9 × 9 = 81
42 = 4 × 4 = 16
81 × 2 – 16 = 162 – 16 = 146
Thus 92 × 2 − 42 = 146

Question 7.
2 × (10 − 2) ÷ 22
________

Answer: 4

Explanation:
2 × (10 − 2) ÷ 22
2 × (10 − 2) ÷ 4
2 × 8 ÷ 4 = 16 ÷ 4 = 4
Thus 2 × (10 − 2) ÷ 22 = 4

Question 8.
30 − (33 − 8)
________

Answer: 11

Explanation:
33 = 3 × 3 × 3 = 27
30 − (33 − 8) = 30 – (27 – 8) = 30 – 19 = 11
30 − (33 − 8) = 11
So, 30 − (33 − 8) is 11.

Write an algebraic expression for the word expression.

Question 9.
the quotient of c and 8
Type below:
_____________

Answer: c ÷ 8
The quotient is nothing but the division of c by 8. So, the expression is c ÷ 8.

Question 10.
16 more than the product of 5 and p
Type below:
_____________

Answer: 16 + 5p

Explanation:
The operation for more than is addition. Here we have to add 16 to the product of 5 and p.
The product is the operation for multiplication. Multiply 5 and p and then add 16 to it.
The expression of the word is 16 + 5p.

Question 11.
9 less than the sum of x and 5
Type below:
_____________

Answer: 9 – x + 5

Explanation:
First, we have to evaluate the expression x and 5.
Sum of is nothing but adding x and 5.
Difference between 9 and x and 5
The expression is 9 – x + 5.

Evaluate the expression for the given value of the variable.

Question 12.
5 × (h + 3) for h = 7
________

Answer: 50

Explanation:
Given expression is 5 × (h + 3)
Substitute h = 7 in the above expression.
5 × (h + 3) = 5 × (7 + 3)
5 × 10 = 50
5 × (h + 3) = 50

Question 13.
2 × (c2 − 5) for c = 4
________

Answer: 22

Explanation:
Given 2 × (c2 − 5)
Substitute c = 4 in the expression
2 × (c2 − 5) = 2 × (42 − 5)
= 2 × (16 – 5) = 2 × 11 = 22
2 × (c2 − 5) = 22

Question 14.
7a − 4a for a = 8
________

Answer: 24

Explanation:
Given, 7a − 4a
Subtract the like terms
7a − 4a = 3a
Now substitute the value a = 8 in the above expression
3a = 3 × 8 = 24
7a − 4a = 24

Page No. 388

Question 15.
The greatest value of any U.S. paper money ever printed is 105 dollars. What is this amount written in standard form?
________

Answer: 100000

Explanation:
105 dollars = 10 × 10 × 10 × 10 × 10
10 is a repeated factor.
10 repeated 5 times.
10 × 10 × 10 × 10 × 10 = 100000 dollars

Question 16.
A clothing store is raising the price of all its sweaters by $3.00. Write an expression that could be used to find the new price of a sweater that originally cost d dollars.
Type below:
_____________

Answer: d + 3

Explanation:
A clothing store is raising the price of all its sweaters by $3.00.
The cost of the sweater is d dollars. The store is going to add $3.
So, the new price of a sweater is the sum of d dollars and $3.
The expression is d + 3.

Question 17.
Kendra bought a magazine for $3 and 4 paperback books for $5 each. The expression 3 + 4 × 5 represents the total cost in dollars of her purchases. What are the terms in this expression?
Type below:
_____________

Answer: 3 and 4 × 5

Explanation:
Kendra bought a magazine for $3 and 4 paperback books for $5 each. The expression 3 + 4 × 5 represents the total cost in dollars of her purchases.
The terms in the expression are 3, 4, and 5.

Question 18.
The expression 5c + 7m gives the number of people who can ride in c cars and m minivans. What are the coefficients in this expression?
Type below:
_____________

Answer: The coefficients in the expression 5c + 7m are 5 and 7.

Question 19.
The formula P = a + b + c gives the perimeter P of a triangle with side lengths a, b, and c. How much greater is the perimeter of a triangular field with sides that measure 33 yards, 56 yards, and 65 yards than the perimeter of a triangular field with sides that measure 26 yards, 49 yards, and 38 yards?
________ yards

Answer: 41 yards

Explanation:
First, we have to calculate the perimeter of the 1st triangle.
Given:
a = 33 yards
b = 56 yards
c = 65 yards
P1 = a + b + c
P1 = 33 + 56 + 65 = 154 yards
Now we have to calculate the perimeter of 2nd triangle.
Given:
a = 26 yards
b = 49 yards
c = 38 yards
P2 = a + b + c
P2 = 26 + 49 + 38 = 113 yards
Now we have to calculate which triangle has greater perimeter and how much greater.
P1 – P2 = 154 yards – 113 yards = 41 yards
Therefore, 41 yards greater is the perimeter of the 1st triangular field than the perimeter of the 2nd triangular field.

Share and Show – Page No. 391

Louisa read that the highest elevation of Mount Everest is 8,848 meters. She wants to know how much higher Mount Everest is than Mount Rainier. Use this information for 1–2.

Question 1.
Write an expression to represent the difference in the heights of the two mountains. Tell what the variable in your expression represents.
Type below:
_____________

Answer: 8848 – h, where h represents the height of the Mount Rainier

Explanation:
Given that, the height of the Mount Everest is 8848 meters
Let the height of the Mount Rainier is h
The difference in height of Mount Everest and height of the Mount Rainier is 8848 – h.

Question 2.
Louisa researches the highest elevation of Mount Rainier and finds that it is 4,392 meters. Use your expression to find the difference in the mountains’ heights.
________ meters

Answer: 4456 meters

Explanation:
The height of the Mount Rainier = 4392 meters
Replace the value of height of the Mount Rainier in the above expression.
8848 – h = 8848 meters – 4392 meters = 4456 meters
Thus the difference between the height of the two mountains is 4456 meters.

On Your Own

A muffin recipe calls for 3 times as much flour as sugar. Use this information for 3–5.

Question 3.
Write an expression that can be used to find the amount of flour needed for a given amount of sugar. Tell what the variable in your expression represents.
Type below:
_____________

Answer:
Let the amount of sugar used represents the variable is s.
The expression to find the amount of flour needed for a given amount of sugar is 3 × m i.e., 3m

Question 4.
Use your expression to find the amount of flour needed when \(\frac{3}{4}\) cup of sugar is used.
______ \(\frac{□}{□}\)

Answer: 2 \(\frac{1}{4}\)

Explanation:
Given that, A muffin recipe calls for 3 times as much flour as sugar.
The amount of flour needed when \(\frac{3}{4}\) cup of sugar used is 3 × \(\frac{3}{4}\) = \(\frac{9}{4}\)
Convert the improper fraction into the mixed fraction.
\(\frac{9}{4}\) = 2 \(\frac{1}{4}\)
Therefore 2 \(\frac{1}{4}\) amount of flour needed when \(\frac{3}{4}\) cup of sugar is used.

Question 5.
Reason Quantitatively Is the value of the variable in your expression restricted to a particular set of numbers? Explain.
Type below:
_____________

Answer: The values that make the denominator equal to zero for a rational expression are known as restricted values. The solutions are the restricted values since they result in a denominator of zero when replaced for the variable(s).

Practice: Copy and Solve Write an algebraic expression for each word expression. Then evaluate the expression for these values of the variable: \(\frac{1}{2}\), 4, and 6.5.

Question 6.
the quotient of p and 4
Type below:
_____________

Answer: p ÷ 4

Explanation:
The expression is p ÷ 4
p = \(\frac{1}{2}\)
\(\frac{1}{2}\) ÷ 4
\(\frac{1}{2}\)/4 = \(\frac{1}{8}\)
p ÷ 4 when p = \(\frac{1}{2}\) is \(\frac{1}{8}\)
p = 4
4 ÷ 4 = 1
p = 6.5
6.5 ÷ 4 = 1.625

Question 7.
4 less than the sum of x and 5
Type below:
_____________

Answer: 4 – (x + 5)

Explanation:
The expression is 4 – (x + 5)
x = 1/2
4 – (x + 5) = 4 – 1/2 + 5
3 1/2 + 5 = 8 1/2
x = 4
4 – x + 5
4 – 4 + 5 = 5
x = 6.5
4 – 6.5 + 5 = 2.5

Problem Solving + Applications – Page No. 392

Use the graph for 8–10.
Go Math Grade 6 Answer Key Chapter 7 Exponents img 9

Question 8.
Write expressions for the distance in feet that each animal could run at top speed in a given amount of time. Tell what the variable in your expressions represents.
Type below:
_____________

Answer:
The expression for distance in feet for Elephant = 22t
The expression for distance in feet for Cheetah = 103t
The expression for distance in feet for Giraffe = 51t
The expression for distance in feet for hippopotamus = 21t
Where t represents the time.

Question 9.
How much farther could a cheetah run in 20 seconds at top speed than a hippopotamus could?
______ feet

Answer: 1640 feet

Explanation:
The expression for distance in feet for Cheetah = 103t
where t = 20 sec
103t = 103 × 20 sec = 2060 feet
The expression for distance in feet for hippopotamus = 21t
where t = 20 sec
21t = 21 × 20 = 420 feet
Now we have to find How much farther could a cheetah run in 20 seconds at top speed than a hippopotamus could
2060 feet – 420 feet = 1640 feet

Question 10.
A giraffe runs at top speed toward a tree that is 400 feet away. Write an expression that represents the giraffe’s distance in feet from the tree after s seconds.
Type below:
_____________

Answer:
The expression representing the giraffe’s distance from tree after s seconds, if the rate is 51 ft per second.
7 43/60 seconds in all

Question 11.
A carnival charges $7 for admission and $2 for each ride. An expression for the total cost of going to the carnival and riding n rides is 7 + 2n.
Complete the table by finding the total cost of going to the carnival and riding n rides.
Go Math Grade 6 Answer Key Chapter 7 Exponents img 10
Type below:
_____________

Answer:
Go-Math-Grade-6-Answer-Key-Chapter-7-Exponents-img-10

Use Algebraic Expressions – Page No. 393

Jeff sold the pumpkins he grew for $7 each at the farmer’s market.

Question 1.
Write an expression to represent the amount of money in dollars Jeff made selling the pumpkins. Tell what the variable in your expression represents
Type below:
_____________

Answer: 7p, where p is the number of pumpkins

Question 2.
If Jeff sold 30 pumpkins, how much money did he make?
$ ________

Answer: 210

Explanation:
The expression is 7p
p = 30 pumpkins
7 × 30 = 210
Thus Jeff sold 30 pumpkins for $210.

An architect is designing a building. Each floor will be 12 feet tall.

Question 3.
Write an expression for the number of floors the building can have for a given building height. Tell what the variable in your expression represents.
Type below:
_____________

Answer: The expression for the number of floors is h/12, where h is the height of the building.

Question 4.
If the architect is designing a building that is 132 feet tall, how many floors can be built?
________ floors

Answer: 11 floors

Explanation:
Given the height of the building is 132 feet
Substitute h in the above expression
h/12 = 132/12 = 11 floors
Thus 11 floors can be built.

Write an algebraic expression for each word expression. Then evaluate the expression for these values of the variable: 1, 6, 13.5.

Question 5.
the quotient of 300 and the sum of b and 24
Type below:
_____________

Answer: 300 ÷ (b + 24)

Explanation:
For b = 1
300 ÷ (b + 24) = 300 ÷ (1 + 24)
300 ÷ 25 = 12
Thus 300 ÷ (b + 24) when b = 1 is 12.
For b = 6
300 ÷ (b + 24) = 300 ÷ (6 + 24)
300 ÷ 30 = 10
Thus 300 ÷ (b + 24) when b = 6 is 10.
For b = 13.5
300 ÷ (b + 24) = 300 ÷ (13.5 + 24)
300 ÷ 37.5 = 8
300 ÷ (b + 24) when b = 13.5 is 8.

Question 6.
13 more than the product of m and 5
Type below:
_____________

Answer: 13 + 5m

Explanation:
For m = 1
13 + 5m = 13 + 5(1) = 13 + 6 = 19
For m = 6
13 + 5m = 13 + 5(6) = 13 + 30 = 43
For m = 13.5
13 + 5m = 13 + 5(13.5) = 13 + 67.5 = 80.5

Problem Solving

Question 7.
In the town of Pleasant Hill, there is an average of 16 sunny days each month. Write an expression to represent the approximate number of sunny days for any number of months. Tell what the variable represents.
Type below:
_____________

Answer: 16m, m for months

Explanation:
In the town of Pleasant Hill, there is an average of 16 sunny days each month. Write an expression to represent the approximate number of sunny days for any number of months.
we have to multiply the number of months with 16
The expression will be 16 times of m = 16m

Question 8.
How many sunny days can a resident of Pleasant Hill expect to have in 9 months?
________ days

Answer: 144 days

Explanation:
The expression to represent the approximate number of sunny days for any number of months is 16m
m = 9
Substitute the value of m in the expression.
16m = 16 × 9 = 144 days

Question 9.
Describe a situation in which a variable could be used to represent any whole number greater than 0.
Type below:
_____________

Answer: To represent the number of people any answer can be accepted.

Lesson Check – Page No. 394

Question 1.
Oliver drives 45 miles per hour. Write an expression that represents the distance in miles he will travel for h hours driven.
Type below:
_____________

Answer: 45h

Explanation:
It is given that Oliver drives 45 miles per hour. Let the number of hours he drove be h. The distance is the product of speed and time. The distance travel by Oliver is defined by the expression is 45h.

Question 2.
Socks cost $5 per pair. The expression 5p represents the cost in dollars of p pairs of socks. Why must p be a whole number?
Type below:
_____________

Answer: p must be a whole number because in almost 100% of all stores it is not allowed to buy a single sock, you must always buy a pair of socks.

Spiral Review

Question 3.
Sterling silver consists of 92.5% silver and 7.5% copper. What decimal represents the portion of the silver in sterling silver?
________

Answer: 0.925

Explanation:
If Sterling silver is 92.5% silver, that means it has 92.5/100 * 100% silver
The fraction 92.5/100 can be simplified by just moving the decimal 2 places to the left:
92.5/100 = .925

Question 4.
How many pints are equivalent to 3 gallons?
________ pints

Answer: 24

Explanation:
Convert from gallons to pints.
1 gallon = 8 pints
3 gallons = 3 × 8 pints = 24 pints
24 pints are equivalent to 3 gallons.

Question 5.
Which operation should be done first to evaluate 10 + (66 – 62)?
Type below:
_____________

Answer: Square 6

Question 6.
Evaluate the algebraic expression h(m + n) ÷ 2 for h = 4, m = 5, and n = 6.
________

Explanation:
Given the expression h(m + n) ÷ 2
h = 4
m = 5
n = 6
h(m + n) ÷ 2 = 4 (5 + 6) ÷ 2
4 (11) ÷ 2 = 44 ÷ 2 = 22
h(m + n) ÷ 2 = 22

Share and Show – Page No. 397

Question 1.
Museum admission costs $7, and tickets to the mammoth exhibit cost $5. The expression 7p + 5p represents the cost in dollars for p people to visit the museum and attend the exhibit. Simplify the expression by combining like terms.
Type below:
_____________

Answer: 12p

Explanation:

7p+5p
When you combine like terms, you just add all the terms that have the same variable
so you get 7p + 5p = 12p

Question 2.
What if the cost of tickets to the exhibit was reduced to $3? Write an expression for the new cost in dollars for p people to visit the museum and attend the exhibit. Then, simplify the expression by combining like terms.
Type below:
_____________

Answer: 10p

Explanation:
Museum admission costs $7, and tickets to the mammoth exhibit cost $5.
The expression 7p + 5p represents the cost in dollars for p people to visit the museum and attend the exhibit.
The cost of tickets to the mammoth exhibit is $5.
If it is reduced to $3 then the cost will be $5 – $2 = $3
12p – 2p = 10p

Question 3.
A store receives tomatoes in boxes of 40 tomatoes each. About 4 tomatoes per box cannot be sold due to damage. The expression 40b − 4b gives the number of tomatoes that the store can sell from a shipment of b boxes. Simplify the expression by combining like terms.
Type below:
_____________

Answer: 36b

Explanation:
Given, A store receives tomatoes in boxes of 40 tomatoes each.
About 4 tomatoes per box cannot be sold due to damage.
The expression 40b − 4b gives the number of tomatoes that the store can sell from a shipment of b boxes.
Subtract 40b and 4b
40b – 4b = 36b

Question 4.
Each cheerleading uniform includes a shirt and a skirt. The shirts cost $12 each, and skirts cost $18 each. The expression 12u + 18u represents the cost in dollars of buying u uniforms. Simplify the expression by combining like terms.
Type below:
_____________

Answer: 30u

Explanation:
The expression 12u + 18u represents the cost in dollars of buying u uniforms.
12u and 18u are the like terms. So add the two terms
12u + 18u = 30u

Question 5.
A shop sells vases holding 9 red roses and 6 white roses. The expression 9v + 6v represents the total number of roses needed for v vases. Simplify the expression by combining like terms.
Type below:
_____________

Answer: 15v

Explanation:
A shop sells vases holding 9 red roses and 6 white roses.
The expression 9v + 6v represents the total number of roses needed for v vases.
The like terms are 9v and 6v
9v + 6v = 15v

On Your Own – Page No. 398

Question 6.
Marco received a gift card. He used it to buy 2 bike lights for $10.50 each. Then he bought a handlebar bag for $18.25. After these purchases, he had $0.75 left on the card. How much money was on the gift card when Marco received it?
$ _______

Answer:
Marco received a gift card. He used it to buy 2 bike lights for $10.50 each.
Then he bought a handlebar bag for $18.25.
After these purchases, he had $0.75 left on the card.
Add total amount = 2 × $10.50 + $18.25 + $0.75
$21 + $19 = $40
$40 was on the gift card when Marco received it.

Question 7.
Lydia collects shells. She has 24 sea snail shells, 16 conch shells, and 32 scallop shells. She wants to display the shells in equal rows, with only one type of shell in each row. What is the greatest number of shells Lydia can put in each row?
_______ shells

Answer: 8 shells

Explanation:
Lydia collects shells. She has 24 sea snail shells, 16 conch shells, and 32 scallop shells.
She wants to display the shells in equal rows, with only one type of shell in each row.
The possible shells in equal rows are 8 because 16, 24, and 32 are the multiples of 8.
Thus the greatest number of shells Lydia can put in each row is 8.

Question 8.
The three sides of a triangle measure 3x + 6 inches, 5x inches, and 6x inches. Write an expression for the perimeter of the triangle in inches. Then simplify the expression by combining like terms.
Type below:
_____________

Answer:
Perimeter of the triangle = a + b + c
Let a = 3x + 6 inches
b = 5x inches
c = 6x inches
P = a + b + c
P = 3x + 6 + 5x + 6x
Combine the like terms 3x, 5x, 6x
P = 14x + 6
Thus the perimeter of the triangle is 14x + 6.

Question 9.
Verify the Reasoning of Others Karina states that you can simplify the expression 20x + 4 by combining like terms to get 24x. Does Karina’s statement make sense? Explain.
Type below:
_____________

Answer: Karina’s statement doesn’t make sense. Because the 20x + 4 are not the like terms.
We can add only the like terms. 20x + 4 ≠ 24x

Question 10.
Vincent is ordering accessories for his surfboard. A set of fins costs $24 each and a leash costs $15. The shipping cost is $4 per order. The expression 24b + 15b + 4 can be used to find the cost in dollars of buying b fins and b leashes plus the cost of shipping.
For numbers 10a–10c, select True or False for each statement.
10a. The terms are 24b, 15b and 4.
10b. The like terms are 24b and 15b.
10c. The simplified expression is 43b.
10a. _____________
10b. _____________
10c. _____________

Answer:
10a. True
10b. True
10c. False

Explanation:
a. The terms of the expression 24b + 15b + 4 area 24b, 15b, 4.
b. The terms are said to be like if they have the common variable. So, the common terms are 24b, 15b.
c. Combine the like terms 24b and 15b
24b + 15b = 39b
Thus the statement is false.

Problem Solving Combine Like Terms – Page No. 399

Read each problem and solve.

Question 1.
A box of pens costs $3 and a box of markers costs $5. The expression 3p + 5p represents the cost in dollars to make p packages that include 1 box of pens and 1 box of markers. Simplify the expression by combining like terms.
Type below:
_____________

Answer: 3p + 5p = 8p

Explanation:
A box of pens costs $3 and a box of markers costs $5.
The expression 3p + 5p represents the cost in dollars to make p packages that include 1 box of pens and 1 box of markers.
Adding the like terms 3p + 5p is 8p.

Question 2.
Riley’s parents got a cell phone plan that has a $40 monthly fee for the first phone. For each extra phone, there is a $15 phone service charge and a $10 text service charge. The expression 40 + 15e + 10e represents the total phone bill in dollars, where e is the number of extra phones. Simplify the expression by combining like terms.
Type below:
_____________

Answer: 25e + 40

Explanation:
Given that,
Riley’s parents got a cell phone plan that has a $40 monthly fee for the first phone.
For each extra phone, there is a $15 phone service charge and a $10 text service charge.
The expression 40 + 15e + 10e represents the total phone bill in dollars,
We have to combine the like terms here
The like terms in the expression are 15e and 10e.
That means 40 + 15e + 10e = 25e + 40

Question 3.
A radio show lasts for h hours. For every 60 minutes of air time during the show, there are 8 minutes of commercials. The expression 60h – 8h represents the air time in minutes available for talk and music. Simplify the expression by combining like terms.
Type below:
_____________

Answer: 52h

Explanation:
A radio show lasts for h hours. For every 60 minutes of air time during the show, there are 8 minutes of commercials.
The expression 60h – 8h represents the air time in minutes available for talk and music.
Now we have to Subtract the like terms 60h – 8h = 52h

Question 4.
A sub shop sells a meal that includes an Italian sub for $6 and chips for $2. If a customer purchases more than 3 meals, he or she receives a $5 discount. The expression 6m + 2m – 5 shows the cost in dollars of the customer’s order for m meals, where m is greater than 3. Simplify the expression by combining like terms.
Type below:
_____________

Answer: 8m – 5

Explanation:
The expression is 6m + 2m – 5
Now combine the like terms 6m + 2m – 5 = 8m – 5

Question 5.
Explain how combining like terms is similar to adding and subtracting whole numbers. How are they different?
Type below:
_____________

Answer: It’s the same because you are adding or subtracting numbers but it’s different because they can only be added or subtracted if the variable attached is the same. There are no variables when adding/subtracting regular whole numbers.

Lesson Check – Page No. 400

Question 1.
For each gym class, a school has 10 soccer balls and 6 volleyballs. All of the classes share 15 basketballs. The expression 10c + 6c + 15 represents the total number of balls the school has for c classes. What is a simpler form of the expression?
Type below:
_____________

Answer: 16c + 15

Explanation:
For each gym class, a school has 10 soccer balls and 6 volleyballs.
All of the classes share 15 basketballs.
c represents classes.
The expression is 10c + 6c + 15
Combine the like terms 10c and 6c
Now add common terms 10c + 6c + 15 = 16c + 15

Question 2.
A public library wants to place 4 magazines and 9 books on each display shelf. The expression 4s + 9s represents the total number of items that will be displayed on s shelves. Simplify this expression.
Type below:
_____________

Answer: 13s

Explanation:
A public library wants to place 4 magazines and 9 books on each display shelf.
The expression is 4s + 9s
Combine the like terms 4s + 9s = 13s

Spiral Review

Question 3.
A bag has 8 bagels. Three of the bagels are cranberry. What percent of the bagels are cranberry?
________ %

Answer: 37.5%

Explanation:
[/latex]\frac{3}{8}[/latex] = 0.375
0.375 × 100 = 37.5 %
37.5% of the bagels are cranberry.

Question 4.
How many kilograms are equivalent to 3,200 grams?
________ kilograms

Answer: 3.2 kg

Explanation:
Convert from grams into kilograms
1000 grams = 1kg
3200 grams = 3200 × 1/1000 kg = 3.2 kg
3.2 kilograms are equivalent to 3,200 grams.

Question 5.
Toni earns $200 per week plus $5 for every magazine subscription that she sells. Write an expression that represents how much she will earn in dollars in a week in which she sells s subscriptions.
Type below:
_____________

Answer: 200 + 5s

Explanation:
Toni earns $200 per week plus $5 for every magazine subscription that she sells.
s represents subscriptions.
200 + 5 × s
Thus the expression that represents how much she will earn in dollars in a week is 200 + 5s

Question 6.
At a snack stand, drinks cost $1.50. Write an expression that could be used to find the total cost in dollars of d drinks.
Type below:
_____________

Answer: 1.5d

Explanation:
At a snack stand, drinks cost $1.50.
To find the total cost in dollars of d drinks we have to multiply 1.50 with d.
1.50 × d
Thus the expression that could be used to find the total cost in dollars of d drinks is 1.5d

Share and Show – Page No. 403

Use properties of operations to write an equivalent expression by combining like terms.

Question 1.
\(3 \frac{7}{10} r-1 \frac{1}{5} r\)
Type below:
_____________

Answer: 2 \(frac{5}{10}\)r

Explanation:
3 \(frac{7}{10}\)r – 1 \(frac{1}{5}\)r
3 + \(frac{7}{10}\)r – 1 – \(frac{1}{5}\)r
3 – 1 = 2
\(frac{7}{10}\)r – \(frac{1}{5}\)r
\(frac{7}{10}\)r – \(frac{2}{10}\)r = \(frac{5}{10}\)r
\(3 \frac{7}{10} r-1 \frac{1}{5} r\) = 2 \(frac{5}{10}\)r

Question 2.
20a + 18 + 16a
Type below:
_____________

Answer: 36a + 18

Explanation:
Combine  the like terms first
16a and 20a are like terms in the given expression.
Add 16a and 20a
16a + 20a +18 = 36a + 18

Question 3.
7s + 8t + 10s + 12t
Type below:
_____________

Answer: 17s + 20t

Explanation:
There are 4 terms in the expression they are 7s, 10s, 8t, 12t.
Now combine the like terms
7s + 10s + 8t + 12t = 17s + 20t

Use the Distributive Property to write an equivalent expression.

Question 4.
8(h + 1.5)
Type below:
_____________

Answer: 8h + 12

Explanation:
Here we have to use the distributive property for the above expression.
8(h + 1.5) = 8 × h + 1.5 × 8
= 8h + 12
Thus 8(h + 1.5) is 8h + 12.

Question 5.
4m + 4p
Type below:
_____________

Answer: 4(m + p)

Explanation:
Here we have to take 4 as a common factor from the expression.
4m + 4p = 4 × m + 4 × p
That implies 4 × (m + p)

Question 6.
3a + 9b
Type below:
_____________

Answer: 3(a + 3b)

Explanation:
Let us take 4 as a common factor from the expression.
3a + 9b = 3 × a + 9 × b
3(a + 3b)
3a + 9b = 3(a + 3b)

On Your Own

Practice: Copy and Solve Use the Distributive Property to write an equivalent expression.

Question 7.
3.5(w + 7)
Type below:
_____________

Answer: 3.5w + 24.5

Explanation:
Use the distributive property.
Multiply within the parentheses.
3.5(w + 7) = 3.5 × w + 3.5 × 7
3.5w + 24.5
Thus 3.5(w + 7) = 3.5w + 24.5

Question 8.
\(\frac{1}{2}\)(f + 10)
Type below:
_____________

Answer: \(\frac{1}{2}\)f + 5

Explanation:
\(\frac{1}{2}\)(f + 10)
Use the distributive property.
Multiply within the parentheses.
\(\frac{1}{2}\) × f + \(\frac{1}{2}\) × 10
= \(\frac{1}{2}\)f + 5
Thus \(\frac{1}{2}\)(f + 10) = \(\frac{1}{2}\)f + 5

Question 9.
4(3z + 2)
Type below:
_____________

Answer: 12z + 8

Explanation:
Use the distributive property.
Multiply within the parentheses.
4(3z + 2) = 4 × 3z + 4 × 2
= 12z + 8
So, 4(3z + 2) = 12z + 8

Question 10.
20b + 16c
Type below:
_____________

Answer: 4(5b + 4c)

Explanation:
20b + 16c
Use the distributive property.
Multiply within the parentheses.
Take 4 as a common factor.
20b + 16c = 4 × 5b + 4 × 4c = 4 (5b + 4c)
Thus the expression 20b + 16c = 4 (5b + 4c)

Question 11.
30d + 18
Type below:
_____________

Answer: 6(5d + 3)

Explanation:
30 and 18 are the factors of 6.
So, take 6 as a common factor.
30d + 18 = 6 × 5d + 6 × 3
6 (5d + 3)
30d + 18 = 6 (5d + 3)

Question 12.
24g − 8h
Type below:
_____________

Answer: 8(3g – h)

Explanation:
Given the expression 24g − 8h
24 and 8 are the factors of 8.
So, let us take 8 as a common factor.
24g − 8h = 8 × 3g – 8 × 1h
= 8(3g – h)

Question 13.
Write an Expression The lengths of the sides of a triangle are 3t, 2t + 1, and t + 4. Write an expression for the perimeter (sum of the lengths). Then, write an equivalent expression with 2 terms.
Type below:
_____________

Answer: 6t + 5

Explanation:
Given that, The lengths of the sides of a triangle are 3t, 2t + 1, and t + 4.
We know that the perimeter of the triangle is P = a + b + c
P = 3t + 2t + 1 + t + 4
Combine the like terms.
P = 6t + 5

Question 14.
Use properties of operations to write an expression equivalent to the sum of the expressions 3(g + 5) and 2(3g − 6).
Type below:
_____________

Answer: 3(3g + 1)

Explanation:
Given two expressions 3(g + 5) and 2(3g − 6).
Use the distributive property to simplify the expressions.
3(g + 5) = 3 × g + 3 × 5 = 3g + 15
2(3g − 6) = 2 × 3g – 2 × 6 = 6g – 12
Add both the expressions and combine the like terms
3g + 15 + 6g – 12 = 9g + 3 = 3(3g + 1)

Problem Solving + Applications – Page No. 404

Question 15.
Sense or Nonsense Peter and Jade are using what they know about properties to write an expression equivalent to 2 × (n + 6) + 3. Whose answer makes sense? Whose answer is nonsense? Explain your reasoning.
Peter’s Work:
Expression: 2 × (n + 6) + 3
Associative Property of Addition: 2 × n + (6 + 3)
Add within parentheses: 2 × n + 9
Multiply: 2n + 9

Jade’s Work:
Expression: 2 × (n + 6) + 3
Distributive Property: (2 × n) + (2 × 6) + 3
Multiply within parentheses: 2n + 12 + 3
Associative Property of Addition: 2n + (12 + 3)
Add within parentheses: 2n + 15
For the answer that is nonsense, correct the statement.
Type below:
_____________

Answer: Jade’s Work makes sense. Peter’s Work makes non-sense because
He must have multiplied n + 6 with 2 but he added 6 with 3.
2 × (n + 6) + 3
2 × n + 2 × 6 + 3 = 2n + 12 + 3
= 2n + 15

Question 16.
Write the algebraic expression in the box that shows an equivalent expression.
Go Math Grade 6 Answer Key Chapter 7 Exponents img 11
Type below:
_____________

Answer:
6(z + 5) = 6 × z + 6 × 5 = 6z + 30
6z + 5z = z(6 + 5) = 11z
2 + 6z + 3 = 6z + 5

Generate Equivalent Expressions – Page No. 405

Use properties of operations to write an equivalent expression by combining like terms.

Question 1.
7h − 3h
Type below:
_____________

Answer: 4h

Explanation:
Combine the like terms
7h and 3h are the common terms
Now subtract 3h from 7h
7h – 3h = 4h

Question 2.
5x + 7 + 2x
Type below:
_____________

Answer: 7x + 7

Explanation:
The given expression is 5x + 7 + 2x
The common terms are 5x and 2x
Combine the like terms 5x + 7 + 2x = 7x + 7

Question 3.
16 + 13p − 9p
Type below:
_____________

Answer: 16 + 4p

Explanation:

Combine the like terms for the above expressions.
The like terms are 13p and 9p
16 + 13p − 9p = 16 + 4p

Question 4.
y2 + 13y − 8y
Type below:
_____________

Answer: y2 + 5y

Explanation:
The given expression is y2 + 13y − 8y
The like terms are 13y and 8y
y2 + 13y − 8y = y2 + 5y

Question 5.
5(2h + 3) + 3h
Type below:
_____________

Answer: 13h + 15

Explanation:
5(2h + 3) + 3h = 10h + 15 + 3h
The like terms are 10h and 3h
10h + 15 + 3h = 13h + 15

Question 6.
12 + 18n + 7 − 14n
Type below:
_____________

Answer: 19 + 4n

Explanation:
The expression is 12 + 18n + 7 − 14n
The like terms are 18n and 14n
12 + 18n + 7 − 14n = 19 + 4n

Use the Distributive Property to write an equivalent expression.

Question 7.
2(9 + 5k)
Type below:
_____________

Answer: 18 + 10k

Explanation:
Use the Distributive property
Multiply within the parentheses.
2(9 + 5k) = (2 × 9) + (2 × 5k)
(2 × 9) + (2 × 5k) = 18 + 10k

Question 8.
4d + 8
Type below:
_____________

Answer: 4(d + 2)

Explanation:
Use the Distributive property
Multiply within the parentheses.
4d + 8 = 4 × d + 4 × 2
The common term is 4.
Take 4 as a common factor.
4d + 8 = 4 (d + 2)

Question 9.
21p + 35q
Type below:
_____________

Answer: 7(3p + 5q)

Explanation:
Use the Distributive property
Multiply within the parentheses.
7 × 3p + 7 × 5q
The common term is 7.
7(3p + 5q)
21p + 35q = 7(3p + 5q)

Problem Solving

Question 10.
The expression 15n + 12n + 100 represents the total cost in dollars for skis, boots, and a lesson for n skiers. Simplify the expression 15n + 12n + 100. Then find the total cost for 8 skiers.
Type below:
_____________

Answer: 27n + 100, $316

Explanation:
The terms that have n can be operated:
15n +12n + 100 = 27n +100. Then, we have that
total cost = 27n +100 for n skiers. So, for 8 skiers we have
total cost = 27(8) +100 = 216 + 100 = 316.
Then, the total cost of 8 skiers is $316.

Question 11.
Casey has n nickels. Megan has 4 times as many nickels as Casey has. Write an expression for the total number of nickels Casey and Megan have. Then simplify the expression.
Type below:
_____________

Answer: n + 4n; 5n

Explanation:
Casey has n nickels. Megan has 4 times as many nickels as Casey has.
Sum of n and 4n
Add the common terms n and 4n.
n + 4n = 5n

Question 12.
Explain how you would use properties to write an expression equivalent to 7y + 4b – 3y.
Type below:
_____________

Answer:
1st you combine like terms so subtract 7y and 3y and you get 4y.
So this is the final answer: 4y+4b.

Lesson Check – Page No. 406

Question 1.
A ticket to a museum costs $8. A ticket to the dinosaur exhibit costs $5. The expression 8n + 5n represents the cost in dollars for n people to visit the museum and the exhibit. What is a simpler form of the expression 8n + 5n?
Type below:
_____________

Answer: 13n

Explanation:
A ticket to a museum costs $8. A ticket to the dinosaur exhibit costs $5.
The expression is the sum of 8n and 5n.
Thus the simpler form of the expression is 8n + 5n = 13n

Question 2.
What is an expression that is equivalent to 3(2p – 3)?
Type below:
_____________

Answer: 6p – 9

Explanation:
Use the distributive property to find the equivalent expression.
3(2p – 3) = 3 × 2p – 3 × 3
= 6p – 9
Thus the expression that is equivalent to 3(2p – 3) is 6p – 9.

Question 3.
A Mexican restaurant received 60 take-out orders. The manager found that 60% of the orders were for tacos and 25% of the orders were for burritos. How many orders were for other items?
______ orders

Answer: 9 orders

Explanation:
Given,
A Mexican restaurant received 60 take-out orders.
The manager found that 60% of the orders were for tacos and 25% of the orders were for burritos.
The answer is 9 because 25% of 60 is 15 plus 60% of 60 is 36 so 36+15=51 and 60-51=9
Thus 9 orders were for other items.

Question 4.
The area of a rectangular field is 1,710 square feet. The length of the field is 45 feet. What is the width of the field?
______ feet

Answer: 38 feet

Explanation:
The area of a rectangular field is 1,710 square feet.
The length of the field is 45 feet.
The width of the field is x feet
A = l × w
1710 square feet = 45 feet × x
x = 1710/45 = 38 feet
Thus the width of the rectangular field is 38 feet.

Question 5.
How many terms are in 2 + 4x + 7y?
______ terms

Answer: 3

Explanation:
Given expression 2 + 4x + 7y
There are 3 terms in the expression 2, 4x, 7y.

Question 6.
Boxes of cereal usually cost $4, but they are on sale for $1 off. A gallon of milk costs $3. The expression 4b – 1b + 3 can be used to find the cost in dollars of buying b boxes of cereal and a gallon of milk. Write the expression in a simpler form.
Type below:
_____________

Answer: 3b + 3

Explanation:
Boxes of cereal usually cost $4, but they are on sale for $1 off. A gallon of milk costs $3.
The expression is 4b – 1b + 3
Combine the like terms for the above expression.
4b – 1b + 3 = 3b + 3 = 3(b + 1)

Share and Show – Page No. 409

Use properties of operations to determine whether the expressions are equivalent.

Question 1.
7k + 4 + 2k and 4 + 9k
The expressions are _____________

Answer: equivalent

Explanation:
7k + 4 + 2k
Combine the like terms 7k and 2k
Add the like terms 7k + 2k + 4 = 9k + 4
9k + 4 and 4 + 9k are equivalent.

Question 2.
9a × 3 and 12a
The expressions are _____________

Answer: not equivalent

Explanation:
Multiply 9a with 3.
9a × 3 = 27a
27a and 12a are not equivalent.
Thus the expressions are not equivalent.

Question 3.
8p + 0 and 8p × 0
The expressions are ______

Answer: not equivalent

Explanation:
8p + 0 = 8p
8p × 0 = 0
8p and 0 are not equivalent.
The expressions 8p + 0 and 8p × 0 are not equivalent.

Question 4.
5(a + b) and (5a + 2b) + 3b
The expressions are _____________

Answer: equivalent

Explanation:
5(a + b) = 5a + 5b
(5a + 2b) + 3b
The like terms are 5a and 2b, 3b
Add the combine terms 5a + 2b + 3b = 5a + 5b
Thus the expressions 5(a + b) and (5a + 2b) + 3b are equivalent.

On Your Own

Use properties of operations to determine whether the expressions are equivalent.

Question 5.
3(v + 2) + 7v and 16v
The expressions are _____________

Answer: not equivalent

Explanation:
3(v + 2) + 7v
Combine the like terms 3v and 7v
3(v + 2) + 7v = 3v + 6 + 7v = 10v + 6
The expressions 10v + 6 and 16v are not equivalent.

Question 6.
14h + (17 + 11h) and 25h + 17
The expressions are _____________

Answer: equivalent

Explanation:
14h + (17 + 11h)
Combine the like terms 14h and 11h.
14h + 17 + 11h = 25h + 17
The expressions 14h + (17 + 11h) and 25h + 17 are equivalent.

Question 7.
4b × 7 and 28b
The expressions are _____________

Answer: equivalent

Explanation:
Multiply 4b with 7.
4b × 7 = 28b
The expressions 4b × 7 and 28b are equivalent.

Question 8.
Each case of dog food contains c cans. Each case of cat food contains 12 cans. Four students wrote the expressions below for the number of cans in 6 cases of dog food and 1 case of cat food. Which of the expressions are correct?
6c + 12     6c × 12      6(c + 2)      (2c + 4) × 3
Type below:
_____________

Answer: The correct expressions are 6c + 12, 6(c + 2), (2c + 4) × 3
6(c + 2) is the distributive form of the expression.

Problem Solving + Applications – Page No. 410

Use the table for 9–11.
Go Math Grade 6 Answer Key Chapter 7 Exponents img 12

Question 9.
Marcus bought 4 packets of baseball cards and 4 packets of animal cards. Write an algebraic expression for the total number of cards Marcus bought.
Type below:
_____________

Answer: 4a + 4b

Explanation:
Marcus bought 4 packets of baseball cards and 4 packets of animal cards.
b represents the number per packet
Multiply 4 with b
4 × b = 4b
a represents the number per packet of animal cards.
Multiply 4 with a.
4 × a = 4a
Therefore the algebraic expression for the total number of cards Marcus bought is the sum of 4a and 4b.
The expression is 4a + 4b

Question 10.
Make Arguments Is the expression for the number of cards Marcus bought equivalent to 4(a + b)? Justify your answer.
Type below:
_____________

Answer: Yes
Use the distributive property to simplify the expression 4a + 4b.
Take 4 as the common factor for the expression 4a + 4b.
4a + 4b = 4(a + b)

Question 11.
Angelica buys 3 packets of movie cards and 6 packets of cartoon cards and adds these to the 3 packets of movie cards she already has. Write three equivalent algebraic expressions for the number of cards Angelica has now
Type below:
_____________

Answer: 3m + 6c + 3m

Explanation:
Angelica buys 3 packets of movie cards and 6 packets of cartoon cards and adds these to the 3 packets of movie cards she already has.
The expression for 3 packets of movie cards is 3m
The expression for 6 packets of cartoon cards is 6c.
Now we have to add 3m to the expression.
3m + 6c + 3m
Thus the three equivalent algebraic expressions for the number of cards Angelica has now is 3m + 6c + 3m

Question 12.
Select the expressions that are equivalent to 3(x + 2). Mark all that apply.
Options:
a. 3x + 6
b. 3x + 2
c. 5x
d. x + 5

Answer: 3x + 6

Explanation:
Use distributive property to solve the expression 3(x + 2).
3(x + 2) = 3 × x + 3 × 2 = 3x + 6
Thus the correct answer is option A.

Identify Equivalent Expressions – Page No. 411

Use properties of operations to determine whether the expressions are equivalent.

Question 1.
2s + 13 + 15s and 17s + 13
The expressions are _____________

Answer: equivalent

Explanation:
2s + 13 + 15s
Combine the like terms
2s + 13 + 15s = 17s + 13
17s + 13 = 17s + 13
Thus the expressions 2s + 13 + 15s and 17s + 13 are equivalent.

Question 2.
5 × 7h and 35h
The expressions are _____________

Answer: equivalent

Explanation:
5 × 7h = 35h
35h = 35h
The expressions 5 × 7h and 35h are equivalent.

Question 3.
10 + 8v − 3v and 18 − 3v
The expressions are _____________

Answer: not equivalent

Explanation:
Combine the like terms 8v and 3v
10 + 8v − 3v = 10 + 5v
10 + 5v ≠ 18 − 3v
Thus the expressions 10 + 8v − 3v and 18 − 3v are not equivalent.

Question 4.
(9w × 0)−12 and 9w – 12
The expressions are _____________

Answer: not equivalent

Explanation:
(9w × 0)−12 = 0 – 12 = – 12
– 12 ≠ 9w – 12
So, the expressions (9w × 0)−12 and 9w – 12 are not equivalent.

Question 5.
11(p + q) and 11p + (7q + 4q)
The expressions are _____________

Answer: equivalent

Explanation:
11(p + q) = 11p + 11q
Combine the terms 7q and 4q
11p + (7q + 4q) = 11p + 11q = 11(p + q)
So, the expressions 11(p + q) and 11p + (7q + 4q) are equivalent.

Question 6.
6(4b + 3d) and 24b + 3d
The expressions are _____________

Answer: not equivalent

Explanation:
6(4b + 3d) = 24b + 18d
24b + 18d ≠ 24b + 3d
So, the expressions 6(4b + 3d) and 24b + 3d are not equivalent.

Question 7.
14m + 9 − 6m and 8m + 9
The expressions are _____________

Answer: equivalent

Explanation:
Combine the like terms 14m and 6m
14m + 9 − 6m = 8m + 9
8m + 9 = 8m + 9
Thus the expressions are equivalent.

Question 8.
(y × 1) + 2 and y + 2
The expressions are _____________

Answer: equivalent

Explanation:
(y × 1) + 2 = y + 2
y + 2 = y + 2
Thus the expressions (y × 1) + 2 and y + 2 are equivalent.

Question 9.
4 + 5(6t + 1) and 9 + 30t
The expressions are _____________

Answer: equivalent

Explanation:
4 + 5(6t + 1) = 4 + 30t + 5 = 9 + 30t
9 + 30t = 9 + 30t
Thus the expressions 4 + 5(6t + 1) and 9 + 30t are equivalent.

Question 10.
9x + 0 + 10x and 19x + 1
The expressions are _____________

Answer: not equivalent

Explanation:
9x + 0 + 10x
Combine the like terms 9x and 10x.
9x + 10x = 19x
19x ≠ 19x + 1
Thus the expressions 9x + 0 + 10x and 19x + 1 are not equivalent.

Question 11.
12c − 3c and 3(4c − 1)
The expressions are _____________

Answer: not equivalent

Explanation:
12c − 3c
Take 3 as a common factor.
3c(4 – 1) or 3 (4c – 1c)
3 (4c – 1c) ≠ 3(4c − 1)
Thus the expressions 12c − 3c and 3(4c − 1) are not equivalent.

Question 12.
6a × 4 and 24a
The expressions are _____________

Answer: equivalent

Explanation:
6a × 4 = 24a
24a = 24a
The expressions 6a × 4 and 24a are equivalent.

Problem Solving

Question 13.
Rachel needs to write 3 book reports with b pages and 3 science reports with s pages during the school year. Write an algebraic expression for the total number of pages Rachel will need to write.
Type below:
_____________

Answer: 3b + 3s

Explanation:
Rachel needs to write 3 book reports with b pages and 3 science reports with s pages during the school year.
Multiply 3 book reports with b pages = 3b.
Multiply 3 science books with s pages = 3s.
The algebraic expression for the total number of pages Rachel will need to write is 3b + 3s.

Question 14.
Rachel’s friend Yassi has to write 3(b + s) pages for reports. Use properties of operations to determine whether this expression is equivalent to the expression for the number of pages Rachel has to write.
This expression is _____________

Answer: equivalent

Explanation:
Rachel’s friend Yassi has to write 3(b + s) pages for reports.
The equivalent expression of 3(b + s) = 3b + 3s

Question 15.
Use properties of operations to show whether 7y + 7b + 3y and 7(y + b) + 3b are equivalent expressions. Explain your reasoning.
Type below:
_____________

Answer:
Use Distributive property to simplify the expressions.
The equivalent expression of 7y + 7b + 3y = 7(y + b) + 3y
Thus 7y + 7b + 3y and 7(y + b) + 3b are equivalent.

Lesson Check – Page No. 412

Question 1.
Ian had 4 cases of comic books and 6 adventure books. Each case holds c comic books. He gave 1 case of comic books to his friend. Write an expression that gives the total number of books Ian has left.
Type below:
_____________

Answer: 3c + 6

Explanation:
Ian had 4 cases of comic books and 6 adventure books. Each case holds c comic books. He gave 1 case of comic books to his friend.
4c + 6 – 1c
Combine the like terms
3c + 6

Question 2.
In May, Xia made 5 flower planters with f flowers in each planter. In June, she made 8 flower planters with f flowers in each planter. Write an expression in the simplest form that gives the number of flowers Xia has in the planters.
Type below:
_____________

Answer: 13f

Explanation:
In May, Xia made 5 flower planters with f flowers in each planter.
The expression is 5f
In June, she made 8 flower planters with f flowers in each planter.
The expression is 8f.
Sum of 5f and 8f is 8f + 5f = 13f

Spiral Review

Question 3.
Keisha wants to read for 90 minutes. So far, she has read 30% of her goal. How much longer does she need to read to reach her goal?
________ minutes

Answer: 63 min

Explanation:
Keisha wants to read for 90 minutes.
So far, she has read 30% of her goal.
30% = 30/100 = 0.3
Multiply 90 with 0.3
90 × 0.3 = 27
Subtract 27 from 90
90 – 27 = 63
She needs to read 63 minutes to reach her goal.

Question 4.
Marvyn travels 105 miles on his scooter. He travels for 3 hours. What is his average speed?
________ miles per hour

Answer: 35 miles per hour

Explanation:
Divide the number of miles by hours traveled.
Average speed = 105 miles/3 hours = 35 miles per hour
Thus the average speed is 35 miles per hour.

Question 5.
The expression 5(F − 32) ÷ 9 gives the Celsius temperature for a Fahrenheit temperature of F degrees. The noon Fahrenheit temperature in Centerville was 86 degrees. What was the temperature in degrees Celsius?
________ degrees Celsius

Answer: 30 degrees Celsius

Explanation:
The expression is 5(F − 32) ÷ 9
F = 86 degrees
Substitute F in the above expression.
5(86 − 32) ÷ 9 = 5(54) ÷ 9
270 ÷ 9 = 30
The temperature is 30 degrees Celsius

Question 6.
At the library book sale, hardcover books sell for $4 and paperbacks sell for $2. The expression 4b + 2b represents the total cost for b hardcover books and b paperbacks. Write a simpler expression that is equivalent to 4b + 2b.
Type below:
_____________

Answer: 6b

Explanation:
Given expression is 4b + 2b
The terms are 4b and 2b
Now combine the like terms
That means 4b + 2b = 6b

Chapter 7 Review/Test – Page No. 413

Question 1.
Use exponents to rewrite the expression.
3 × 3 × 3 × 3 × 5 × 5
Type below:
_____________

Answer: 34 × 52

Explanation:
3 is a repeated factor.
The number 3 is repeated four times.
5 is a repeated factor.
The number 5 is repeated two times.
The exponential form of 3 × 3 × 3 × 3 × 5 × 5 is 34 × 52

Question 2.
A plumber charges $10 for transportation and $55 per hour for repairs. Write an expression that can be used to find the cost in dollars for a repair that takes h hours.
Type below:
_____________

Answer: 10 + 55h

Explanation:
A plumber charges $10 for transportation and $55 per hour for repairs.
Multiply 55 with an hour
Sum of 10 and product of 55 and h.
The expression is 10 + 55h.

Question 3.
Ellen is 2 years older than her brother Luke. Let k represent Luke’s age. Identify the expression that can be used to find Ellen’s age.
Options:
a. k−2
b. k+2
c. 2k
d. \(\frac{k}{2}\)

Answer: k+2

Explanation:
Given, Ellen is 2 years older than her brother Luke. Let k represent Luke’s age.
Older is nothing but more so we have to add 2 years to k.
That means k + 2.
Thus the correct answer is option B.

Question 4.
Write 43 using repeated multiplication. Then find the value of 43.
________

Answer:
43 = 4 × 4 × 4 = 64
The value of 43 is 64.

Question 5.
Jasmine is buying beans. She bought r pounds of red beans that cost $3 per pound and b pounds of black beans that cost $2 per pound. The total amount of her purchase is given by the expression 3r + 2b. Select the terms of the expression. Mark all that apply
Options:
a. 2
b. 2b
c. 3
d. 3r

Answer: B, D

Explanation:
The expression is 3r + 2b
The terms of the expressions are 3r and 2b.
Thus the correct answers are B and D.

Chapter 7 Review/Test – Page No. 414

Question 6.
Choose the number that makes the sentence true. The formula V= s3 gives the volume V of a cube with side length s.
The volume of a cube that has a side length of 8 inches
inches is _____________ cubed

Answer: 512

Explanation:
Use the formula V= s3
s = 8
V = 83 = 8 × 8 × 8 = 512

Question 7.
Liang is ordering new chairs and cushions for his dining room table. A new chair costs $88 and a new cushion costs $12. Shipping costs $34. The expression 88c + 12c + 34 gives the total cost for buying c sets of chairs and cushions. Simplify the expression by combining like terms.
Type below:
_____________

Answer: 100c + 34

Explanation:
Liang is ordering new chairs and cushions for his dining room table.
A new chair costs $88 and a new cushion costs $12. Shipping costs $34.
The expression is 88c + 12c + 34.
Combine the like terms
88c + 12c + 34 = 100c + 34

Question 8.
Mr. Ruiz writes the expression 5 × (2 + 1)2 ÷ 3 on the board. Chelsea says the first step is to evaluate 12. Explain Chelsea’s mistake. Then, evaluate the expression
_____________

Answer:
She should have done what was in the parentheses (2 + 1) and then the exponent 32= 9
5 × (2 + 1)2 ÷ 3 = 5 × 9 ÷ 3
5 × 3 = 15

Question 9.
Jake writes this word expression.
Go Math Grade 6 Answer Key Chapter 7 Exponents img 13
Write an algebraic expression for the word expression. Then, evaluate the expression for m = 4. Show your work.
________

Answer:
The expression is 7m
Replace m = 4 with m
7m = 7 × 4 = 28

Chapter 7 Review/Test – Page No. 415

Question 10.
Sora has some bags that each contain 12 potatoes. She takes 3 potatoes from each bag. The expression 12p – 3p represents the number of potatoes p left in the bags. Simplify the expression by combining like terms. Draw a line to match the expression with the simplified expression.
Go Math Grade 6 Answer Key Chapter 7 Exponents img 14
Type below:
_____________

Answer: 9p
Go-Math-Grade-6-Answer-Key-Chapter-7-Exponents-img-14

Question 11.
Logan works at a florist. He earns $600 per week plus $5 for each floral arrangement he delivers. Write an expression that gives the amount in dollars that Logan earns for delivering f floral arrangements. Use the expression to find the amount Logan will earn if he delivers 45 floral arrangements in one week. Show your work.
$ ________

Answer: $825

Explanation:
Logan works at a florist. He earns $600 per week plus $5 for each floral arrangement he delivers. Write an expression that gives the amount in dollars that Logan earns for delivering f floral arrangements.
The expression is 600 + 5f
f = 45
600 + 5f = 600 + 5(45)
600 + 225 = 825
Thus Logan earned $825 for delivering f floral arrangements.

Question 12.
Choose the word that makes the sentence true.
Dara wrote the expression 7 × (d + 4) in her notebook. She used the _____ Property to write the equivalent expression 7d + 28.
Answer: Dara wrote the expression 7 × (d + 4) in her notebook. She used the Distributive Property to write the equivalent expression 7d + 28.
Use the distributive property to simplify the expression.
7 × (d + 4) = 7d + 28

Chapter 7 Review/Test – Page No. 416

Question 13.
Use properties of operations to determine whether 5(n + 1) + 2n and 7n + 1 are equivalent expressions.
The expressions are _____________

Answer:
5n + 5 + 2n is Distributive property
5n + 2n + 5 is Commutative property of addition
7n + 5 combine like term
5n + 5 + 2n is equivalent to 7n + 5
Since it is not equivalent to 7n + 1, 7n + 5 is not equivalent to 7n.

Question 14.
Alisha buys 5 boxes of peanut butter granola bars and 5 boxes of cinnamon granola bars. Let p represent the number of bars in a box of peanut butter granola bars and c represents the number of bars in a box of cinnamon granola bars. Jaira and Emma each write an expression that represents the total number of granola bars Alisha bought. Are the equivalent of the expression? Justify your answer
Jaira
5p + 5c
Emma
5(p + c)
Type below:
_____________

Answer:
They are equivalent statements.
5p + 5c = 5(p + c) by the distributive property.

Question 15.
Abe is 3 inches taller than Chen. Select the expressions that represent Abe’s height if Chen’s height is h inches. Mark all that apply
Options:
a. h−3
b. h+3
c. the sum of h and 3
d. the difference between h and 3

Answer:
Abe is 3 inches taller than Chen.
Let Chen’s height is h.
The expression is the sum of Chen’s height and 3.
So, the suitable answers are h + 3 and the sum of h and 3.
Thus the correct answers are option B and C.

Question 16.
Write the algebraic expression in the box that shows an equivalent expression.
Go Math Grade 6 Answer Key Chapter 7 Exponents img 15
Type below:
_____________

Answer:
3(k + 2) = 3k + 6
3k + 2k = 5k
2 + 6k + 3 = 6k + 5

Chapter 7 Review/Test – Page No. 417

Question 17.
Draw a line to match the property with the expression that shows the property.
Go Math Grade 6 Answer Key Chapter 7 Exponents img 16
Type below:
_____________

Answer:
Go-Math-Grade-6-Answer-Key-Chapter-7-Exponents-img-16

Question 18.
A bike rental company charges $10 to rent a bike plus $2 for each hour the bike is rented. An expression for the total cost of renting a bike for h hours is 10 + 2h. Complete the table to find the total cost of renting a bike for h hours.
Go Math Grade 6 Answer Key Chapter 7 Exponents img 17
Type below:
_____________

Answer:
Go-Math-Grade-6-Answer-Key-Chapter-7-Exponents-img-17

Question 19.
An online sporting goods store charges $12 for a pair of athletic socks. Shipping is $2 per order
Part A
Write an expression that Hana can use to find the total cost in dollars for ordering n pairs of socks.
Type below:
_____________

Answer: 12n + 2

Explanation:
Let n represents a pair of socks.
Multiply the price of pair of athletic socks with pair of socks = 12 × n
Shipping is $2 per order
The expression for the total cost in dollars for ordering n pairs of socks is 12n + 2

Question 19.
Part B
Hana orders 3 pairs of athletic socks and her friend, Charlie, order 2 pairs of athletic socks. What is the total cost, including shipping, for both orders? Show your work.
$ ________

Answer:
The cost of Hannah’s order is 12 × 3 + 2 = 36 + 2 = 38
The cost of Charlie’s order is 12 × 2 + 2 = 24 + 2 = 26
The total cost for both is 38 + 26 = 64

Chapter 7 Review/Test – Page No. 418

Question 20.
Fernando simplifies the expression (6 + 2)2 – 4 × 3.
Part A
Fernando shows his work on the board. Use numbers and words to explain his mistake.
(6 + 2)2 – 4 × 3
(6 + 4) – 4 × 3
10 − 4 × 3
6 × 3
18
Type below:
_____________

Answer: Fernando did not use the correct order of operations. He should have added 6 and 2, then evaluate the exponent. He also subtracted before multiplying. He should have multiplied first.

Question 20.
Part B
Simplify the expression (6 + 2)2 − 4 × 3 using the order of operations.
_______

Answer: 52

Explanation:
(6 + 2)2 − 4 × 3
First, add 6 and 2 and then subtract with 12.
82 – 4 × 3
= 64 – 12 = 52
(6 + 2)2 − 4 × 3 = 52

Conclusion:

In addition to the exercise and homework problems, we have given the solutions for the mid-chapter and review test. Hence the students of grade 6 can check whether the answers are right or wrong. Feel free to post your comments in the below comment box if you have any queries. Bookmark our ccssmathanswers.com to get the go math answer key for all grade 6 chapters.

Go Math Grade 5 Answer Key Chapter 7 Multiply Fractions

go-math-grade-5-chapter-7-multiply-fractions-answer-key

If you are looking for Chapter 7 Go Math Grade 5 Answer Key then you have reached the right destination.  We have mentioned Go Math Grade 5 Answer Key Chapter 7 Multiply Fractions with straight forward explanation for all the Problems. Parents who wanted to give their kids enough practice on concepts of Multiplying Fractions can give them the Go Math 5th Grade Answer Key Chapter 7 Multiply Fractions. Enhance your Math Skills by practicing from the HMH Go Math Grade 5 Answer Key Ch 7 Multiply Fractions.

Go Math Grade 5 Answer Key Chapter 7 Multiply Fractions

Do you feel the concept of Multiplying Fractions difficult? Not anymore with our Go Math Answer Key Grade 5 Chapter 7 Multiply Fractions as they have concepts explained in detail. Start learning the basics involved right from the basic level with our 5th Grade Go Math Answer Key Ch 7 Multiply Fractions. Test your preparation level by answering the questions at the end of the Go Math Grade 5 Answer Key for Multiply Fractions. By doing so, you can have an estimation of your preparation standards.

Chapter 7 – Lesson 1: Find Part of a Group

Chapter 7 – Lesson 2: Investigate • Multiply Fractions and Whole Numbers

Chapter 7 – Lesson 3: Fraction and Whole Number Multiplication

Chapter 7 – Lesson 4: Investigate • Multiply Fractions

Chapter 7 – Lesson 5: Compare Fraction Factors and Products

Chapter 7 – Lesson 6: Fraction Multiplication

Chapter 7 – Mid-Chapter Checkpoint

Chapter 7 – Lesson 7: Investigate • Area and Mixed Numbers

Chapter 7 – Lesson 8: Compare Mixed Number Factors and Products

Chapter 7 – Lesson 9: Multiply Mixed Numbers

Chapter 7 – Lesson 10: Problem Solving • Find Unknown Lengths

Chapter 7 – Chapter 7 Review/Test

Share and Show – Page No. 293

Question 1.
Complete the model to solve.
Go Math Grade 5 Answer Key Chapter 7 Multiply Fractions img 1
\(\frac{7}{8}\) of 16, or \(\frac{7}{8}\) × 16
How many rows of counters are there?
_____ rows

Answer: 8
By seeing the above figure we can say that the number of counters is 8 rows.

Question 1.
How many counters are in each row?
_____ counters

Answer: 2
There are 2 counters in each row.

Question 1.
Circle ____ rows to solve the problem.
_____ rows

Answer: 7

• • • • • • •
• • • • • • •
• • • • • • •
• • • • • • •
• • • • • • •
• • • • • • •
• • • • • • •
• • • • • • •

Question 1.
How many counters are circled?
\(\frac{7}{8}\) of 16=
or \(\frac{7}{8}\) × 16 =
_____ counters

Answer: 14
\(\frac{7}{8}\) × 16
8 divides 16 two times.
So, \(\frac{7}{8}\) × 16 = 7 × 2 = 14
Therefore 14 counters are circled.

Use a model to solve.

Question 2.
\(\frac{2}{3}\) × 18 = _____

Answer: 12

Explanation:
\(\frac{2}{3}\) × 18
3 divides 18 six times.
2 × 6 = 12

Question 3.
\(\frac{2}{5}\) × 15 = _____

Answer: 6

Explanation:
\(\frac{2}{5}\) × 15
5 divides 15 three times.
2 × 3 = 6
Thus \(\frac{2}{5}\) × 15 = 6

Question 4.
\(\frac{2}{3}\) × 6 = _____

Answer: 4

Explanation:
\(\frac{2}{3}\) × 6
3 divides 6 two times.
\(\frac{2}{3}\) × 6
2 × 2 = 4
\(\frac{2}{3}\) × 6 = 4

On Your Own

Use a model to solve.

Question 5.
\(\frac{5}{8}\) × 24 = _____

Answer: 15

Explanation:
\(\frac{5}{8}\) × 24
8 divides 24 three times.
5 × 3 = 15
\(\frac{5}{8}\) × 24 = 15

Question 6.
\(\frac{3}{4}\) × 24 = _____

Answer: 18

Explanation:
\(\frac{3}{4}\) × 24
4 divides 24 six times.
\(\frac{3}{4}\) × 24 = 3 × 6 = 18
So, \(\frac{3}{4}\) × 24 = 18

Question 7.
\(\frac{4}{7}\) × 21 = _____

Answer: 12

Explanation:
\(\frac{4}{7}\) × 21
7 divides 21 three times.
4 × 3 = 12
\(\frac{4}{7}\) × 21 = 12

Question 8.
\(\frac{2}{9}\) × 27 = _____

Answer: 6

Explanation:
\(\frac{2}{9}\) × 27
9 divides 27 three times.
2 × 3 = 6
\(\frac{2}{9}\) × 27 = 6

Question 9.
\(\frac{3}{5}\) × 20 = _____

Answer: 12

Explanation:
\(\frac{3}{5}\) × 20
5 divides 20 four times.
3 × 4 = 12
Thus \(\frac{3}{5}\) × 20 = 12

Question 10.
\(\frac{7}{11}\) × 22 = _____

Answer: 14

Explanation:
\(\frac{7}{11}\) × 22
11 divides 22 two times.
7 × 2 = 14
\(\frac{7}{11}\) × 22 = 14

Problem Solving – Page No. 294

Use the table for 11-12.
Go Math Grade 5 Answer Key Chapter 7 Multiply Fractions img 2

Question 11.
Four-fifths of Zack’s stamps have pictures of animals. How many stamps with pictures of animals does Zack have? Use a model to solve.
_____ stamps

Answer: 24 stamps

Explanation:
Given that, Four-fifths of Zack’s stamps have pictures of animals.
Number of stamps that Zack collected is 30
30 × \(\frac{4}{5}\)
5 divides 30 six times.
6 × 4 = 24
Zack has 24 stamps with pictures of animals.

Question 12.
Zack, Teri, and Paco combined the foreign stamps from their collections for a stamp show. Out of their collections, \(\frac{3}{10}\) of Zack’s stamps, \(\frac{5}{6}\) of Teri’s stamps, and \(\frac{3}{8}\) of Paco’s stamps were from foreign countries. How many stamps were in their display? Explain how you solved the problem.
_____ stamps

Answer: 33 stamps

Explanation:
Zack, Teri, and Paco combined the foreign stamps from their collections for a stamp show.
Out of their collections, \(\frac{3}{10}\) of Zack’s stamps, \(\frac{5}{6}\) of Teri’s stamps, and \(\frac{3}{8}\) of Paco’s stamps were from foreign countries.
Number of stamps Zack collected = 30
Number of stamps Teri collected = 18
Number of stamps Paco collected = 24
\(\frac{3}{10}\) of 30
\(\frac{3}{10}\) × 30 = 3 × 3 = 9
\(\frac{5}{6}\) × 18 = 5 × 3 = 15
\(\frac{3}{8}\) × 24 = 3 × 3 = 9
Now add all the stamps = 9 + 9 + 15 = 33

Question 13.
Paula has 24 stamps in her collection. Among her stamps, \(\frac{1}{3}\) have pictures of animals. Out of her stamps with pictures of animals, \(\frac{3}{4}\) of those stamps have pictures of birds. How many stamps have pictures of birds on them?
_____ stamps

Answer: 6 stamps

Explanation:
Paula has 24 stamps in her collection. Among her stamps, \(\frac{1}{3}\) have pictures of animals.
Out of her stamps with pictures of animals, \(\frac{3}{4}\) of those stamps have pictures of birds.
\(\frac{1}{3}\) × \(\frac{3}{4}\) × 24 = 24/4 = 6
Therefore 6 stamps have pictures of birds.

Question 14.
Test Prep Barry bought 21 stamps from a hobby shop. He gave \(\frac{3}{7}\) of them to his sister. How many stamps did he have left?
Go Math Grade 5 Answer Key Chapter 7 Multiply Fractions img 3
Options:
a. 3 stamps
b. 6 stamps
c. 9 stamps
d. 12 stamps

Answer: 9 stamps

Explanation:
Test Prep Barry bought 21 stamps from a hobby shop. He gave \(\frac{3}{7}\) of them to his sister.
\(\frac{3}{7}\) × 21
7 divides 21 three times.
3 × 3 = 9 stamps.
Thus the correct answer is option C.

Share and Show – Page No. 297

Use the model to find the product.

Question 1.
\(\frac{5}{6}\) × 3
Go Math Grade 5 Answer Key Chapter 7 Multiply Fractions img 4
______ \(\frac{□}{□}\)

Answer: 2 \(\frac{1}{2}\)

Explanation:
Place three whole fractions strips side by side.
Find six fraction strips all with the same denominator that fits exactly under the three whole numbers.
Circle \(\frac{5}{6}\) of 3 on the model you drew.
Complete the number sentence. \(\frac{5}{6}\) of 3
\(\frac{5}{6}\) × 3 = \(\frac{5}{2}\)
2 \(\frac{1}{2}\)

Question 2.
2 × \(\frac{5}{6}\)
Go Math Grade 5 Answer Key Chapter 7 Multiply Fractions img 5
______ \(\frac{□}{□}\)

Answer: 1 \(\frac{2}{3}\)

Explanation:
Place two whole fractions strips side by side.
Find six fraction strips all with the same denominator that fits exactly under the two whole numbers.
2 of \(\frac{5}{6}\) = \(\frac{5}{6}\) × 2
\(\frac{5}{3}\)
The mixed fraction of \(\frac{5}{3}\) is 1 \(\frac{2}{3}\)

Find the product.

Question 3.
\(\frac{5}{12}\) × 3 = ______ \(\frac{□}{□}\)

Answer: 1 \(\frac{1}{4}\)

Explanation:
\(\frac{5}{12}\) × 3
Place three whole fractions strips side by side.
Find six fraction strips all with the same denominator that fits exactly under the two whole numbers.
\(\frac{5}{12}\) × 3
3 divides 12 four times
\(\frac{5}{12}\) × 3 = \(\frac{5}{4}\)
The mixed fraction of \(\frac{5}{4}\) is 1 \(\frac{1}{4}\)
\(\frac{5}{12}\) × 3 = 1 \(\frac{1}{4}\)

Question 4.
9 × \(\frac{1}{3}\) = ______

Answer: 3

Explanation:
9 × \(\frac{1}{3}\)
Place nine whole fractions strips side by side.
Find three fraction strips all with the same denominator that fits exactly under the two whole numbers.
9 × \(\frac{1}{3}\)
3 divides 9 three times.
9 × \(\frac{1}{3}\) = 3
Thus 9 × \(\frac{1}{3}\) = 3

Question 5.
\(\frac{7}{8}\) × 4 = ______ \(\frac{□}{□}\)

Answer: 3 \(\frac{1}{2}\)

Explanation:
\(\frac{7}{8}\) × 4
Place four whole fractions strips side by side.
\(\frac{7}{8}\) × 4
4 divides 8 two times.
\(\frac{7}{8}\) × 4 = \(\frac{7}{2}\)
The mixed fraction of  \(\frac{7}{2}\) is 3 \(\frac{1}{2}\)
\(\frac{7}{8}\) × 4 = 3 \(\frac{1}{2}\)

Question 6.
4 × \(\frac{3}{5}\) = ______ \(\frac{□}{□}\)

Answer: 2 \(\frac{2}{5}\)

Explanation:
4 × \(\frac{3}{5}\)
Place four whole fractions strips side by side.
Place three \(\frac{1}{5}\) fraction strips all with the same denominator that fits exactly under the two whole numbers.
4 of \(\frac{3}{5}\)
4 × \(\frac{3}{5}\) = \(\frac{12}{5}\)
The mixed fraction of \(\frac{12}{5}\) is 2 \(\frac{2}{5}\)

Question 7.
\(\frac{7}{8}\) × 2 = ______ \(\frac{□}{□}\)

Answer: 1 \(\frac{3}{4}\)

Explanation:
\(\frac{7}{8}\) × 2
Place two whole fractions strips side by side.
Place seven \(\frac{1}{8}\) fraction strips all with the same denominator that fits exactly under the two whole numbers.
\(\frac{7}{8}\) of 2
\(\frac{7}{8}\) × 2 = \(\frac{7}{4}\)
The mixed fraction of \(\frac{7}{4}\) is 1 \(\frac{3}{4}\)

Question 8.
7 × \(\frac{2}{5}\) = ______ \(\frac{□}{□}\)

Answer: 2 \(\frac{4}{5}\)

Explanation:
7 × \(\frac{2}{5}\)
Place seven whole fractions strips side by side.
Place two \(\frac{1}{5}\) fraction strips all with the same denominator that fits exactly under the two whole numbers.
7 × \(\frac{2}{5}\) = \(\frac{14}{5}\)
The mixed fraction of \(\frac{14}{5}\) = 2 \(\frac{4}{5}\)

Question 9.
\(\frac{3}{8}\) × 4 = ______

Answer: \(\frac{3}{2}\)

Explanation:
\(\frac{3}{8}\) × 4
Place four whole fractions strips side by side.
Place three \(\frac{1}{8}\) fraction strips all with the same denominator that fits exactly under the two whole numbers.

Question 10.
11 × \(\frac{3}{4}\) = ______ \(\frac{□}{□}\)

Answer: 8 \(\frac{1}{4}\)

Explanation:
11 × \(\frac{3}{4}\)
Place Eleven whole fractions strips side by side.
Place three \(\frac{1}{4}\) fraction strips all with the same denominator that fits exactly under the two whole numbers.
11 of \(\frac{3}{4}\)
11 × \(\frac{3}{4}\) = \(\frac{33}{4}\)
Convert the improper fraction to the mixed fraction.
\(\frac{33}{4}\) = 8 \(\frac{1}{4}\)
11 × \(\frac{3}{4}\) = 8 \(\frac{1}{4}\)

Question 11.
\(\frac{4}{15}\) × 5 = ______ \(\frac{□}{□}\)

Answer: 5 \(\frac{1}{3}\)

Explanation:
\(\frac{4}{15}\) × 5
Place five whole fractions strips side by side.
Place four \(\frac{1}{15}\) fraction strips all with the same denominator that fits exactly under the two whole numbers.
\(\frac{4}{15}\) of 5
\(\frac{4}{15}\) × 5 = \(\frac{4}{3}\)
Convert the improper fraction to the mixed fraction.
\(\frac{4}{3}\) = 5 \(\frac{1}{3}\)

Question 12.
Matt has a 5-pound bag of apples. To make a pie, he needs to use \(\frac{3}{5}\) of the bag. How many pounds of apples will he use for the pie? Explain what a model for this problem might look like.
______ pound(s)

Answer: 3 pounds

Explanation:
Given, Matt has a 5-pound bag of apples.
To make a pie, he needs to use \(\frac{3}{5}\) of the bag.
\(\frac{3}{5}\) × 5 = 3
Therefore Matt used 3 pounds of apples to make a pie.

Problem Solving – Page No. 298

Pose a Problem

Question 13.
Tarique drew the model below for a problem. Write 2 problems that can be solved using this model. One of your problems should involve multiplying a whole number by a fraction and the other problem should involve multiplying a fraction by a whole number.
Go Math Grade 5 Answer Key Chapter 7 Multiply Fractions img 6
Pose problems.                                       Solve your problems.
How could you change the model to give you an answer of 4 \(\frac{4}{5}\)?
Explain and write a new equation.
Type below:
_________

Answer:
The five children in the Smith family each spend 2/5 of an hour doing household chores on Saturday. How much time did the spend altogether on their chores?
Multiply the numerator with the whole number.
5 × \(\frac{2}{5}\) = \(\frac{10}{5}\) = 2

Share and Show – Page No. 301

Find the product. Write the product in simplest form.

Question 1.
3 × \(\frac{2}{5}\) =
• Multiply the numerator by the whole number. Write the product over the denominator.
Go Math Grade 5 Answer Key Chapter 7 Multiply Fractions img 7
• Write the answer as a mixed number in simplest form.
Go Math Grade 5 Answer Key Chapter 7 Multiply Fractions img 8
______ \(\frac{□}{□}\)

Answer: 1 \(\frac{1}{5}\)

Explanation:
Multiply the whole number with the numerator.
3 \(\frac{2}{5}\) = \(\frac{6}{5}\)
Now write the improper fraction in the form of the mixed fraction.
\(\frac{6}{5}\) = 1 \(\frac{1}{5}\)

Question 2.
\(\frac{2}{3}\) × 5 = ______ \(\frac{□}{□}\)

Answer: 3 \(\frac{1}{3}\)

Explanation:
Multiply the whole number with the numerator.
\(\frac{2}{3}\) × 5 = \(\frac{10}{3}\)
Now write the improper fraction in the form of the mixed fraction.
\(\frac{10}{3}\) = 3 \(\frac{1}{3}\)

Question 3.
6 × \(\frac{2}{3}\) = ______

Answer: 4

Explanation:
6 × \(\frac{2}{3}\)
Multiply the whole number with the numerator.
6 × \(\frac{2}{3}\) = \(\frac{12}{3}\)
Now write the improper fraction in the form of the mixed fraction.
\(\frac{12}{3}\) = 4

Question 4.
\(\frac{5}{7}\) × 4 = ______ \(\frac{□}{□}\)

Answer: 2 \(\frac{6}{7}\)

Explanation:
\(\frac{5}{7}\) × 4
Multiply the whole number with the numerator.
\(\frac{5}{7}\) × 4 = \(\frac{20}{7}\)
Now write the improper fraction in the form of the mixed fraction.
2 \(\frac{6}{7}\)
Thus, \(\frac{5}{7}\) × 4 = 2 \(\frac{6}{7}\)

On Your Own

Find the product. Write the product in simplest form.

Question 5.
5 × \(\frac{2}{3}\) = ______ \(\frac{□}{□}\)

Answer: 3 \(\frac{1}{3}\)

Explanation:
5 × \(\frac{2}{3}\)
Multiply the whole number with the numerator.
5 × \(\frac{2}{3}\) = \(\frac{10}{3}\)
Now write the improper fraction in the form of the mixed fraction.
3 \(\frac{1}{3}\)
5 × \(\frac{2}{3}\) = 3 \(\frac{1}{3}\)

Question 6.
\(\frac{1}{4}\) × 3 = ______ \(\frac{□}{□}\)

Answer: \(\frac{3}{4}\)

Explanation:
\(\frac{1}{4}\) × 3
Multiply the whole number with the numerator.
\(\frac{1}{4}\) × 3 = \(\frac{3}{4}\)

Question 7.
7 × \(\frac{7}{8}\) = ______ \(\frac{□}{□}\)

Answer: 6 \(\frac{1}{8}\)

Explanation:
7 × \(\frac{7}{8}\)
Multiply the whole number with the numerator.
\(\frac{49}{8}\)
Now write the improper fraction in the form of the mixed fraction.
\(\frac{49}{8}\) = 6 \(\frac{1}{8}\)
Thus, 7 × \(\frac{7}{8}\) = 6 \(\frac{1}{8}\)

Question 8.
2 × \(\frac{4}{5}\) = ______ \(\frac{□}{□}\)

Answer: 1 \(\frac{3}{5}\)

Explanation:
2 × \(\frac{4}{5}\)
Multiply the whole number with the numerator.
2 × \(\frac{4}{5}\) = \(\frac{8}{5}\)
Now write the improper fraction in the form of the mixed fraction.
\(\frac{8}{5}\) = 1 \(\frac{3}{5}\)

Question 9.
4 × \(\frac{3}{4}\) = ______

Answer: 3

Explanation:
Multiply the whole number with the numerator.
4 × \(\frac{3}{4}\) = \(\frac{12}{4}\)
4 divides 12 three times.
So, \(\frac{12}{4}\) = 3
4 × \(\frac{3}{4}\) = 3

Question 10.
\(\frac{7}{9}\) × 2 = ______ \(\frac{□}{□}\)

Answer: 1 \(\frac{5}{9}\)

Explanation:
\(\frac{7}{9}\) × 2
Multiply the whole number with the numerator.
\(\frac{7}{9}\) × 2 = \(\frac{14}{9}\)
Now write the improper fraction in the form of the mixed fraction.
\(\frac{14}{9}\) = 1 \(\frac{5}{9}\)

Practice: Copy and Solve. Find the product. Write the product in simplest form.

Question 11.
\(\frac{3}{5}\) × 11 = ______ \(\frac{□}{□}\)

Answer: 6 \(\frac{3}{5}\)

Explanation:
\(\frac{3}{5}\) × 11
Multiply the whole number with the numerator.
\(\frac{3}{5}\) × 11 = \(\frac{33}{5}\)
Now write the improper fraction in the form of the mixed fraction.
\(\frac{33}{5}\) = 6 \(\frac{3}{5}\)

Question 12.
3 × \(\frac{3}{4}\) = ______ \(\frac{□}{□}\)

Answer: 2 \(\frac{1}{4}\)

Explanation:
3 × \(\frac{3}{4}\)
Multiply the whole number with the numerator.
3 × \(\frac{3}{4}\) = \(\frac{9}{4}\)
Now write the improper fraction in the form of the mixed fraction.
\(\frac{9}{4}\) = 2 \(\frac{1}{4}\)

Question 13.
\(\frac{5}{8}\) × 3 = ______ \(\frac{□}{□}\)

Answer: 1 \(\frac{7}{8}\)

Explanation:
\(\frac{5}{8}\) × 3
Multiply the whole number with the numerator.
\(\frac{5}{8}\) × 3 = \(\frac{15}{8}\)
Now write the improper fraction in the form of the mixed fraction.
\(\frac{15}{8}\) = 1 \(\frac{7}{8}\)

Algebra Find the unknown digit.

Question 14.
\(\frac{■}{2}\) × 8 = 4
■ = ______

Answer: 1

Explanation:
\(\frac{■}{2}\) × 8 = 4
\(\frac{■}{2}\) = 4/8
■ = 4 × 2/8 = 1
■ = 1

Question 15.
■ × \(\frac{5}{6}\) = \(\frac{20}{6}\) or 3 \(\frac{1}{3}\)
■ = ______

Answer: 4

Explanation:
■ × \(\frac{5}{6}\) = \(\frac{20}{6}\)
■ = 20/6 × 6/5
■ = 20/5 = 4
■ = 4

Question 16.
\(\frac{1}{■}\) × 18 = 3
■ = ______

Answer: 6

Explanation:
\(\frac{1}{■}\) × 18 = 3
\(\frac{1}{3}\) × 18 = ■
■ = 18/3 = 6
■ = 6

UNLOCK the Problem – Page No. 302

Go Math Grade 5 Answer Key Chapter 7 Multiply Fractions img 9

Question 17.
The caterer wants to have enough turkey to feed 24 people. If he wants to provide \(\frac{3}{4}\) of a pound of turkey for each person, how much turkey does he need?
a. What do you need to find?
Type below:
__________

Answer: I need to find How much turkey the caterer needs to provide for each person.

Question 17.
b. What operation will you use?
Type below:
__________

Answer: I will use the multiplication operation to solve the problem.

Question 17.
c. What information are you given?
Type below:
__________

I am given the information about the number of people to feed and the fraction of pounds of turkey each person gets.

Question 17.
d. Solve the problem.
Type below:
__________

Answer:
The caterer wants to serve 24 people
\(\frac{3}{4}\) × 24
4 divides 24 six times.
3 × 6 = 18
Thus the caterer needs 18 pounds of Turkey.

Question 17.
e. Complete the sentences.
The caterer wants to serve 24 people _____ of a pound of turkey each.
He will need ____ × ____ , or ______ pounds of turkey.
Type below:
__________

Answer: \(\frac{3}{4}\) × 24

Question 17.
f. Fill in the bubble for the correct answer choice.
Options:
a. 72 pounds
b. 24 pounds
c. 18 pounds
d. 6 pounds

Answer: 18 pounds

Explanation:
The caterer wants to serve 24 people
\(\frac{3}{4}\) × 24
4 divides 24 six times.
3 × 6 = 18
The correct answer is option C.

Question 18.
Patty wants to run \(\frac{5}{6}\) of a mile every day for 5 days. How far will she run in that time?
Options:
a. 25 miles
b. 5 miles
c. 4 \(\frac{1}{6}\) miles
d. 1 \(\frac{2}{3}\) miles

Answer: 4 \(\frac{1}{6}\) miles

Explanation:
Patty wants to run \(\frac{5}{6}\) of a mile every day for 5 days.
\(\frac{5}{6}\) × 5 = \(\frac{25}{6}\)
Convert the improper fraction to the mixed fraction.
\(\frac{25}{6}\) = 4 \(\frac{1}{6}\) miles
Thus the correct answer is option C.

Question 19.
Doug has 33 feet of rope. He wants to use \(\frac{2}{3}\) of it for his canoe. How many feet of rope will he use for his canoe?
Options:
a. 11 feet
b. 22 feet
c. 33 feet
d. 66 feet

Answer: 22 feet

Explanation:
Doug has 33 feet of rope. He wants to use \(\frac{2}{3}\) of it for his canoe.
\(\frac{2}{3}\) × 33 feet
3 divides 33 eleven times.
2 × 11 = 22 feet
The correct answer is option B.

Share and Show – Page No. 304

Use the model to find the product.

Question 1.
Go Math Grade 5 Answer Key Chapter 7 Multiply Fractions img 10
\(\frac{3}{5} \times \frac{1}{3}=\)
\(\frac{□}{□}\)

Answer: \(\frac{1}{5}\)

Explanation:
The fraction \(\frac{3}{5}\) represents the rows and columns.
The fraction \(\frac{1}{3}\) indicates the shaded part of the figure.
\(\frac{3}{5}\) × \(\frac{1}{3}\) = \(\frac{1}{5}\)

Question 2.
Go Math Grade 5 Answer Key Chapter 7 Multiply Fractions img 11
\(\frac{2}{3} \times \frac{3}{5}=\)
\(\frac{□}{□}\)

Answer: \(\frac{2}{5}\)

Explanation:

The above figure shows that the circle is divided into 5 parts in which 2 parts are non shaded and 3 parts are shaded.
So, the fraction of the circle is \(\frac{2}{3}\)
The fraction for the shaded part of the circle is \(\frac{3}{5}\)
\(\frac{2}{3}\) × \(\frac{3}{5}\) = \(\frac{2}{5}\)

Share and Show – Page No. 305

Find the product. Draw a model.

Question 3.
\(\frac{2}{3} \times \frac{1}{5}=\) \(\frac{□}{□}\)

Answer: \(\frac{2}{15}\)

Explanation:
\(\frac{2}{3}\) × \(\frac{1}{5}\)
Multiply the denominators of both the fractions.
\(\frac{2}{15}\)
\(\frac{2}{3} \times \frac{1}{5}=\) \(\frac{2}{15}\)

Question 4.
\(\frac{1}{2} \times \frac{5}{6}=\) \(\frac{□}{□}\)

Answer: \(\frac{5}{12}\)

Explanation:
\(\frac{1}{2}\) × \(\frac{5}{6}\)
Multiply the numerators and the denominators.
\(\frac{1}{2}\) × \(\frac{5}{6}\) = \(\frac{5}{12}\)
\(\frac{1}{2} \times \frac{5}{6}=\) \(\frac{5}{12}\)

Question 5.
\(\frac{3}{5} \times \frac{1}{3}=\) \(\frac{□}{□}\)

Answer: \(\frac{1}{5}\)

Explanation:
\(\frac{3}{5}\) × \(\frac{1}{3}\)
Multiply the denominators and the numerators of the fractions.
\(\frac{3}{5}\) × \(\frac{1}{3}\) = \(\frac{3}{15}\)
\(\frac{3}{15}\) = \(\frac{1}{5}\)
\(\frac{3}{5} \times \frac{1}{3}=\) \(\frac{1}{5}\)

Question 6.
\(\frac{3}{4} \times \frac{1}{6}=\) \(\frac{□}{□}\)

Answer: \(\frac{1}{8}\)

Explanation:
\(\frac{3}{4}\) × \(\frac{1}{6}\)
Multiply the denominators and the numerators of the fractions.
\(\frac{3}{4}\) × \(\frac{1}{6}\) = \(\frac{3}{24}\)
3 divides 24 eight times.
So, \(\frac{3}{24}\) = \(\frac{1}{8}\)
Thus, \(\frac{3}{4} \times \frac{1}{6}=\) \(\frac{1}{8}\)

Question 7.
\(\frac{2}{5} \times \frac{5}{6}=\) \(\frac{□}{□}\)

Answer: \(\frac{1}{3}\)

Explanation:
\(\frac{2}{5}\) × \(\frac{5}{6}\)
Multiply the denominators and the numerators of the fractions.
\(\frac{2}{5}\) × \(\frac{5}{6}\) = \(\frac{10}{30}\)
10 divides 30 three times.
\(\frac{10}{30}\) = \(\frac{1}{3}\)
\(\frac{2}{5} \times \frac{5}{6}=\) \(\frac{1}{3}\)

Question 8.
\(\frac{5}{6} \times \frac{3}{5}=\) \(\frac{□}{□}\)

Answer: \(\frac{1}{2}\)

Explanation:
\(\frac{5}{6}\) × \(\frac{3}{5}\)
Multiply the denominators and the numerators of the fractions.
\(\frac{5}{6}\) × \(\frac{3}{5}\) = \(\frac{15}{30}\)
\(\frac{5}{6}\) × \(\frac{3}{5}\) = \(\frac{15}{30}\) = \(\frac{1}{2}\)
\(\frac{5}{6} \times \frac{3}{5}=\) \(\frac{1}{2}\)

Problem Solving – Page No. 306

What’s the Error?
Go Math Grade 5 Answer Key Chapter 7 Multiply Fractions img 12

Question 9.
Cheryl and Marcus are going to make a two-tiered cake. The smaller tier is \(\frac{2}{3}\) the size of the larger tier. The recipe for the bottom tier calls for \(\frac{3}{5}\) cup of water. How much water will they need to make the smaller tier?

They made a model to represent the problem. Cheryl says they need \(\frac{6}{9}\) cup of water. Marcus says they need \(\frac{2}{5}\) cup water. Who is correct? Explain.
Go Math Grade 5 Answer Key Chapter 7 Multiply Fractions img 13
Cheryl’s answer               Marcus’ answer
Type below:
_________

Answer: Marcus’ answer is correct.

Explanation:
Cheryl and Marcus are going to make a two-tiered cake.
The smaller tier is \(\frac{2}{3}\) the size of the larger tier.
The recipe for the bottom tier calls for \(\frac{3}{5}\) cup of water.
\(\frac{3}{5}\) × \(\frac{2}{3}\) = \(\frac{2}{5}\)

Share and Show – Page No. 309

Complete the statement with equal to, greater than, or less than.

Question 1.
4 × \(\frac{7}{8}\) will be ___________ \(\frac{7}{8}\)
Go Math Grade 5 Answer Key Chapter 7 Multiply Fractions img 14
_________

Answer: Greater than

Explanation:
4 × \(\frac{7}{8}\) = \(\frac{7}{2}\)
The denominator with a greater number will be the smallest number.
So, \(\frac{7}{2}\) is greater than \(\frac{7}{8}\)

Question 2.
\(\frac{3}{5} \times \frac{2}{7}\) will be ___________ \(\frac{3}{5}\)

Answer: Less than

Explanation:
\(\frac{3}{5}\) × \(\frac{2}{7}\) = \(\frac{6}{35}\)
The denominator with the greatest number will be the smallest fraction.
So, \(\frac{6}{35}\) is less than \(\frac{3}{5}\)

Question 3.
\(\frac{5}{8} \times 6\) will be ___________ \(\frac{5}{8}\)

Answer: Greater than

Explanation:
\(\frac{5}{8}\) × 6 = \(\frac{15}{4}\)
\(\frac{15}{4}\) = 3 \(\frac{3}{4}\)
3 \(\frac{3}{4}\) is greater than \(\frac{5}{8}\)

Question 4.
\(\frac{2}{3} \times \frac{5}{5}\) will be ___________ \(\frac{2}{3}\)

Answer: Equal to

Explanation:
\(\frac{2}{3}\) × \(\frac{5}{5}\) = \(\frac{2}{3}\)
\(\frac{2}{3}\) is equal to \(\frac{2}{3}\)

Question 5.
\(8 \times \frac{7}{8}\) will be ___________ 8

Answer: Less than

Explanation:
8 × \(\frac{7}{8}\)= 7
7 is less than 8.
\(8 \times \frac{7}{8}\) will be less than 8.

On Your Own

Complete the statement with equal to, greater than, or less than.

Question 6.
\(\frac{4}{9} \times \frac{3}{8}\) will be ___________ \(\frac{3}{8}\)

Answer: Less than

Explanation:
\(\frac{4}{9}\) × \(\frac{3}{8}\) = \(\frac{12}{72}\)
= \(\frac{1}{6}\)
\(\frac{1}{6}\) is less than \(\frac{3}{8}\)
\(\frac{4}{9} \times \frac{3}{8}\) will be less than \(\frac{3}{8}\)

Question 7.
\(7 \times \frac{9}{10}\) will be ___________ \(\frac{9}{10}\)

Answer: Greater than

Explanation:
7 × \(\frac{9}{10}\) = \(\frac{63}{10}\)
Denominators are same so compare the numerators.
\(\frac{63}{10}\) is greater than \(\frac{9}{10}\)

Question 8.
\(5 \times \frac{1}{3}\) will be ___________ \(\frac{1}{3}\)

Answer: Greater than

Explanation:
5 × \(\frac{1}{3}\) = \(\frac{5}{3}\)
Denominators are same so compare the numerators.
\(\frac{5}{3}\) is greater than \(\frac{1}{3}\)

Question 9.
\(\frac{6}{11} \times 1\) will be ___________ \(\frac{6}{11}\)

Answer: Equal to

Explanation:
\(\frac{6}{11}\) × 1 = \(\frac{6}{11}\)
\(\frac{6}{11}\) is equal to \(\frac{6}{11}\).

Question 10.
\(\frac{1}{6} \times \frac{7}{7}\) will be ___________ 1

Answer: Less than

Explanation:
\(\frac{1}{6}\) × \(\frac{7}{7}\) = \(\frac{1}{6}\)
\(\frac{1}{6}\) is less than 1

Question 11.
\(4 \times \frac{3}{5}\) will be ___________ \(\frac{3}{5}\)

Answer: Greater than

Explanation:
4 × \(\frac{3}{5}\) = \(\frac{12}{5}\)
Denominators are same so compare the numerators.
\(\frac{12}{5}\) is greater than \(\frac{3}{5}\)

Problem Solving – Page No. 310

Question 12.
Lola is making cookies. She plans to multiply the recipe by 3 so she can make enough cookies for the whole class. If the recipe calls for \(\frac{2}{3}\) cup of sugar, will she need more than \(\frac{2}{3}\) or less than \(\frac{2}{3}\) cup of sugar to make all the cookies?
_________ \(\frac{2}{3}\) cup of sugar

Answer: More than

Explanation:
ola is making cookies. She plans to multiply the recipe by 3 so she can make enough cookies for the whole class.
3 × \(\frac{2}{3}\) = 2
So, Lola needs more than \(\frac{2}{3}\) cup of sugar.

Question 13.
Peter is planning on spending \(\frac{2}{3}\) as many hours watching television this week as he did last week. Is Peter going to spend more hours or fewer hours watching television this week?
_________ hours

Answer: Fewer

Explanation:
Peter is planning on spending \(\frac{2}{3}\) as many hours watching television this week as he did last week.
7 × \(\frac{2}{3}\) = \(\frac{14}{3}\)
\(\frac{14}{3}\) = 4 \(\frac{2}{3}\)
Thus peter going to spend more hours or fewer hours watching television this week.

Question 14.
Test Prep Rochelle saves \(\frac{1}{4}\) of her allowance. If she decides to start saving \(\frac{1}{2}\) as much, which statement below is true?
Options:
a. She will be saving the same amount.
b. She will be saving more.
c. She will be saving less.
d. She will be saving twice as much.

Answer: She will be saving more

Explanation:
Test Prep Rochelle saves \(\frac{1}{4}\) of her allowance.
\(\frac{1}{4}\) is greater than \(\frac{1}{2}\)
So, the answer is option B.

Connect to Art

A scale model is a representation of an object with the same shape as the real object. Models can be larger or smaller than the actual object but are often smaller.

Architects often make scale models of the buildings or structures they plan to build. Models can give them an idea of how the structure will look when finished. Each measurement of the building is scaled up or down by the same factor.
Go Math Grade 5 Answer Key Chapter 7 Multiply Fractions img 15

Bob is building a scale model of his bike. He wants his model to be \(\frac{1}{5}\) as long as his bike.

Question 15.
If Bob’s bike is 60 inches long, how long will his model be?
_____ in.

Answer: 12 inches

Explanation:
Given that, Bob is building a scale model of his bike. He wants his model to be \(\frac{1}{5}\) as long as his bike.
If Bob’s bike is 60 inches long then multiply with the fraction \(\frac{1}{5}\)
\(\frac{1}{5}\) × 60 = 12 inches
The model will be 12 inches long.

Question 16.
If one wheel on Bob’s model is 4 inches across, how many inches across is the actual wheel on his bike? Explain.
\(\frac{□}{□}\) in.

Answer: \(\frac{4}{5}\) in.

Explanation:
Given that, one wheel on Bob’s model is 4 inches across.
4 × \(\frac{1}{5}\) = \(\frac{4}{5}\) in.

Share and Show – Page No. 313

Find the product. Write the product in simplest form.

Question 1.
\(6 \times \frac{3}{8}\)
\(\frac{6}{1} \times \frac{3}{8}\) = \(\frac{■}{■}\)
______ \(\frac{□}{□}\)

Answer: 2 \(\frac{1}{4}\)

Explanation:
\(\frac{6}{1} \times \frac{3}{8}\) = \(\frac{■}{■}\)
6 × \(\frac{3}{8}\) = \(\frac{18}{8}\) = \(\frac{9}{4}\)
\(\frac{9}{4}\) = 2 \(\frac{1}{4}\)
2 \(\frac{1}{4}\) = \(\frac{■}{■}\)
\(\frac{■}{■}\) = 2 \(\frac{1}{4}\)

Question 2.
\(\frac{3}{8} \times \frac{8}{9}\) = \(\frac{□}{□}\)

Answer: \(\frac{1}{3}\)

Explanation:
\(\frac{3}{8} \times \frac{8}{9}\) = \(\frac{□}{□}\)
\(\frac{3}{8}\) × \(\frac{8}{9}\) = \(\frac{1}{3}\)
Thus, \(\frac{3}{8} \times \frac{8}{9}\) = \(\frac{1}{3}\)

Question 3.
\(\frac{2}{3} \times 27\) = ______

Answer: 18

Explanation:
27 × \(\frac{2}{3}\)
3 divides 27 nine times.
Thus, 27 × \(\frac{2}{3}\) = 18

Question 4.
\(\frac{5}{12} \times \frac{3}{5}\) = \(\frac{□}{□}\)

Answer: \(\frac{1}{4}\)

Explanation:
\(\frac{5}{12}\) × \(\frac{3}{5}\) = \(\frac{3}{12}\)
3 divides 12 four times.
\(\frac{3}{12}\) = \(\frac{1}{4}\)
\(\frac{5}{12} \times \frac{3}{5}\) = \(\frac{1}{4}\)

Question 5.
\(\frac{1}{2} \times \frac{3}{5}\) = \(\frac{□}{□}\)

Answer: \(\frac{3}{10}\)

Explanation:
\(\frac{1}{2}\) × \(\frac{3}{5}\)
Multiply the numerators and the denominators.
\(\frac{1}{2}\) × \(\frac{3}{5}\)  = \(\frac{3}{10}\)
\(\frac{1}{2} \times \frac{3}{5}\) = \(\frac{3}{10}\)

Question 6.
\(\frac{2}{3} \times \frac{4}{5}\) = \(\frac{□}{□}\)

Answer: \(\frac{8}{15}\)

Explanation:
\(\frac{2}{3}\) × \(\frac{4}{5}\)
Multiply the numerators and the denominators.
\(\frac{2}{3}\) × \(\frac{4}{5}\) = \(\frac{8}{15}\)

Question 7.
\(\frac{1}{3} \times \frac{5}{8}\) = \(\frac{□}{□}\)

Answer: \(\frac{5}{24}\)

Explanation:
\(\frac{1}{3}\) × \(\frac{5}{8}\)
Multiply the numerators and the denominators.
\(\frac{1}{3} \times \frac{5}{8}\) = \(\frac{5}{24}\)

Question 8.
\(4 \times \frac{1}{5}\) = \(\frac{□}{□}\)

Answer: \(\frac{4}{5}\)

Explanation:
Multiply the numerator with the whole number.
4 × \(\frac{1}{5}\) = \(\frac{4}{5}\)
\(4 \times \frac{1}{5}\) = \(\frac{4}{5}\)

On Your Own

Find the product. Write the product in simplest form.

Question 9.
\(2 \times \frac{1}{8}\) = \(\frac{□}{□}\)

Answer: \(\frac{1}{4}\)

Explanation:
Multiply the whole number with the numerator.
2 × \(\frac{1}{8}\)
2 divides 8 four times.
2 × \(\frac{1}{8}\) = \(\frac{1}{4}\)
\(2 \times \frac{1}{8}\) = \(\frac{1}{4}\)

Question 10.
\(\frac{4}{9} \times \frac{4}{5}\) = \(\frac{□}{□}\)

Answer: \(\frac{16}{45}\)

Explanation:
\(\frac{4}{9}\) × \(\frac{4}{5}\)
Multiply the numerators and the denominators.
\(\frac{4}{9}\) × \(\frac{4}{5}\) = \(\frac{16}{45}\)
\(\frac{4}{9} \times \frac{4}{5}\) = \(\frac{16}{45}\)

Question 11.
\(\frac{1}{12} \times \frac{2}{3}\) = \(\frac{□}{□}\)

Answer: \(\frac{1}{18}\)

Explanation:
\(\frac{1}{12}\) × \(\frac{2}{3}\)
Multiply the numerators and the denominators.
\(\frac{1}{12}\) × \(\frac{2}{3}\) = \(\frac{2}{36}\)
\(\frac{2}{36}\) = \(\frac{1}{18}\)
\(\frac{1}{12} \times \frac{2}{3}\) = \(\frac{1}{18}\)

Question 12.
\(\frac{1}{7} \times 30\) = _____ \(\frac{□}{□}\)

Answer: 4 \(\frac{2}{7}\)

Explanation:
30 × \(\frac{1}{7}\) = \(\frac{30}{7}\)
Convert improper fraction to the mixed fraction.
\(\frac{30}{7}\) = 4 \(\frac{2}{7}\)
\(\frac{1}{7} \times 30\) = 4 \(\frac{2}{7}\)

Question 13.
Of the pets in the pet show, \(\frac{5}{6}\) are cats. \(\frac{4}{5}\) of the cats are calico cats. What fraction of the pets are calico cats?
\(\frac{□}{□}\) calico cats

Answer: \(\frac{2}{3}\)

Explanation:
Of the pets in the pet show, \(\frac{5}{6}\) are cats. \(\frac{4}{5}\) of the cats are calico cats.
\(\frac{5}{6}\) × \(\frac{4}{5}\) = \(\frac{20}{30}\) = \(\frac{2}{3}\)
\(\frac{2}{3}\) fraction of the pets are calico cats.

Question 14.
Five cats each ate \(\frac{1}{4}\) cup of food. How much food did they eat altogether?
_____ \(\frac{□}{□}\) cups of food

Answer: 1 \(\frac{1}{4}\)

Explanation:
Five cats each ate \(\frac{1}{4}\) cup of food.
5 × \(\frac{1}{4}\) = \(\frac{5}{4}\)
The mixed fraction of \(\frac{5}{4}\) is 1 \(\frac{1}{4}\)

Algebra Evaluate for the given value.

Question 15.
\(\frac{2}{5}\) × c for c = \(\frac{4}{7}\)
\(\frac{□}{□}\)

Answer: \(\frac{8}{35}\)

Explanation:
\(\frac{2}{5}\) × c = \(\frac{4}{7}\)
c = \(\frac{4}{7}\) × \(\frac{2}{5}\)
c = \(\frac{8}{35}\)

Question 16.
m × \(\frac{4}{5}\) for m = \(\frac{7}{8}\)
\(\frac{□}{□}\)

Answer: \(\frac{7}{10}\)

Explanation:
m = \(\frac{4}{5}\) × \(\frac{7}{8}\)
Multiply the numerators and denominators.
\(\frac{4}{5}\) × \(\frac{7}{8}\) = \(\frac{7}{10}\)

Question 17.
\(\frac{2}{3}\) × t for t = \(\frac{1}{8}\)
\(\frac{□}{□}\)

Answer: \(\frac{1}{12}\)

Explanation:
\(\frac{2}{3}\) × t for t = \(\frac{1}{8}\)
t = \(\frac{1}{8}\) × \(\frac{2}{3}\)
t = \(\frac{1}{12}\)

Question 18.
y × \(\frac{2}{3}\) for y = 5
_______

Answer: 4

Explanation:
y × \(\frac{2}{3}\) for y = 5
6 × \(\frac{2}{3}\) = 4

Problem Solving – Page No. 314

Speedskating is a popular sport in the Winter Olympics. Many young athletes in the U.S. participate in speedskating clubs and camps.
Go Math Grade 5 Answer Key Chapter 7 Multiply Fractions img 16

Question 19.
At a camp in Green Bay, Wisconsin, \(\frac{7}{9}\) of the participants were from Wisconsin. Of that group, \(\frac{3}{5}\) were 12 years old. What fraction of the group was from Wisconsin and 12 years old?
\(\frac{□}{□}\)

Answer: \(\frac{7}{15}\)

Explanation:
Given that,
At a camp in Green Bay, Wisconsin, \(\frac{7}{9}\) of the participants were from Wisconsin.
Of that group, \(\frac{3}{5}\) were 12 years old.
To find the fraction of the group was from Wisconsin and 12 years old
We have to multiply the fraction \(\frac{7}{9}\) and \(\frac{3}{5}\)
\(\frac{7}{9}\) × \(\frac{3}{5}\) = \(\frac{21}{45}\)
\(\frac{21}{45}\) = \(\frac{7}{15}\)
Thus the fraction of the group was from Wisconsin and 12 years old is \(\frac{7}{15}\).

Question 20.
Maribel wants to skate 1 \(\frac{1}{2}\) miles on Monday. If she skates \(\frac{9}{10}\) mile Monday morning and \(\frac{2}{3}\) of that distance Monday afternoon, will she reach her goal? Explain.
_____

Answer: Yes

Explanation:
Maribel wants to skate 1 \(\frac{1}{2}\) miles on Monday.
To find whether Maribel reached her goal we have to multiply the fractions \(\frac{9}{10}\) and \(\frac{2}{3}\)
\(\frac{9}{10}\) × \(\frac{2}{3}\) = \(\frac{3}{5}\)
By this we can say that Maribel reaches her goal.
So, the answer is yes.

Question 21.
On the first day of camp, \(\frac{5}{6}\) of the skaters were beginners. Of the beginners, \(\frac{1}{3}\) were girls. What fraction of the skaters were girls and beginners? Explain why your answer is reasonable.
\(\frac{□}{□}\)

Answer: \(\frac{5}{18}\)

Explanation:
On the first day of camp, \(\frac{5}{6}\) of the skaters were beginners. Of the beginners, \(\frac{1}{3}\) were girls.
Multiply the fraction of the skaters were beginning and the fraction of skaters were girls.
\(\frac{5}{6}\) × latex]\frac{1}{3}[/latex] = latex]\frac{5}{18}[/latex]
The fraction of the skaters were girls and beginners are latex]\frac{5}{18}[/latex]

Question 22.
Test Prep On Wednesday, Danielle skated \(\frac{2}{3}\) of the way around the track in 2 minutes. Her younger brother skated \(\frac{3}{4}\) of Danielle’s distance in 2 minutes. What fraction of the track did Danielle’s brother finish in 2 minutes?
Options:
a. \(\frac{1}{3}\)
b. \(\frac{1}{2}\)
c. \(\frac{5}{7}\)
d. \(\frac{3}{4}\)

Answer: \(\frac{1}{2}\)

Explanation:
Test Prep On Wednesday, Danielle skated \(\frac{2}{3}\) of the way around the track in 2 minutes.
Her younger brother skated \(\frac{3}{4}\) of Danielle’s distance in 2 minutes.
Multiply the fraction of Danielle skated and her younger brother skated.
\(\frac{2}{3}\) × \(\frac{3}{4}\) = \(\frac{1}{2}\)
Thus the correct answer is option B.

Mid-Chapter Checkpoint – Page No. 315

Concept and Skills

Question 1.
Explain how you would model 5 × \(\frac{2}{3}\)
Type below:
__________

Answer: \(\frac{10}{3}\)

Question 2.
When you multiply \(\frac{2}{3}\) by a fraction less than one, how does the product compare to the factors?
Type below:
__________

Answer: \(\frac{2}{3}\) × \(\frac{1}{2}\)
= \(\frac{1}{3}\)

Find the product. Write the product in simplest form.

Question 3.
\(\frac{2}{3} \times 6\)
______

Answer: 4

Explanation:
6 × \(\frac{2}{3}\)
Multiply the numerator with the whole numbers.
\(\frac{1}{3}\)

Question 4.
\(\frac{4}{5} \times 7\)
______ \(\frac{□}{□}\)

Answer: 5 \(\frac{3}{5}\)

Explanation:
Multiply the numerator with the whole numbers.
\(\frac{4}{5} \times 7\)
7 × \(\frac{4}{5}\) = \(\frac{28}{5}\)
Convert the improper fraction to the mixed fraction.
\(\frac{28}{5}\) = 5 \(\frac{3}{5}\)
\(\frac{4}{5} \times 7\) = 5 \(\frac{3}{5}\)

Question 5.
\(8 \times \frac{5}{7}\)
______ \(\frac{□}{□}\)

Answer: 5 \(\frac{5}{7}\)

Explanation:
8 × \(\frac{5}{7}\)
Multiply the numerator with the whole numbers.
8 × \(\frac{5}{7}\) = \(\frac{40}{7}\)
Convert the improper fraction to the mixed fraction.
\(\frac{40}{7}\) = 5 \(\frac{5}{7}\)

Question 6.
\(\frac{7}{8} \times \frac{3}{8}\)
\(\frac{□}{□}\)

Answer: \(\frac{21}{64}\)

Explanation:
\(\frac{7}{8}\) × \(\frac{3}{8}\)
Multiply the numerators and denominators of the fractions.
\(\frac{7}{8}\) × \(\frac{3}{8}\) = \(\frac{21}{64}\)

Question 7.
\(\frac{1}{2} \times \frac{3}{4}\)
\(\frac{□}{□}\)

Answer: \(\frac{3}{8}\)

Explanation:
Multiply the numerators and denominators of the fractions.
\(\frac{1}{2} \times \frac{3}{4}\)
\(\frac{1}{2}\) × \(\frac{3}{4}\) = \(\frac{3}{8}\)
\(\frac{1}{2} \times \frac{3}{4}\) = \(\frac{3}{8}\)

Question 8.
\(\frac{7}{8} \times \frac{4}{7}\)
\(\frac{□}{□}\)

Answer: \(\frac{1}{2}\)

Explanation:
Multiply the numerators and denominators of the fractions.
7 in the numerator and 7 in the denominator will be canceled.
4 divides 8 two times.
Thus the fraction is \(\frac{1}{2}\)
\(\frac{7}{8} \times \frac{4}{7}\) = \(\frac{1}{2}\)

Question 9.
\(2 \times \frac{3}{11}\)
\(\frac{□}{□}\)

Answer: \(\frac{6}{11}\)

Explanation:
Multiply the numerator with the whole numbers.
2 × \(\frac{3}{11}\)
2 × 3 = 6
2 × \(\frac{3}{11}\) = \(\frac{6}{11}\)
Thus, \(2 \times \frac{3}{11}\) = \(\frac{6}{11}\)

Question 10.
\(\frac{5}{8} \times \frac{2}{3}\)
\(\frac{□}{□}\)

Answer: \(\frac{5}{12}\)

Explanation:
\(\frac{5}{8} \times \frac{2}{3}\)
Multiply the numerators and denominators of the fractions.
\(\frac{5}{8}\) × \(\frac{2}{3}\) = \(\frac{10}{24}\)
\(\frac{10}{24}\) = \(\frac{5}{12}\)
\(\frac{5}{8} \times \frac{2}{3}\) = \(\frac{5}{12}\)

Question 11.
\(\frac{7}{12} \times 8\)
______ \(\frac{□}{□}\)

Answer: 4 \(\frac{2}{3}\)

Explanation:
8 × \(\frac{7}{12}\)
Multiply the numerator with the whole numbers.
8 × \(\frac{7}{12}\) = \(\frac{56}{12}\) = \(\frac{14}{3}\)
Convert the improper fraction to the mixed fraction.
\(\frac{14}{3}\) = 4 \(\frac{2}{3}\)

Complete the statement with equal to, greater than, or less than.

Question 12.
3 × \(\frac{2}{3}\) _________ 3

Answer: Less Than

Explanation:
3 × \(\frac{2}{3}\)
Multiply the numerator with the whole numbers.
3 in the denominator will be canceled.
3 × \(\frac{2}{3}\) = 2
2 is less than 3.
3 × \(\frac{2}{3}\) less than 3.

Question 13.
\(\frac{5}{7}\) × 3 _________ \(\frac{5}{7}\)

Answer: Greater than

Explanation:
\(\frac{5}{7}\) × 3
Multiply the numerator with the whole numbers.
\(\frac{5}{7}\) × 3 = \(\frac{15}{7}\)
Convert it into mixed fraction.
\(\frac{15}{7}\) = 2 \(\frac{1}{4}\)
2 \(\frac{1}{4}\) is greater than \(\frac{5}{7}\)

Mid-Chapter Checkpoint – Page No. 316

Question 14.
There is \(\frac{5}{6}\) of an apple pie left from dinner. Tomorrow, Victor plans to eat \(\frac{1}{6}\) of the pie that was left. How much of the whole pie will be left after he eats tomorrow?
\(\frac{□}{□}\) of the whole pie

Answer: \(\frac{25}{36}\) of the whole pie

Explanation:
Gīven that,
An apple pie left from the dinner is \(\frac{5}{6}\)
Victor plans to eat pie which was left is \(\frac{1}{6}\)
The whole pie will be left after Victor eats tomorrow =?
Pie left from dinner = \(\frac{5}{6}\)
Victor plans to eat pie which was left  = \(\frac{1}{6}\)
\(\frac{5}{6}\) × \(\frac{1}{6}\) = \(\frac{5}{36}\)
To find the whole pie will be left after he eats tomorrow:
\(\frac{5}{6}\) – \(\frac{5}{36}\)
LCD = 36
\(\frac{5}{6}\) × \(\frac{6}{6}\) – \(\frac{5}{36}\)
\(\frac{30}{36}\) – \(\frac{5}{36}\) = \(\frac{25}{36}\)
Therefore, whole pie left after Victor eats tomorrow is \(\frac{25}{36}\)

Question 15.
Everett and Marie are going to make fruit bars for their family reunion. They want to make 4 times the amount the recipe makes. If the recipe calls for \(\frac{2}{3}\) cup of oil, how much oil will they need?
______ \(\frac{□}{□}\) cup of oil

Answer: 2 \(\frac{2}{3}\)

Explanation:
Everett and Marie are going to make fruit bars for their family reunion.
They want to make 4 times the amount the recipe makes.
4 × \(\frac{2}{3}\) = \(\frac{8}{3}\)
The mixed fraction of \(\frac{8}{3}\) is 2 \(\frac{2}{3}\)
Thus Everett and Marie need Everett and Marie of oil.

Question 16.
Matt made the model below to help him solve his math problem. Write an expression that matches Matt’s model.
Go Math Grade 5 Answer Key Chapter 7 Multiply Fractions Mid-Chapter Checkpoint img 17
Type below:
__________

Answer: \(\frac{3}{4}\) × \(\frac{1}{3}\)

Explanation:
By seeing the above figure we can say that the fraction for the Matt’s model is \(\frac{3}{4}\) and \(\frac{2}{3}\).
Multiply the fractions \(\frac{3}{4}\) × \(\frac{2}{3}\) = \(\frac{1}{4}\)

Share and Show – Page No. 319

Use the grid to find the area. Let each square represent \(\frac{1}{3}\) meter by \(\frac{1}{3}\) meter.

Question 1.
1 \(\frac{2}{3}\) × 1 \(\frac{1}{3}\)
• Draw a diagram to represent the dimensions.
Go Math Grade 5 Answer Key Chapter 7 Multiply Fractions img 18
• How many squares cover the diagram?
• What is the area of each square?
• What is the area of the diagram?
______ \(\frac{□}{□}\)

Answer: 2 \(\frac{2}{9}\)

Explanation:
20 squares cover the diagram.
Each square represents \(\frac{1}{9}\) square meter
20 × \(\frac{1}{9}\) = \(\frac{20}{9}\)
Convert the fraction into the mixed fraction.
\(\frac{20}{9}\) = 2 \(\frac{2}{9}\)
Thus the area of the diagram is 2 \(\frac{2}{9}\)

Use the grid to find the area. Let each square represent \(\frac{1}{4}\) meter by \(\frac{1}{4}\) meter.

Question 2.
1 \(\frac{3}{4}\) × 1 \(\frac{2}{4}\)
Go Math Grade 5 Answer Key Chapter 7 Multiply Fractions img 19
______ \(\frac{□}{□}\)

Answer: 2 \(\frac{5}{8}\)

Explanation:
42 squares cover the diagram.
Each square represents \(\frac{1}{16}\) square meters.
42 × \(\frac{1}{16}\) = \(\frac{21}{8}\)
Convert the fraction into the mixed fraction.
\(\frac{21}{8}\) = 2 \(\frac{5}{8}\)
The area of the diagram is 2 \(\frac{5}{8}\) square meter.

Question 3.
1 \(\frac{1}{4}\) × 1 \(\frac{1}{2}\)
Go Math Grade 5 Answer Key Chapter 7 Multiply Fractions img 20
______ \(\frac{□}{□}\)

Answer: 1 \(\frac{7}{8}\)

Explanation:
30 squares cover the diagram.
Each square represents \(\frac{1}{16}\) square meters.
30 × \(\frac{1}{16}\) = \(\frac{15}{8}\)
Convert the fraction into the mixed fraction.
\(\frac{15}{8}\) = 1 \(\frac{7}{8}\)

Use an area model to solve.

Question 4.
1 \(\frac{3}{4}\) × 2 \(\frac{1}{2}\)
______ \(\frac{□}{□}\)

Answer: 4 \(\frac{3}{8}\)

Explanation:

54 squares covers the diagram.
Each square represents \(\frac{1}{16}\) square meters.
54 × \(\frac{1}{16}\) = \(\frac{27}{8}\)
Convert the fraction into the mixed fraction.
\(\frac{27}{8}\) = 4 \(\frac{3}{8}\)

Question 5.
1 \(\frac{3}{8}\) × 2 \(\frac{1}{2}\)
______ \(\frac{□}{□}\)

Answer: 3 \(\frac{7}{16}\)

Explanation:
55 squares cover the diagram.
Each square represents \(\frac{1}{16}\) square meters.
55 × \(\frac{1}{16}\) = \(\frac{55}{16}\)
Convert the fraction into the mixed fraction.
\(\frac{55}{16}\) = 3 \(\frac{7}{16}\)

Question 6.
1 \(\frac{1}{9}\) × 1 \(\frac{2}{3}\)
______ \(\frac{□}{□}\)

Answer: 1 \(\frac{23}{27}\)

Explanation:
130 squares the diagram.
Each square represents \(\frac{1}{16}\) square meters.
1 \(\frac{1}{9}\) × 1 \(\frac{2}{3}\)
\(\frac{10}{9}\) × \(\frac{5}{3}\) = \(\frac{50}{27}\)
Convert the fraction into the mixed fraction.
\(\frac{50}{27}\) = 1 \(\frac{23}{27}\)

Question 7.
Explain how finding the area of a rectangle with whole-number side lengths compares to finding the area of a rectangle with fractional side lengths.
Type below:
__________

Answer:
Go-Math-Grade-5-Answer-Key-Chapter-7-Multiply-Fractions-img-20
15 squares cover the diagram.
Each square is \(\frac{1}{16}\) square unit.
The area of the diagram is \(\frac{15}{16}\) square units.

Problem Solving – Page No. 320

Pose a Problem

Question 8.
Terrance is designing a garden. He drew the following diagram of his garden. Pose a problem using mixed numbers that can be solved using his diagram.
Go Math Grade 5 Answer Key Chapter 7 Multiply Fractions img 21
Go Math Grade 5 Answer Key Chapter 7 Multiply Fractions img 22
Pose a Problem.                 Solve your problem.
Describe how you decided on the dimensions of Terrance’s garden.
Type below:
__________

Answer:
how finding the area of a rectangle with mixed fractions side compares to finding the area of the rectangle with the fractional side lengths.
6 × 1 \(\frac{1}{8}\) = \(\frac{□}{□}\)
Let each square represent \(\frac{1}{2}\) meter by \(\frac{1}{2}\)
From the above figure, we can say that the number of squares is 27.
So, 27 squares cover the diagram.
Each square is \(\frac{1}{4}\) square unit.
27 × \(\frac{1}{4}\)  = \(\frac{27}{4}\)
Convert the fraction into the mixed fraction.
\(\frac{27}{4}\) = 6 \(\frac{3}{4}\)

Share and Show – Page No. 323

Complete the statement with equal to, greater than, or less than.

Question 1.
\(\frac{5}{6}\) × 2 \(\frac{1}{5}\) will be __________ 2 \(\frac{1}{5}\)
Go Math Grade 5 Answer Key Chapter 7 Multiply Fractions img 23 Shade the model to
show \(\frac{5}{6}\) × 2 \(\frac{1}{5}\) .
__________

Answer: Less than

Explanation:
\(\frac{5}{6}\) × 2 \(\frac{1}{5}\)
Convert the mixed fraction to the improper fraction.
2 \(\frac{1}{5}\) = \(\frac{11}{5}\)
\(\frac{5}{6}\) × \(\frac{11}{5}\) = \(\frac{55}{30}\)
1 \(\frac{25}{30}\) = 1 \(\frac{5}{6}\)
Thus 1 \(\frac{5}{6}\) is less than 2 \(\frac{1}{5}\)

Question 2.
1 \(\frac{1}{5}\) × 2 \(\frac{2}{3}\) will be __________ 2 \(\frac{2}{3}\)

Answer: Greater than

Explanation:
1 \(\frac{1}{5}\) × 2 \(\frac{2}{3}\)
First, Convert the mixed fraction to the improper fraction.
\(\frac{6}{5}\) × \(\frac{8}{3}\) = \(\frac{48}{15}\)
\(\frac{48}{15}\) = 3 \(\frac{3}{15}\)
3 \(\frac{3}{15}\) is greater than 2 \(\frac{2}{3}\)

Question 3.
\(\frac{4}{5}\) × 2 \(\frac{2}{5}\) will be __________ 2 \(\frac{2}{5}\)

Answer: Less than

Explanation:
\(\frac{4}{5}\) × \(\frac{12}{5}\)
First, Convert the mixed fraction to the improper fraction.
\(\frac{4}{5}\) × \(\frac{12}{5}\) = \(\frac{48}{5}\)
\(\frac{48}{5}\) = 9 \(\frac{3}{5}\)
9 \(\frac{3}{5}\) is less than 2 \(\frac{2}{5}\)

On Your Own

Complete the statement with equal to, greater than, or less than.

Question 4.
\(\frac{2}{2}\) × 1 \(\frac{1}{2}\) will be __________ 1 \(\frac{1}{2}\)

Answer: Equal to

Explanation:
\(\frac{2}{2}\) × \(\frac{3}{2}\) = \(\frac{6}{4}\)
\(\frac{6}{4}\) = 1 \(\frac{1}{2}\)
1 \(\frac{1}{2}\) is equal to 1 \(\frac{1}{2}\)
\(\frac{2}{2}\) × 1 \(\frac{1}{2}\) will be equal to 1 \(\frac{1}{2}\)

Question 5.
\(\frac{2}{3}\) × 3 \(\frac{1}{6}\) will be __________ 3 \(\frac{1}{6}\)

Answer: Less than

Explanation:
\(\frac{2}{3}\) × 3 \(\frac{1}{6}\)
First, Convert the mixed fraction to the improper fraction.
\(\frac{2}{3}\) × \(\frac{19}{6}\) = \(\frac{38}{18}\)
\(\frac{38}{18}\) = 2 \(\frac{2}{18}\)
2 \(\frac{2}{18}\) is less than 3 \(\frac{1}{6}\)
\(\frac{2}{3}\) × 3 \(\frac{1}{6}\) will be less than 3 \(\frac{1}{6}\)

Question 6.
2 × 2 \(\frac{1}{4}\) will be __________ 2 \(\frac{1}{4}\)

Answer: Greater than

Explanation:
2 × 2 \(\frac{1}{4}\)
First, Convert the mixed fraction to the improper fraction.
2 × \(\frac{9}{4}\) = \(\frac{18}{4}\)
\(\frac{18}{4}\) = 4 \(\frac{2}{4}\)
4 \(\frac{1}{2}\) is greater than 2 \(\frac{1}{4}\)

Question 7.
4 × 1 \(\frac{3}{7}\) will be __________ 1 \(\frac{3}{7}\)

Answer: Greater than

Explanation:
4 × 1 \(\frac{3}{7}\)
First, Convert the mixed fraction to the improper fraction.
4 × \(\frac{10}{7}\) = \(\frac{40}{7}\)
4 × 1 \(\frac{3}{7}\) = 5 \(\frac{5}{7}\)
5 \(\frac{5}{7}\) is greater than 1 \(\frac{3}{7}\)

Algebra Tell whether the unknown factor is less than 1 or greater than 1.

Question 8.
■ × 1 \(\frac{2}{3}\) = \(\frac{5}{6}\)
The unknown factor is __________ 1.

Answer: Less than

Explanation:
■ × 1 \(\frac{2}{3}\) = \(\frac{5}{6}\)
■ × \(\frac{5}{3}\) = \(\frac{5}{6}\)
■ = \(\frac{1}{2}\)
Thus the unknown factor is \(\frac{1}{2}\)
\(\frac{1}{2}\) is less than 1.

Question 9.
■ × 1 \(\frac{1}{4}\) = 2 \(\frac{1}{2}\)
The unknown factor is __________ 1.

Answer: Greater than

Explanation:
■ × 1 \(\frac{1}{4}\) = 2 \(\frac{1}{2}\)
■ = 2 \(\frac{1}{2}\) ÷ 1 \(\frac{1}{4}\)
■ = 2 × 2 = 4
■ = 4
Thus the unknown factor is 4
4 is greater than 1.

Problem Solving – Page No. 324

Question 10.
Kyle is making a scale drawing of his math book. The dimensions of his drawing will be \(\frac{1}{3}\) the dimensions of his book. If the width of his book is 8 \(\frac{1}{2}\) inches, will the width of his drawing be equal to, greater than, or less than 8 \(\frac{1}{2}\) inches?
__________

Answer: Less than

Explanation:
Given that,
Kyle is making a scale drawing of his math book.
The dimensions of his drawing will be \(\frac{1}{3}\) the dimensions of his book.
\(\frac{1}{3}\) × 8 \(\frac{1}{2}\)
First, Convert the mixed fraction to the improper fraction.
\(\frac{1}{3}\) × \(\frac{17}{2}\) = \(\frac{17}{6}\)
Convert the fraction into the mixed fraction.
\(\frac{17}{6}\) = 2 \(\frac{5}{6}\)
2 \(\frac{5}{6}\) is less than 8 \(\frac{1}{2}\) inches.

Question 11.
Sense or Nonsense?
Penny wants to make a model of a beetle that is larger than life-size. Penny says she is going to use a scaling factor of \(\frac{7}{12}\). Does this make sense or is it nonsense? Explain.
Type below:
__________

Answer: It is nonsense because Penny wants to make beetle Larger than life size. So, the scaling factor \(\frac{7}{12}\) is not corresponding, because when we multiply any value with the number less than 1 we get a smaller number.

Question 12.
Shannon, Mary, and John earn a weekly allowance. Shannon earns an amount that is \(\frac{2}{3}\) of what John earns. Mary earns an amount that is 1 \(\frac{2}{3}\) of what John earns. John earns $20 a week. Who earns the greatest allowance? Who earns the least?
__________ earns the greatest allowance.
__________ earns the least allowance

Answer:
Mary earns the greatest allowance.
Shannon earns the least allowance.

Explanation:

Shannon, Mary, and John earn a weekly allowance.
Shannon earns an amount that is \(\frac{2}{3}\) of what John earns.
Mary earns an amount that is 1 \(\frac{2}{3}\) of what John earns.
John earns $20 a week.
\(\frac{2}{3}\) ________ 1 \(\frac{2}{3}\)
Convert the mixed fraction into the improper fraction.
1 \(\frac{2}{3}\) = \(\frac{5}{3}\)
\(\frac{2}{3}\) is less than 1 \(\frac{2}{3}\)
Thus Shannon earns the least allowance and Mary earns the greatest allowance.

Question 13.
Test Prep Addie’s puppy weighs 1 \(\frac{2}{3}\) times what it weighed when it was born. It weighed 1 \(\frac{1}{3}\) pounds at birth. Which statement below is true?
Options:
a. The puppy weighs the same as it did at birth.
b. The puppy weighs less than it did at birth.
c. The puppy weighs more than it did at birth.
d. The puppy weighs twice what it did at birth.

Answer: The puppy weighs more than it did at birth.

Explanation:
Test Prep Addie’s puppy weighs 1 \(\frac{2}{3}\) times what it weighed when it was born.
It weighed 1 \(\frac{1}{3}\) pounds at birth.
1 \(\frac{2}{3}\) is greater than 1 \(\frac{1}{3}\).
So, the puppy weighs more than it did at birth.
Thus the correct answer is option C.

Share and Show – Page No. 327

Find the product. Write the product in simplest form.

Question 1.
1 \(\frac{2}{3}\) × 3 \(\frac{4}{5}\) = \(\frac{■}{3}\) × \(\frac{■}{5}\)
= \(\frac{■}{■}\)
=?
_____ \(\frac{□}{□}\)

Answer: 6 \(\frac{1}{3}\)

Explanation:

1 \(\frac{2}{3}\) × 3 \(\frac{4}{5}\)
\(\frac{5}{3}\) × \(\frac{19}{5}\)
\(\frac{■}{3}\) × \(\frac{■}{5}\) = \(\frac{5}{3}\) × \(\frac{19}{5}\)
\(\frac{5}{3}\) × \(\frac{19}{5}\) = 6 \(\frac{1}{3}\)

Question 2.
\(\frac{1}{2}\) × 1 \(\frac{1}{3}\)
Go Math Grade 5 Answer Key Chapter 7 Multiply Fractions img 24
Shade the model to find the product.
\(\frac{□}{□}\)

Answer: \(\frac{2}{3}\)

Explanation:
\(\frac{1}{2}\) × 1 \(\frac{1}{3}\)
1 \(\frac{1}{3}\) = \(\frac{4}{3}\)
\(\frac{1}{2}\) × \(\frac{4}{3}\) = \(\frac{4}{6}\)

Question 3.
\(1 \frac{1}{8} \times 2 \frac{1}{3}\) = ______ \(\frac{□}{□}\)

Answer: 2 \(\frac{5}{8}\)

Explanation:
1 \(\frac{1}{8}\) × 2 \(\frac{1}{3}\)
\(\frac{9}{8}\) × \(\frac{7}{3}\) = \(\frac{63}{24}\)
\(\frac{63}{24}\) = 2 \(\frac{63}{24}\) = 2 \(\frac{15}{24}\)
2 \(\frac{15}{24}\) = 2 \(\frac{5}{8}\)
\(1 \frac{1}{8} \times 2 \frac{1}{3}\) = 2 \(\frac{5}{8}\)

Question 4.
\(\frac{3}{4} \times 6 \frac{5}{6}\) = ______ \(\frac{□}{□}\)

Answer: 5 \(\frac{1}{8}\)

Explanation:
\(\frac{3}{4}\) × 6 \(\frac{5}{6}\)
\(\frac{3}{4}\) × \(\frac{41}{6}\)
\(\frac{123}{24}\) = \(\frac{41}{8}\)
Convert the fraction to the mixed fraction.
\(\frac{41}{8}\) = 5 \(\frac{1}{8}\)

Question 5.
\(1 \frac{2}{7} \times 1 \frac{3}{4}\) = ______ \(\frac{□}{□}\)

Answer: 2 \(\frac{1}{4}\)

Explanation:
1 \(\frac{2}{7}\) × 1 \(\frac{3}{4}\)
Multiply the numerators and the denominators.
Convert the mixed fraction to the improper fraction.
\(\frac{9}{7}\) × \(\frac{7}{4}\) = \(\frac{63}{28}\)
\(\frac{63}{28}\) = 2 \(\frac{1}{4}\)
\(1 \frac{2}{7} \times 1 \frac{3}{4}\) = 2 \(\frac{1}{4}\)

Question 6.
\(\frac{3}{4} \times 1 \frac{1}{4}\) = ______ \(\frac{□}{□}\)

Answer: \(\frac{15}{16}\)

Explanation:
\(\frac{3}{4}\) × 1 \(\frac{1}{4}\)
\(\frac{3}{4}\) × \(\frac{5}{4}\) = \(\frac{15}{16}\)
\(\frac{3}{4} \times 1 \frac{1}{4}\) = \(\frac{15}{16}\)

Use the Distributive Property to find the product.

Question 7.
\(16 \times 2 \frac{1}{2}\) = ______

Answer: 40

Explanation:
\(16 \times 2 \frac{1}{2}\)
(16 × 2) + (16 × \(\frac{1}{2}\))
32 + 8 = 40
\(16 \times 2 \frac{1}{2}\) = 40

Question 8.
\(1 \frac{4}{5} \times 15\) = ______

Answer: 27

Explanation:
\(1 \frac{4}{5} \times 15\)
15 × 1 \(\frac{4}{5}\)
(1 × 15) + (15 × \(\frac{4}{5}\))
15 + \(\frac{60}{5}\)
15 + 12 = 27
Thus \(1 \frac{4}{5} \times 15\) = 27

On Your Own

Find the product. Write the product in simplest form.

Question 9.
\(\frac{3}{4} \times 1 \frac{1}{2}\) = ______ \(\frac{□}{□}\)

Answer: 1 \(\frac{1}{8}\)

Explanation:
\(\frac{3}{4}\) × 1 \(\frac{1}{2}\)
\(\frac{3}{4}\) × \(\frac{3}{2}\) = \(\frac{9}{8}\)
Now convert the improper fraction to the mixed fraction.
\(\frac{9}{8}\) = 1 \(\frac{1}{8}\)

Question 10.
\(4 \frac{2}{5} \times 1 \frac{1}{2}\) = ______ \(\frac{□}{□}\)

Answer: 6 \(\frac{3}{5}\)

Explanation:
4 \(\frac{2}{5}\) × 1 \(\frac{1}{2}\)
Convert the mixed fraction to the improper fraction.
\(\frac{22}{5}\) × \(\frac{3}{2}\) = \(\frac{66}{10}\)
The mixed fraction of \(\frac{66}{10}\) is 6 \(\frac{3}{5}\)

Question 11.
\(5 \frac{1}{3} \times \frac{3}{4}\) = ______

Answer: 4

Explanation:
5 \(\frac{1}{3}\) × \(\frac{3}{4}\)
Convert the mixed fraction to the improper fraction.
\(\frac{16}{3}\) × \(\frac{3}{4}\) = \(\frac{48}{12}\)
12 divides 48 four times.
Thus \(5 \frac{1}{3} \times \frac{3}{4}\) = 4

Question 12.
\(2 \frac{1}{2} \times 5 \frac{1}{5}\) = ______

Answer: 13

Explanation:
2 \(\frac{1}{2}\) × 5 \(\frac{1}{5}\)
\(\frac{5}{2}\) × \(\frac{26}{5}\) = \(\frac{130}{10}\)
10 divides 130 thirteen times.
\(\frac{130}{10}\) = 13
\(2 \frac{1}{2} \times 5 \frac{1}{5}\) = 13

Question 13.
\(12 \frac{3}{4} \times 2 \frac{2}{3}\) = ______

Answer: 34

Explanation:
12 \(\frac{3}{4}\) × 2 \(\frac{2}{3}\)
\(\frac{51}{4}\) × \(\frac{6}{3}\)
3 divides 51 seventeen times.
17 × 2 = 34

Question 14.
\(3 \times 4 \frac{1}{2}\) = ______ \(\frac{□}{□}\)

Answer: 13 \(\frac{1}{2}\)

Explanation:
3 × 4 \(\frac{1}{2}\)
3 × \(\frac{9}{2}\) = \(\frac{27}{2}\)
Convert the fraction to the mixed fraction
\(\frac{27}{2}\) = 13 \(\frac{1}{2}\)

Question 15.
\(2 \frac{3}{8} \times \frac{4}{9}\) = ______ \(\frac{□}{□}\)

Answer: 1 \(\frac{1}{18}\)

Explanation:
2 \(\frac{3}{8}\) × \(\frac{4}{9}\)
\(\frac{19}{8}\) × \(\frac{4}{9}\) = \(\frac{76}{72}\)
\(\frac{76}{72}\) = 1 \(\frac{1}{18}\)
\(2 \frac{3}{8} \times \frac{4}{9}\) = 1 \(\frac{1}{18}\)

Question 16.
\(1 \frac{1}{3} \times 1 \frac{1}{4} \times 1 \frac{1}{5}\) = ______

Answer: 2

Explanation:

1 \(\frac{1}{3}\) × 1 \(\frac{1}{4}\) × 1 \(\frac{1}{5}\)
\(\frac{4}{3}\) × \(\frac{5}{4}\) × \(\frac{6}{5}\) = 2
\(1 \frac{1}{3} \times 1 \frac{1}{4} \times 1 \frac{1}{5}\) = 2

Use the Distributive Property to find the product.

Question 17.
\(10 \times 2 \frac{3}{5}\) = ______

Answer: 26

Explanation:
10 × 2 \(\frac{3}{5}\)
Now use the Distributive Property to find the product.
(10 × 2) + (10 × \(\frac{3}{5}\))
20 + \(\frac{30}{5}\)
5 divides 30 6 times.
20 + 6 = 26

Question 18.
\(3 \frac{3}{4} \times 12\) = ______

Answer: 45

Explanation:
3 \(\frac{3}{4}\) × 12
Now use the Distributive Property to find the product.
(12 × 3) + (12 × \(\frac{3}{4}\))
36 + \(\frac{36}{4}\)
36 + 9 = 45
\(3 \frac{3}{4} \times 12\) = 45

Share and Show Connect to health – Page No. 328

Changing Recipes

You can make a lot of recipes more healthful by reducing the amounts of fat, sugar, and salt.
Go Math Grade 5 Answer Key Chapter 7 Multiply Fractions img 25

Kelly has a muffin recipe that calls for 1 \(\frac{1}{2}\) cups of sugar. She wants to use \(\frac{1}{2}\) that amount of sugar and more cinnamon and vanilla. How much sugar will she use?
Multiply 1 \(\frac{1}{2}\) by \(\frac{1}{2}\) to find what part of the original amount of sugar to use.
Write the mixed number as a fraction greater than 1. Then, multiply.
\(\frac{1}{2} \times 1 \frac{1}{2}=\frac{1}{2} \times \frac{3}{2}\)
= \(\frac{3}{4}\)
So, Kelly will use \(\frac{3}{4}\) cup of sugar.

Question 19.
Michelle has a recipe that calls for 2 \(\frac{1}{2}\) cups of vegetable oil. She wants to use \(\frac{2}{3}\) that amount of oil and use applesauce to replace the rest. How much vegetable oil will she use?
______ \(\frac{□}{□}\) cups

Answer: 1 \(\frac{2}{3}\)

Explanation:
Michelle has a recipe that calls for 2 \(\frac{1}{2}\) cups of vegetable oil.
She wants to use \(\frac{2}{3}\) that amount of oil and use applesauce to replace the rest
Multiply 2 \(\frac{1}{2}\) by \(\frac{2}{3}\) to find how much vegetable oil she will use.
2 \(\frac{1}{2}\) × \(\frac{2}{3}\)
Convert the mixed fractions into the fractions.
\(\frac{5}{2}\) × \(\frac{2}{3}\) = \(\frac{10}{6}\)
\(\frac{10}{6}\) = \(\frac{5}{3}\) = 1 \(\frac{2}{3}\)
She will use 1 \(\frac{2}{3}\) cups of vegetable oil.

Question 20.
Tony’s recipe for soup calls for 1 \(\frac{1}{4}\) teaspoons of salt. He wants to use \(\frac{1}{2}\) that amount. How much salt will he use?
\(\frac{□}{□}\) teaspoon

Answer: \(\frac{5}{8}\)

Explanation:
Tony’s recipe for soup calls for 1 \(\frac{1}{4}\) teaspoons of salt.
He wants to use \(\frac{1}{2}\) that amount.
Multiply the fractions to find how much salt he will use in the recipe for soup.
1 \(\frac{1}{4}\) × \(\frac{1}{2}\)
Convert the mixed fractions to the improper fractions.
\(\frac{5}{4}\) × \(\frac{1}{2}\) = \(\frac{5}{8}\)
Thus Tony use \(\frac{5}{8}\) teaspoon of salt for soup.

Question 21.
Jeffrey’s recipe for oatmeal muffins calls for 2 \(\frac{1}{4}\) cups of oatmeal and makes one dozen muffins. If he makes 1 \(\frac{1}{2}\) dozen muffins for a club meeting, how much oatmeal will he use?
_____ \(\frac{□}{□}\) cups

Answer: 3 \(\frac{3}{8}\)

Explanation:
Jeffrey’s recipe for oatmeal muffins calls for 2 \(\frac{1}{4}\) cups of oatmeal and makes one dozen muffins.
To find how much oatmeal he will use we need to multiply the fractions.
2 \(\frac{1}{4}\) × 1 \(\frac{1}{2}\)
Convert the mixed fractions to the improper fractions.
\(\frac{9}{4}\) × \(\frac{3}{2}\)
\(\frac{27}{8}\) = 3 \(\frac{3}{8}\)
Thus he will use 3 \(\frac{3}{8}\) cups of oatmeal to make oatmeal muffins.

Question 22.
Cara’s muffin recipe calls for 1 \(\frac{1}{2}\) cups of flour for the muffins and \(\frac{1}{4}\) cup of flour for the topping. If she makes \(\frac{1}{2}\) of the original recipe, how much flour will she use?
\(\frac{□}{□}\) cup of flour

Answer: \(\frac{7}{8}\)

Explanation:
Convert mixed fractions into improper fractions.
1 \(\frac{1}{2}\) = \(\frac{3}{2}\)
\(\frac{3}{2}\) + \(\frac{1}{4}\) = \(\frac{7}{4}\)
Now we can find how much flour she will use to make \(\frac{1}{2}\) of the original recipe, when multiply
\(\frac{7}{4}\) by \(\frac{1}{2}\)
\(\frac{7}{4}\) × \(\frac{1}{2}\) = \(\frac{7}{8}\)

Share and Show – Page No. 331

Go Math Grade 5 Answer Key Chapter 7 Multiply Fractions img 26

Question 1.
When Pascal built a dog house, he knew he wanted the floor of the house to have an area of 24 square feet. He also wanted the width to be \(\frac{2}{3}\) the length. What are the dimensions of the dog house?
First, choose two numbers that have a product of 24.
Guess: ____ feet and ____ feet
Then, check those numbers. Is the greater number \(\frac{2}{3}\) of the other number?
Check: \(\frac{2}{3}\) × _____ = _____
My guess is ______.
Finally, if the guess is not correct, revise it and check again. Continue until you find the correct answer.
_____ feet by _____ feet

Answer: 4 feet by 6 feet

Explanation:
When Pascal built a dog house, he knew he wanted the floor of the house to have an area of 24 square feet.
He also wanted the width to be \(\frac{2}{3}\) the length.
My guess for 24 square feet is 4 feet and 6 feet.
Now let us check the numbers.
6 × \(\frac{2}{3}\) = 4
So my guess is correct.
Thus the dimensions are 4 feet by 6 feet

Question 2.
What if Pascal wanted the area of the floor to be 54 square feet and the width still to be \(\frac{2}{3}\) the length? What would the dimensions of the floor be?
_____ feet by _____ feet

Answer: 6 feet by 9 feet

Explanation:
My guess for 54 square feet is  6 feet and 9 feet.
9 × \(\frac{2}{3}\)
3 divides 9 three times.
9 × \(\frac{2}{3}\) = 6
So, my guess is correct.
Therefore the dimensions of the will be 6 feet by 9 feet

Question 3.
Leo wants to paint a mural that covers a wall with an area of 1,440 square feet. The height of the wall is \(\frac{2}{5}\) of its length. What is the length and the height of the wall?
_____ feet by _____ feet

Answer: 24 feet by 60 feet

Explanation:
Leo wants to paint a mural that covers a wall with an area of 1,440 square feet. The height of the wall is \(\frac{2}{5}\) of its length.
Guess: 1,440 square feet = 24 feet × 60 feet
\(\frac{2}{5}\) × 60 = 24
So, our guess is correct.
.Thus the dimensions of the wall are 24 feet by 60 feet.

On Your Own – Page No. 332

Question 4.
Barry wants to make a drawing that is \(\frac{1}{4}\) the size of the original. If a tree in the original drawing is 14 inches tall, how tall will the tree in Barry’s drawing be?
_____ \(\frac{□}{□}\) inches

Answer: 3 \(\frac{1}{2}\) inches

Explanation:
Given:
Barry wants to make a drawing that is \(\frac{1}{4}\) the size of the original.
The tree is 14 inches tall in the drawing.
14 × \(\frac{1}{4}\) = \(\frac{14}{4}\) = \(\frac{7}{2}\)
Convert the fraction to the mixed fraction.
\(\frac{7}{2}\) = 3 \(\frac{1}{2}\) inches

Question 5.
A blueprint is a scale drawing of a building. The dimensions of the blueprint for Penny’s doll house are \(\frac{1}{4}\) of the measurements of the actual doll house. The floor of the doll house has an area of 864 square inches. If the width of the doll house is \(\frac{2}{3}\) the length, what are the dimensions of the floor on the blueprint of the doll house?
_____ inches by _____ inches

Answer: 9 inches by 6 inches

Explanation:
A blueprint is a scale drawing of a building.
The dimensions of the blueprint for Penny’s dollhouse are \(\frac{1}{4}\) of the measurements of the actual dollhouse.
The floor of the dollhouse has an area of 864 square inches.
The area of the dollhouse is 54 square inches.
My guess is 9 inches by 6 inches
Let us check the numbers
9 × \(\frac{2}{3}\) = 6
My guess is correct.
Therefore the dimensions of the floor on the blueprint of the dollhouse is 9 inches by 6 inches

Question 6.
Pose a Problem Look back at Exercise 4. Write a similar problem using a different measurement and a different fraction. Then solve your problem.
Type below:
__________

Answer:
Kyle is making reusable grocery bags and lunch bags. She used a 3/4 yard of cloth to make the grocery bag. A lunch bag requires 2/3 of the amount of cloth of a grocery bag’s needs. How much does she need to make the lunch bag?
\(\frac{3}{4}\) × \(\frac{2}{3}\) = \(\frac{1}{2}\)
Thus Kyle needs \(\frac{1}{2}\) of the grocery bag to make the lunch bag.

Question 7.
Test Prep Albert’s photograph has an area of 80 square inches. The length of the photo is 1 \(\frac{1}{4}\) the width. Which of the following could be the dimensions of the photograph?
Options:
a. 5 inches by 16 inches
b. 12 inches by 10 inches
c. 6 inches by 5 inches
d. 10 inches by 8 inches

Answer: 10 inches by 8 inches

Explanation:
Albert’s photograph has an area of 80 square inches.
The length of the photo is 1 \(\frac{1}{4}\) the width.
My guess for 80 square inches is 10 inches by 8 inches.
Now let us check the numbers.
8 × 1 \(\frac{1}{4}\) = 8 × \(\frac{5}{4}\) = 10
Thus the correct answer is option D.

Chapter Review/Test – Page No. 333

Concepts and Skills

Question 1.
When you multiply 3 \(\frac{1}{4}\) by a number greater than one, how does the product compare to 3 \(\frac{1}{4}\)? Explain.
Type below:
__________

Answer:
Your product will be greater than 3 1/4 because anytime you multiply a fraction times a whole number less than 1 you get a fraction less than one and any time you multiply by a fraction and a whole number greater than 1 your answer is greater than 1.

Use a model to solve.

Question 2.
\(\frac{2}{3}\) × 6 = _____

Answer: 4

Explanation:
\(\frac{2}{3}\) × 6
3 divides 6 two times.
2 × 2 = 4
\(\frac{2}{3}\) × 6 = 4

Question 3.
\(\frac{3}{7}\) × 14 = _____

Answer: 6

Explanation:
\(\frac{3}{7}\) × 14
7 divides 14 two times.
3 × 2 = 6
\(\frac{3}{7}\) × 14 = 6

Question 4.
\(\frac{5}{8}\) × 24 = _____

Answer: 15

Explanation:

\(\frac{5}{8}\) × 24
8 divides 24 three times.
5 × 3 = 15
\(\frac{5}{8}\) × 24 = 15

Find the product. Write the product in simplest form.

Question 5.
\(\frac{3}{5}\) × 8 = _____ \(\frac{□}{□}\)

Answer: 4 \(\frac{4}{5}\)

Explanation:
\(\frac{3}{5}\) × 8 = \(\frac{24}{5}\)
The mixed fraction of \(\frac{24}{5}\) is 4 \(\frac{4}{5}\)
\(\frac{3}{5}\) × 8 = 4 \(\frac{4}{5}\)

Question 6.
\(\frac{1}{4}\) × 10 = _____ \(\frac{□}{□}\)

Answer: 2 \(\frac{1}{2}\)

Explanation:
\(\frac{1}{4}\) × 10
2 divides 10 five times.
\(\frac{1}{2}\) × 5 = \(\frac{5}{2}\)
The mixed fraction of \(\frac{5}{2}\) is 2 \(\frac{1}{2}\)
\(\frac{1}{4}\) × 10 = 2 \(\frac{1}{2}\)

Question 7.
\(\frac{7}{5}\) × 15 = _____

Answer: 21

\(\frac{7}{5}\) × 15
5 divides 15 three times.
\(\frac{7}{5}\) × 15 = 7 × 3 = 21
\(\frac{7}{5}\) × 15 = 21

Question 8.
\(\frac{5}{6}\) × \(\frac{2}{3}\) = \(\frac{□}{□}\)

Answer: \(\frac{5}{9}\)

Explanation:
\(\frac{5}{6}\) × \(\frac{2}{3}\) = \(\frac{10}{18}\)
\(\frac{10}{18}\) = \(\frac{5}{9}\)
Thus \(\frac{5}{6}\) × \(\frac{2}{3}\) = \(\frac{5}{9}\)

Question 9.
\(\frac{1}{5}\) × \(\frac{5}{7}\) = \(\frac{□}{□}\)

Answer: \(\frac{1}{7}\)

Explanation:
\(\frac{1}{5}\) × \(\frac{5}{7}\)
5 in the numerator and 5 in the denominator gets canceled.
= \(\frac{1}{7}\)
Thus \(\frac{1}{5}\) × \(\frac{5}{7}\) = \(\frac{1}{7}\)

Question 10.
\(\frac{3}{8}\) × \(\frac{1}{6}\) = \(\frac{□}{□}\)

Answer: \(\frac{1}{16}\)

Explanation:
\(\frac{3}{8}\) × \(\frac{1}{6}\)
3 divides 6 two times
\(\frac{3}{8}\) × \(\frac{1}{6}\) = \(\frac{1}{8}\) × \(\frac{1}{2}\)
Multiply the denominators.
= \(\frac{1}{16}\)
Thus \(\frac{3}{8}\) × \(\frac{1}{6}\) = \(\frac{1}{16}\)

Complete the statement with equal to, greater than, or less than.

Question 11.
\(\frac{7}{8}\) × \(\frac{6}{6}\) will be __________ \(\frac{7}{8}\)

Answer: Equal to

Explanation:
\(\frac{7}{8}\) × \(\frac{6}{6}\)
\(\frac{6}{6}\) = 1
\(\frac{7}{8}\) × 1 = \(\frac{7}{8}\)
\(\frac{7}{8}\) = \(\frac{7}{8}\)
Thus \(\frac{7}{8}\) × \(\frac{6}{6}\) will be equal to \(\frac{7}{8}\)

Question 12.
\(\frac{1}{2}\) × \(\frac{8}{9}\) will be __________ \(\frac{8}{9}\)

Answer: Less than

Explanation:
\(\frac{1}{2}\) × \(\frac{8}{9}\)
Multiply the numerators and denominators
\(\frac{1}{2}\) × \(\frac{8}{9}\) = \(\frac{4}{9}\)
\(\frac{4}{9}\) is less than \(\frac{8}{9}\)
So, \(\frac{1}{2}\) × \(\frac{8}{9}\) will be less than \(\frac{8}{9}\)

Chapter Review/Test – Page No. 334

Fill in the bubble completely to show your answer.

Question 13.
Wolfgang wants to enlarge a picture he developed. Which factor listed below would scale up (enlarge) his picture the most if he used it to multiply its current dimensions?
Options:
a. \(\frac{7}{8}\)
b. \(\frac{14}{14}\)
c. 1 \(\frac{4}{9}\)
d. \(\frac{3}{2}\)

Answer: 1 \(\frac{4}{9}\)

Explanation:
The greatest fraction among all the fractions is 1 \(\frac{4}{9}\).
1 \(\frac{4}{9}\) is greater than 1.
Thus the correct answer is option C.

Question 14.
Rachel wants to reduce the size of her photo. Which factor listed below would scale down (reduce) the size of her picture the most?
Options:
a. \(\frac{5}{8}\)
b. \(\frac{11}{16}\)
c. 1 \(\frac{3}{4}\)
d. \(\frac{8}{5}\)

Answer: \(\frac{5}{8}\)

Explanation:
Compare to all the fractions \(\frac{5}{8}\) is smaller.
So, Rachel would reduce the size of her picture to \(\frac{5}{8}\)
So, the correct answer is option A.

Question 15.
Marteen wants to paint \(\frac{2}{3}\) of her room today. She wants to paint \(\frac{1}{4}\) of that before lunch. How much of her room will she paint today before lunch?
Options:
a. \(\frac{1}{12}\)
b. \(\frac{1}{6}\)
c. 1 \(\frac{5}{12}\)
d. \(\frac{11}{12}\)

Answer: \(\frac{1}{6}\)

Explanation:
Marteen wants to paint \(\frac{2}{3}\) of her room today.
She wants to paint \(\frac{1}{4}\) of that before lunch.
\(\frac{2}{3}\) × \(\frac{1}{4}\) = \(\frac{1}{6}\)
So, the answer is option B.

Chapter Review/Test – Page No. 335

Fill in the bubble completely to show your answer.

Question 16.
Gia’s bus route to school is 5 \(\frac{1}{2}\) miles. The bus route home is 1 \(\frac{3}{5}\) times as long. How long is Gia’s bus route home?
Options:
a. 5 \(\frac{3}{10}\) miles
b. 8 miles
c. 8 \(\frac{4}{5}\) miles
d. 17 \(\frac{3}{5}\) miles

Answer: 8 \(\frac{4}{5}\) miles

Explanation:
Gia’s bus route to school is 5 \(\frac{1}{2}\) miles. The bus route home is 1 \(\frac{3}{5}\) times as long.
5 \(\frac{1}{2}\) × 1 \(\frac{3}{5}\)
Convert the mixed fractions to improper fractions.
\(\frac{11}{2}\) × \(\frac{8}{5}\) = \(\frac{88}{10}\) = \(\frac{44}{5}\)
The mixed fraction of \(\frac{44}{5}\) is 8 \(\frac{4}{5}\) miles
Therefore the answer is option C.

Question 17.
Carl’s dog weighs 2 \(\frac{1}{3}\) times what Judy’s dog weighs. If Judy’s dog weighs 35 \(\frac{1}{2}\) pounds, how much does Carl’s dog weigh?
Options:
a. 88 \(\frac{3}{4}\) pounds
b. 82 \(\frac{5}{6}\) pounds
c. 81 \(\frac{2}{3}\) pounds
d. 71 pounds

Answer: 82 \(\frac{5}{6}\) pounds

Explanation:
Carl’s dog weighs 2 \(\frac{1}{3}\) times what Judy’s dog weighs.
To find the weigh of Carl’s dog we need to multiply the fractions
2 \(\frac{1}{3}\) and 35 \(\frac{1}{2}\)
\(\frac{7}{3}\) × \(\frac{71}{2}\) = \(\frac{497}{6}\)
The mixed fraction of \(\frac{497}{6}\) is 82 \(\frac{5}{6}\) pounds.
Thus the correct answer is option B.

Question 18.
In a fifth grade class, \(\frac{4}{5}\) of the girls have brown hair. Of the brown-haired girls, \(\frac{3}{5}\) of the girls have long hair. What fraction of the girls in the class have long brown hair?
Options:
a. \(\frac{1}{20}\)
b. \(\frac{1}{5}\)
c. \(\frac{3}{5}\)
d. \(\frac{1}{4}\)

Answer: \(\frac{1}{5}\)

Explanation:
In a fifth grade class, \(\frac{4}{5}\) of the girls have brown hair. Of the brown-haired girls, \(\frac{3}{5}\) of the girls have long hair.
\(\frac{4}{5}\) – \(\frac{3}{5}\) = \(\frac{1}{5}\)
The correct answer is option B.

Chapter Review/Test – Page No. 336

Constructed Response

Question 19.
Tasha plans to tile the floor in her room with square tiles that are \(\frac{1}{4}\) foot long. Will she use more or fewer tiles if she is only able to purchase square tiles that are \(\frac{1}{3}\) foot long? Explain.
_________ tiles

Answer: Fewer

Explanation:
\(\frac{1}{4}\) is less than \(\frac{1}{3}\)
So, Tasha will use fewer tiles if she is only able to purchase square tiles that are \(\frac{1}{3}\) long.

Performance Task

Question 20.
For a bake sale, Violet wants to use the recipe below.
Go Math Grade 5 Answer Key Chapter 7 Multiply Fractions Chapter Review/Test img 27
A). If she wants to double the recipe, how much flour will she need?
_____ \(\frac{□}{□}\) cups flour

Answer: 5 \(\frac{1}{2}\) cups flour

Explanation:
To bake the sugar cookies she needs 2 \(\frac{3}{4}\) cups flour.
If she wants to double the recipe, she needs to multiply the 2 \(\frac{3}{4}\) cups flour by 2.
2 \(\frac{3}{4}\) + 2 \(\frac{3}{4}\) = 5 \(\frac{1}{2}\) cups flour

Question 20.
B). Baxter wants to make 1 \(\frac{1}{2}\) times the recipe. Will he need more or less sugar than Violet needs if she doubles the recipe? Explain.
__________ sugar

Answer: less

Explanation:
If violet doubles the recipe he need 2 × 1 \(\frac{1}{2}\) = 3 cups of sugar
Baxter wants to make 1 \(\frac{1}{2}\) times the recipe.
1 \(\frac{1}{2}\) × \(\frac{1}{2}\) = 1 \(\frac{3}{4}\)
Baxter needs less sugar when compared to Violet’s recipe.

Question 20.
C). As shown, the recipe makes 60 cookies. Jorge wants to bring 150 cookies. How much flour will he need to make 150 cookies? Explain how you got your answer. (Hint: what can you multiply 60 by to get 150?)
_____ \(\frac{□}{□}\) cups flour

Answer: 2 \(\frac{1}{2}\) cups flour

Explanation:
The recipe makes 60 cookies. Jorge wants to bring 150 cookies.
Let the amount of flour be x.
60 × x = 150
x = 150/60 = 5/2
The mixed fraction of \(\frac{5}{2}\) is 2 \(\frac{1}{2}\)
Thus, Jorge need 2 \(\frac{1}{2}\) cups flour to make 150 cookies.

Conclusion

Make the most out of the Go Math Answer Key for Grade 5 Ch 7 Multiply Fractions. Our guide on 5th Grade Chapter 7 Multiply Fractions helps you to clarify the concepts related in no time. Keep in touch with our site to avail Go Math Answer Key for different Grades all in one place.

Go Math Grade 3 Answer Key Chapter 8 Understand Fractions Assessment Test

Go Math Grade 3 Answer Key Chapter 8 Understand Fractions Assessment Test contains all the topics of chapter 8. We have mentioned Step by Step Solutions for all the Questions in Practice Test, Chapter Test, Cumulative Practice, etc. Make sure to cross-check your knowledge in Chapter 8 by referring to our Go Math Grade 3 Answer Key Understand Fractions Assessment Test.

Go Math Grade 3 Answer Key Chapter 8 Understand Fractions Assessment Test

Enhance your Math Skills by referring to the Go Math Grade 3 Answer Key Chapter 8 Understand Fractions Assessment Test. Score better grades in your exam taking the help of the 3rd Grade Go Math Answer Key Ch 8 Understand Fractions Assessment Test. Understand where you went wrong and clarify all your queries. Students can practice using these Go Math Grade 3 Chapter 8 Understand Fractions Assessment Test where you lag.

Chapter 8: Understand Fractions Assessment Test

Test – Page 1 – Page No. 81

Question 1.
Each shape is divided into equal parts. Select the shapes that show fourths. Mark all that apply.
Options:
a. Go Math Grade 3 Answer Key Chapter 8 Understand Fractions Assessment Test Test - Page 1 img 1
b. Go Math Grade 3 Answer Key Chapter 8 Understand Fractions Assessment Test Test - Page 1 img 2
c. Go Math Grade 3 Answer Key Chapter 8 Understand Fractions Assessment Test Test - Page 1 img 3
d. Go Math Grade 3 Answer Key Chapter 8 Understand Fractions Assessment Test Test - Page 1 img 4

Answers: a,c,d 

Explanation: a,c,d shapes that show fourths. As a,c,d shapes are divided into four parts.

Question 2.
What fraction names the shaded part of the shape?
Go Math Grade 3 Answer Key Chapter 8 Understand Fractions Assessment Test Test - Page 1 img 5
Options:
a. 3 eighths
b. 5 eighths
c. 6 eighths
d. 8 eighths

Answer: b

Explanation: The given shape is divided into 8 equal parts in which 5 parts are shaded, so fraction name of the shaded part is 5 eights. i.e. 5/8.

Question 3.
The shaded part of the model shows what part of a garden is planted with peas. What fraction names the shaded part? Explain how you know how to write the fraction.
Go Math Grade 3 Answer Key Chapter 8 Understand Fractions Assessment Test Test - Page 1 img 6
\(\frac{□}{□}\)

Answer: \(\frac{1}{6}\).

Explanation: Given a model of the garden shows 6 equal parts in it. Out of which, 1 part is where peas are planted is shaded. Therefore, fraction names of the shaded part is \(\frac{1}{6}\).

Test – Page 2 – Page No. 82

Question 4.
What fraction names point A on the number line?
Go Math Grade 3 Answer Key Chapter 8 Understand Fractions Assessment Test Test - Page 2 img 7
\(\frac{□}{□}\)

Answer: \(\frac{7}{8}\)

Explanation: Given the number line is divided into 8 parts, A is at 7th position. Therefore, the fraction name of point A on the number line is \(\frac{7}{8}\).

Question 5.
Kwan divided this circle into equal parts. Circle the word that makes the sentence true.
Go Math Grade 3 Answer Key Chapter 8 Understand Fractions Assessment Test Test - Page 2 img 8
The circle is divided into Go Math Grade 3 Answer Key Chapter 8 Understand Fractions Assessment Test Test - Page 2 img 9
_________

Answer: Sixths.

Explanation: Kwan divided the circle into six parts. Therefore the answer is Sixths.

Question 6.
Sophie uses 16 beads to make a necklace. One fourth of the beads are purple. How many of the beads are purple?
_____ beads

Answer: 4

Explanation:
Total no of beads used by Sophie is = 16
a portion of purple beads used = ¼
Therefore total no of purple beads = 16× ¼ = 4 beads.

Question 7.
Four brothers work together to mow 3 equal-sized fields. Each brother mows the same amount.
Go Math Grade 3 Answer Key Chapter 8 Understand Fractions Assessment Test Test - Page 2 img 10
How much does each brother mow? Mark all that apply.
Options:
a. 4 thirds of a field
b. 1 whole and 1 third of a field
c. 3 fourths of a field
d. 2 thirds of a field
e. 1 fourth of a field

Answer: c

Explanation:
No of persons to work = 4
total no of fields = 3
So dividing 3 equal-sized fields 4 equal parts = ¾
Therefore each person gets ¾ of the field as their share to mow.

Test – Page 3 – Page No. 83

Question 8.
Bailey shaded this model.
Go Math Grade 3 Answer Key Chapter 8 Understand Fractions Assessment Test Test - Page 3 img 11
Select one number from each column to show the part of the model Bailey shaded.
Go Math Grade 3 Answer Key Chapter 8 Understand Fractions Assessment Test Test - Page 3 img 12

Answer: Numerator – 4, Denominator – 6.

Explanation: Given the model shows 6 equal parts in which Bailey shaded 4 parts. Therefore, the fractional part Bailey shaded is 4/6. From the fraction Numerator is 4, Denominator is 6.

Question 9.
Jayson baked a pan of cornbread for a family dinner. He cut the cornbread into equal size pieces. At the end of the dinner, there were 2 pieces left. Explain how you find the number of pieces in the whole pan of cornbread if Jayson told you that 16 of the pan was left. Use a drawing to show your work.
_____ pieces

Answer: 12 pieces

Explanation:

Go Math Grade 3 Answer Key Chapter 8 Understand Fractions Assessment Test
No of bread pieces left after dinner = 2
Considering 1/6 of the pan was left, we draw 6 ovals since the denominator is 6. Since 2 pieces are 1/6 of the pan, we represent them two circles in each oval. Therefore total no of pieces in the whole pan of cornbread are = 6 × 2 = 12.

Test – Page 4 – Page No. 84

Question 10.
The model shows one whole. What fraction of the model is NOT shaded?
Go Math Grade 3 Answer Key Chapter 8 Understand Fractions Assessment Test Test - Page 4 img 13
\(\frac{□}{□}\)

Answer: \(\frac{3}{4}\) of the model is not shaded.

Explanation: Given model is a circle that is divided into 4 parts. Out of which 1 part is shaded. Thus \(\frac{3}{4}\) of the model is not shaded.

Question 11.
Michael replaced \(\frac{1}{3}\) of the buttons on his coat. If he replaced a total of 3 buttons, how many buttons are on his coat? Show your work.
______ buttons

Answer: 9 buttons.

Explanation:
Let’s consider the total no of buttons be X
Given that Michael replaced 1/3 of his coat buttons which are 3 in number
which means 1/3 × X = 3
therefore X = 3×3 = 9
So the total no of buttons on his coat are 9.

Question 12.
Four teachers share 3 gallons of paint equally. How much paint does each teacher get?
Go Math Grade 3 Answer Key Chapter 8 Understand Fractions Assessment Test Test - Page 4 img 14
_____ fourths

Answer: ¾

Explanation:
No of teachers to share paint = 4
the total amount of paint = 3 gallons
So dividing 3 gallons of paint into 4 equal parts = ¾
Therefore each teacher gets ¾ of paint as their share.

Question 13.
Each shape is 1 whole.
Go Math Grade 3 Answer Key Chapter 8 Understand Fractions Assessment Test Test - Page 4 img 15
For numbers 13a–13e, choose Yes or No to show whether the number names the parts that are shaded.
a. 3
i. yes
ii. no

Answer: No

Explanation: The total 4 models are shaded.

Question 13.
b. 4
i. yes
ii. no

Answer: Yes

Explanation: The 4 models are shaded.

Question 13.
c. \(\frac{12}{3}\)
i. yes
ii. no

Answer: Yes

Explanation: \(\frac{12}{3}\) means 4 parts, which represents 4 shaded parts/model.

Question 13.
d. \(\frac{12}{4}\)
i. yes
ii. no

Answer: No

Explanation: \(\frac{12}{4}\) means 3 parts, which represents 3 shaded parts/model. So answer is no.

Question 13.
e. \(\frac{3}{12}\)
i. yes
ii. no

Answer: No.

Explanation: \(\frac{3}{12}\) means \(\frac{1}{4}\) part is shaded. So answer is no.

Test – Page 5 – Page No. 85

Question 14.
A store sold 6 fruit trees. Five of the trees were apple trees. What fraction of the trees were apple trees?
\(\frac{□}{□}\)

Answer: \(\frac{5}{6}\)

Explanation:
Given that total of 6 trees were sold
no of apple trees are 5
Therefore fraction of apple trees out of total sold trees are = No of apple trees / Total no of trees
= \(\frac{5}{6}\).

Question 15.
Christina and Erin made vegetable trays for a neighborhood picnic.
Part A
Christina put 15 vegetables on her tray. If \(\frac{1}{5}\) of the vegetables were carrots, how many carrots were on the tray? Make a drawing to show your work.
_____ carrots

Answer: 3 carrots.

Explanation:

Go Math Grade 3 Answer Key Chapter 8 Understand Fractions Assessment Test

Total vegetables on Christina tray = 15
part of carrots on the tray  = 1/5
Total of carrots = (Total no of vegetables × part of the carrots)
= 15 × 1/5 = 3 carrots.

Question 15.
Part B
Erin put 20 vegetables on her tray. If \(\frac{1}{4}\) of them were carrots, how many carrots were on Christina’s and Erin’s trays? Explain how you found your answer.
_____ carrots

Answer: Total 8 carrots.

Explanation:
No of carrots in Christina’s tray = 3
Total vegetables on Erin’s tray = 20
part of carrots on the tray  = 1/4
Total of carrots = (Total no of vegetables × part of the carrots)
= 20 × 1/4
= 5 carrots.
Therefore total no of carrots on both of their trays = 5 + 3 = 8 carrots

Question 16.
Maria has 8 tulip bulbs. She gives 5 of the tulip bulbs to her neighbor. What fraction of her tulip bulbs does she give to her neighbor?
\(\frac{□}{□}\) tulip bulbs

Answer: \(\frac{5}{8}\)

Explanation:
Total no of tulip bulbs Maria has = 8
No of tuple bulbs she gave to neighbor = 5
Therefore fraction of tuple bulbs she gave =\(\frac{5}{8}\).

Test – Page 6 – Page No. 86

Question 17.
Lucy rode her bike around the block 4 times for a total of 1 mile yesterday. Today she wants to ride her bike \(\frac{3}{4}\) of a mile. How many times will she need to ride her bike around the block?
Go Math Grade 3 Answer Key Chapter 8 Understand Fractions Assessment Test Test - Page 6 img 16
______ times

Answer: 3 times

Explanation:
No of times Lucy rode to complete 1 mile = 4
So in one time, she completes ¼ mile
Therefore to complete ¾ mile, she has to complete = ¼ + ¼ + ¼, which is 3 times

Question 18.
Jackson colored some shapes.
Go Math Grade 3 Answer Key Chapter 8 Understand Fractions Assessment Test Test - Page 6 img 17
Select one number from each column to show a fraction greater than 1 that names the parts Jackson colored.
Go Math Grade 3 Answer Key Chapter 8 Understand Fractions Assessment Test Test - Page 6 img 18

Answer: Numerator – 6, Denominator – 2

Explanation: As there are 6 halves that are shaded and the fraction greater than 1 is \(\frac{6}{2}\).

Question 19.
Samira ran around a park loop that was \(\frac{1}{3}\) mile long. She ran around the loop 9 times. Samira says she ran \(\frac{9}{3}\) miles. Her brother Amal says she ran 3 miles. Who is correct? Use words and drawings to explain how you know.
_________

Answer: Both are correct.

Explanation:
The distance of the loop is = \(\frac{1}{3}\) mile
Samira ran around nine times = 9 × \(\frac{1}{3}\)
=\(\frac{9}{3}\)
= 3miles

So both are correct.

Go Math Grade 3 Answer Key Chapter 8 Understand Fractions Assessment Test

Each circle represents a park where each shaded part we considered as 1/3 mile. Each circle can be completed in 3 rounds, therefore running nine times covers all the three circles = 9 ×\(\frac{1}{3}\)
= \(\frac{9}{3}\).
= 3miles.

Conclusion

We wish the knowledge shed regarding Go Math Grade 3 Answer Key Chapter 8 Assessment Test is helpful to you. In case of any other queries feel free to reach us and we will try to help you. Do check your knowledge by accessing our Go Math Grade 3 Answer Key Chapter 8 Understand Fractions Extra Practice and improve your math skills.

Go Math Grade 3 Answer Key Chapter 10 Time, Length, Liquid Volume, and Mass

go-math-grade-3-chapter-10-time-length-liquid-volume-and-mass-answer-key

Make use of the Go Math Grade 3 Answer Key Chapter 10 Time, Length, Liquid Volume, and Mass and download them free of cost. Refer to HMH Go Math 3rd Grade Chapter 10 during your preparation and understand the topics easily. Solve the Questions from Chapter Test, Practice Test, and cross-check your answers using the Go Math Grade 3 Answer Key. Master the subject and get a grip on the concepts in Chapter 10 Time Length Liquid Volume and Mass with the Grade 3 Go Math Answer Key for Chapter 10.

Go Math 3rd Grade Answer Key Chapter 10 Time Length Liquid Volume and Mass

You will have different topics like Time to the Minute, Measure Time Intervals, Use Time Intervals, etc. Concepts of 3rd Grade Chapter 10 will be no longer difficult with our Go Math Grade 3 Solutions Key Ch 10 Time Length Liquid Volume and Mass. Step by Step Solutions provided helps you to master the concepts and become a pro in the subject.

Time to the Minute – Page No. 565

Write the time. Write one way you can read the time.

Question 1.
Go Math Grade 3 Answer Key Chapter 10 Time, Length, Liquid Volume, and Mass Time to the Minute img 1

Answer:  1:16; sixteen minutes after one

Explanation:
Count on by fives and ones from the 12 on the clock to where the minute hand is pointing. Write the missing counting numbers next to the clock.
Write: 1 Hour 16 minutes
Read:  sixteen minutes after  one

Question 2.
Go Math Grade 3 Answer Key Chapter 10 Time, Length, Liquid Volume, and Mass Time to the Minute img 2
Type below:
___________

Answer: 10:20; twenty minutes after ten

Explanation:
Count on by fives and ones from the 12 on the clock to where the minute hand is pointing. Write the missing counting numbers next to the clock.
Write: 10 Hour 20 minutes
Read:  twenty minutes after  ten

Question 3.
Go Math Grade 3 Answer Key Chapter 10 Time, Length, Liquid Volume, and Mass Time to the Minute img 3
Type below:
____________

Answer: 4:13;  thirteen minutes after four

Explanation:
Count on by fives and ones from the 12 on the clock to where the minute hand is pointing. Write the missing counting numbers next to the clock.
Write: 4 Hour 13 minutes
Read:  thirteen minutes after  four

Question 4.
Go Math Grade 3 Answer Key Chapter 10 Time, Length, Liquid Volume, and Mass Time to the Minute img 4
Type below:
____________

Answer: 12:05; five minutes after twelve

Explanation:
Count on by fives and ones from the 12 on the clock to where the minute hand is pointing. Write the missing counting numbers next to the clock.
Write: 12 Hour 05 minutes
Read:  five minutes after  twelve

Question 5.
Go Math Grade 3 Answer Key Chapter 10 Time, Length, Liquid Volume, and Mass Time to the Minute img 5
Type below:
____________

Answer: 7:24; twenty four minutes after seven

Explanation:
Count on by fives and ones from the 12 on the clock to where the minute hand is pointing. Write the missing counting numbers next to the clock.
Write: 7 Hour 24 minutes
Read: twenty-four minutes after seven

Question 6.
Go Math Grade 3 Answer Key Chapter 10 Time, Length, Liquid Volume, and Mass Time to the Minute img 6
Type below:
____________

Answer: 2:51; nine minutes before three

Explanation:
Count by fives and ones from the 12 on the clock back to where the minute hand is pointing. Write the missing counting numbers next to the clock.
Write: 2:51
Read:
nine minutes before three

Write the time another way.

Question 7.
23 minutes after 4
______ : ______

Answer: 4:23

Explanation:
Count on by fives and ones from the 12 on the clock to where the minute hand is pointing. Write the missing counting numbers next to the clock.

Question 8.
18 minutes before 11
______ : ______

Answer: 10:42

Explanation:
Count by fives and ones from the 12 on the clock back to where the minute hand is pointing. Write the missing counting numbers next to the clock.

Question 9.
10 minutes before 9
______ : ______

Answer: 8:50

Explanation:
Count by fives and ones from the 12 on the clock back to where the minute hand is pointing. Write the missing counting numbers next to the clock.

Question 10.
7 minutes after 1
______ : ______

Answer: 1:07

Explanation:
Count on by fives and ones from the 12 on the clock to where the minute hand is pointing. Write the missing counting numbers next to the clock.

Problem Solving

Question 11.
What time is it when the hour hand is a little past the 3 and the minute hand is pointing to the 3?
______ : ______

Answer: 3:15

Explanation:
Count on by fives and ones from the 12 on the clock to where the minute hand is pointing. Write the missing counting numbers next to the clock.

Question 8.

Question 12.
Pete began practicing at twenty-five minutes before eight. What is another way to write this time?
______ : ______

Answer: 7:35

Explanation:
Count by fives and ones from the 12 on the clock back to where the minute hand is pointing. Write the missing counting numbers next to the clock.

Time to the Minute – Page No. 566

Lesson Check

Question 1.
Which is another way to write 13 minutes before 10?
Options:
a. 9:47
b. 10:13
c. 10:47
d. 11:13

Answer: a. 9:47

Explanation:
Count by fives and ones from the 12 on the clock back to where the minute hand is pointing. Write the missing counting numbers next to the clock.

Question 2.
What time does the clock show?
Go Math Grade 3 Answer Key Chapter 10 Time, Length, Liquid Volume, and Mass Time to the Minute img 7
Options:
a. 2:20
b. 2:40
c. 3:20
d. 4:10

Answer: a. 2:20

Explanation:
Count on by fives and ones from the 12 on the clock to where the minute hand is pointing. Write the missing counting numbers next to the clock.

Spiral Review

Question 3.
Each bird has 2 wings. How many wings will 5 birds have?
Options:
a. 7
b. 8
c. 9
d. 10

Answer: d. 10

Explanation:
STEP 1 Draw 2 counters in each group.
STEP 2 Skip count to find how many wings in all. Skip count by 2s until you say 5 numbers (groups)
There are 5 groups with 2 wings in each group. So, there are 10 wings in all.

Question 4.
Find the unknown factor.
9 × ■ = 36
Options:
a. 4
b. 6
c. 8
d. 27

Answer: a. 4

Explanation:
Count how many rows of 36 counters there are.
There are  9 rows of 36 counters. The unknown factor is 4 (c0lumns). n = 4
9× 4 = 36

Question 5.
Mr. Wren has 56 paintbrushes. He places 8 paintbrushes on each of the tables in the art room. How many tables are in the art room?
Options:
a. 6
b. 7
c. 9
d. 48

Answer: b. 7

Explanation:
STEP 1 Draw 56 counters.
STEP 2 Make a group of 8 counters by drawing a circle around them. Continue circling groups of 8 until all 56 counters are in groups.
There are 7 groups of 56 counters.

Question 6.
Which number completes the equations?
4 × ▲ = 20 20 ÷ 4 = ▲
Options:
a. 4
b. 5
c. 6
d. 16

Answer: b. 5

Explanation:
Count how many rows of 20 counters there are. There are  4 rows of 20 counters. The unknown factor is 5 (c0lumns). n = 5
4× 5 = 20
STEP 1 Draw 20 counters.
STEP 2 Make a group of 4 counters by drawing a circle around them. Continue circling groups of 4 until all 20 counters are in groups.
There are 5 groups of 20 counters.

A.M. and P.M. – Page No. 571

Write the time for the activity. Use A.M. or P.M.

Question 1.
eat lunch
Go Math Grade 3 Answer Key Chapter 10 Time, Length, Liquid Volume, and Mass A.M. and P.M. img 8

Answer: 12:20 P.M.

Explanation:
The times afternoon and before midnight is written with P.M. Since we have our lunch in the afternoon around 12:00 noon to 1:00 P.M. which falls in the period of 12:00 noon to 12:00 at midnight hence the time 12:20 is considered to be P.M.

Question 2.
go home after school
Go Math Grade 3 Answer Key Chapter 10 Time, Length, Liquid Volume, and Mass A.M. and P.M. img 9
_____ : _____ _____ (A.M./P.M.)

Answer: 2:53 P.M.

Explanation:
The times afternoon and before midnight is written with P.M. Since the school hours end in the evening and students return to their home around 2:30 P.M. to 3:30 P.M. which falls in the period of 12:00 noon to 12:00 at midnight hence the time 2:53 is considered to be P.M.

Question 3.
see the sunrise
Go Math Grade 3 Answer Key Chapter 10 Time, Length, Liquid Volume, and Mass A.M. and P.M. img 10
_____ : _____ _____ (A.M./P.M.)

Answer: 6:18 A.M.

Explanation:
The times after midnight and before noon is written with A.M. At the early hours of the day sunrises which falls in the period of  12:00 at midnight to 12:00 noon, therefore the time 6:18 is considered to be A.M.

Question 4.
go for a walk
Go Math Grade 3 Answer Key Chapter 10 Time, Length, Liquid Volume, and Mass A.M. and P.M. img 11
_____ : _____ _____ (A.M./P.M.)

Answer: 3:55 P.M.

Explanation:
The times afternoon and before midnight is written with P.M. Since the people go for a walk to refresh their minds in the evening hours which falls in the period of 12:00 noon to 12:00 at midnight the time 3:55 is considered to be P.M.

Question 5.
go to school
Go Math Grade 3 Answer Key Chapter 10 Time, Length, Liquid Volume, and Mass A.M. and P.M. img 12
_____ : _____ _____ (A.M./P.M.)

Answer: 8:10 A.M.

Explanation:
The times after midnight and before noon is written with A.M. Since students go to the school in the morning hours which falls in the period of  12:00 at midnight to 12:00 noon, therefore, the time 8:10 is considered to be A.M.

Question 6.
get ready for art class
Go Math Grade 3 Answer Key Chapter 10 Time, Length, Liquid Volume, and Mass A.M. and P.M. img 13
_____ : _____ _____ (A.M./P.M.)

Answer: 10:36 A.M.

Explanation:
The times after midnight and before noon is written with A.M. Since students go to the art class in the morning hours which falls in the period of  12:00 at midnight to 12:00 noon, therefore, the time 10:36 is considered to be A.M.

Write the time. Use A.M. or P.M.

Question 7.
13 minutes after 5:00 in the morning
_____ : _____ _____ (A.M./P.M.)

Answer: 5:13 A.M.

Explanation:
The times after midnight and before noon is written with A.M. Since 5:00 in the morning falls in the period of 12:00 at midnight to 12:00 noon, therefore, the time is considered to be A.M.

 

Question 8.
19 minutes before 9:00 at night
_____ : _____ _____ (A.M./P.M.)

Answer: 8:41 P.M.

Explanation:
The times afternoon and before midnight is written with P.M.  Since 8:41 in the night time falls in the period of 12:00 noon to 12:00 at midnight, therefore, the time 8:41 is considered to be P.M.

Question 9.
quarter before midnight
_____ : _____ _____ (A.M./P.M.)

Answer: 11:45P.M.

Explanation:
The times afternoon and before midnight is written with P.M. Since 11:45 in the night time falls in the period of 12:00 noon to 12:00 at midnight, therefore, the time 11:45 is considered to be P.M.

Question 10.
one-half hour after 4:00 in the morning
_____ : _____ _____ (A.M./P.M.)

Answer: 4:30 A.M.

Explanation:
One-half hour = 30 minutes So, by adding 30 minutes to 4:00 A.M. in the morning. We get the time to be 4:30 A.M.
The times after midnight and before noon is written with A.M.  Since 4:30 in the morning falls in the period of 12:00 at midnight to 12:00 noon, therefore, the time is considered to be A.M

Problem Solving

Question 11.
Jaime is in math class. What time is it? Use A.M. or P.M.
Go Math Grade 3 Answer Key Chapter 10 Time, Length, Liquid Volume, and Mass Go Math Grade 3 Answer Key Chapter 10 Time, Length, Liquid Volume, and Mass img 14 img 14
_____ : _____ _____ (A.M./P.M.)

Answer: 1:25 P.M.

Explanation:
The times afternoon and before midnight is written with P.M. Since 1:25 in the afternoon hours fall in the period of 12:00 noon to 12:00 at midnight, therefore, the time1:25 is considered to be P.M.

Question 12.
Pete began practicing his trumpet at fifteen minutes past three. Write this time using A.M. or P.M.
_____ : _____ _____ (A.M./P.M.)

Answer: 3:15 P.M.

Explanation:
The times afternoon and before midnight is written with P.M. Since 3:15 in the evening hours fall in the period of 12:00 noon to 12:00 at midnight, therefore, the time 3:15 is considered to be P.M.

A.M. and P.M. – Page No. 572

Lesson Check

Question 1.
Steven is doing his homework. What time is it? Use A.M. or P.M.
Go Math Grade 3 Answer Key Chapter 10 Time, Length, Liquid Volume, and Mass A.M. and P.M. img 15
Options:
a. 4:15 P.M.
b. 4:25 A.M.
c. 4:35 P.M.
d. 4:35 A.M.

Answer: c. 4:35 P.M.

Explanation:
The times afternoon and before midnight is written with P.M. Since 4:35 in the evening hours fall in the period of 12:00 noon to 12:00 at midnight, therefore, the time 4:35 is considered to be P.M.

Question 2.
After he finished breakfast, Mr. Edwards left for work at fifteen minutes after seven. What time is this? Use A.M. or P.M.
Options:
a. 6:15 A.M.
b. 7:15 A.M.
c. 6:45 P.M.
d. 7:30 P.M.

Answer: b. 7:15 A.M.

Explanation:
The times after midnight and before noon is written with A.M. Since 7:15 in the morning falls in the period of 12:00 at midnight to 12:00 noon, therefore, the time 7:15 is considered to be A.M.

Spiral Review

Question 3.
Which division equation is related to the multiplication equation 4 × 6 = 24
Options:
a. 24 ÷ 8 = 3
b. 12 ÷ 3 = 4
c. 6 × 4 = 24
d. 24 ÷ 4 = 6

Answer: d. 24 ÷ 4 = 6

Explanation:
STEP 1 Draw 24 counters.
STEP 2 Make a group of 4 counters by drawing a circle around them. Continue circling groups of 4 until all 24 counters are in groups.
There are 6 groups of 24 counters.

Question 4.
There are 50 toothpicks in each box. Jaime buys 4 boxes for her party platter. How many toothpicks does Jaime buy in all?
Options:
a. 20
b. 54
c. 200
d. 2,000

Answer: c. 200

Explanation:
Step1 let there be 50 toothpicks in each box.
Step2 the number of boxes required is 4
Step3 total number of toothpicks will be the number of boxes multiplied by the number of toothpicks in each box.
4 × 50=200

Question 5.
A pet store sold 145 bags of beef-flavored dog food and 263 bags of cheese-flavored dog food. How many bags of dog food were sold in all?
Options:
a. 118
b. 308
c. 408
d. 422

Answer: c. 408

Explanation:
Total number of bags of dog food sold=145+263

 

 

 

Question 6.
Victoria and Melody are comparing fraction strips. Which statement is NOT correct?
Options:
a. \(\frac{1}{4}\) < \(\frac{4}{4}\)
b. \(\frac{3}{6}\) > \(\frac{4}{6}\)
c. \(\frac{2}{8}\) > \(\frac{1}{8}\)
d. \(\frac{2}{3}\) > \(\frac{3}{3}\)

Answer: b. \(\frac{3}{6}\) > \(\frac{4}{6}\)

Explanation:

Take two blocks of the same size and both are made of 6 equal-size squares.
3  squares are shaded green while 4 are shaded with red.
Compare 3/6 and 4/6.
The greater fraction will have the larger
amount of the whole shaded.
Therefore, 3/6>4/6.

Measure Time Intervals – Page No. 577

Find the elapsed time.

Question 1.
Start: 8:10 A.M. End: 8:45 A.M.
Go Math Grade 3 Answer Key Chapter 10 Time, Length, Liquid Volume, and Mass Measure Time Intervals img 16

Answer: 35 minutes

Explanation:
STEP 1 Find 8:10 A.M. on the number line. Count on from 8:10 A.M. to 8:45 A.M. Draw marks and record the times on the number line. Then draw and label the jumps.
STEP 2 Add to find the total minutes from 8:10 A.M. to 8:45 A.M. From 8:10 A.M. to 8:45 A.M. is 35 minutes.
So, the elapsed time is 35 minutes

Question 2.
Start: 6:45 P.M. End: 6:54 P.M.
Go Math Grade 3 Answer Key Chapter 10 Time, Length, Liquid Volume, and Mass Measure Time Intervals img 17
_______ minutes

Answer: 9 minutes

Explanation:
STEP 1 Find the starting time on the clock.
STEP 2 Count the minutes by counting on by fives and ones to 6:54 P.M. Write the missing counting numbers next to the clock.
So, the elapsed time is 9 minutes.

Question 3.
Start: 3:00 P.M. End: 3:37 P.M.
Go Math Grade 3 Answer Key Chapter 10 Time, Length, Liquid Volume, and Mass Measure Time Intervals img 18
_______ minutes

Answer: 37 minutes

Explanation:
STEP 1 Find3:00 P.M. on the number line. Count on from 3:00 P.M. to 3:37 P.M. Draw marks and record the times on the number line. Then draw and label the jumps.
STEP 2 Add to find the total minutes from3:00 P.M. to 3:37 P.M. From 3:00 P.M. to 3:37 P.M. is 37 minutes.
So, the elapsed time is 37 minutes

Question 4.
Start: 10:05 A.M. End: 10:21 A.M.
Go Math Grade 3 Answer Key Chapter 10 Time, Length, Liquid Volume, and Mass Measure Time Intervals img 19
_______ minutes

Answer: 16 minutes

Explanation:
STEP 1 Find10:05 A.M. on the number line. Count on from 10:05 A.M. to 10:21 A.M. Draw marks and record the times on the number line. Then draw and label the jumps.
STEP 2 Add to find the total minutes from 10:05 A.M. to 10:21 A.M. From 10:05 A.M. to 10:21 A.M. is 16 minutes.
So, the elapsed time is 16 minutes

Question 5.
Start: 7:30 A.M. End: 7:53 A.M.
Go Math Grade 3 Answer Key Chapter 10 Time, Length, Liquid Volume, and Mass Measure Time Intervals img 20
_______ minutes

Answer: 23 minutes

Explanation:
STEP 1 Find the starting time on the clock.
STEP 2 Count the minutes by counting on by fives and ones to7:53 A.M. Write the missing counting numbers next to the clock.
So, the elapsed time is 23 minutes.

Question 6.
Start: 5:20 A.M. End: 5:47 A.M.
Go Math Grade 3 Answer Key Chapter 10 Time, Length, Liquid Volume, and Mass Measure Time Intervals img 21
_______ minutes

Answer: 27 minutes

Explanation:
STEP 1 Find the starting time on the clock.
STEP 2 Count the minutes by counting on by fives and ones to 5:47 A.M. Write the missing counting numbers next to the clock.
So, the elapsed time is 27 minutes.

Problem Solving

Question 7.
A show at the museum starts at 7:40 P.M. and ends at 7:57 P.M. How long is the show?
_______ minutes

Answer: 17 minutes

Explanation:
STEP 1 Find 7:40 P.M. on the number line. Count on from 7:40 P.M. to 7:57 P.M. Draw marks and record the times on the number line. Then draw and label the jumps.
STEP 2 Add to find the total minutes from 7:40 P.M. to 7:57 P.M. From 7:40 P.M. to 7:57 P.M. is 17 minutes.
So, the elapsed time is 17 minutes
The duration of the show is 17 minutes.

Question 8.
The first train leaves the station at 6:15 A.M. The second train leaves at 6:55 A.M. How much later does the second train leave the station?
_______ minutes

Answer: 40 minutes later

Explanation:
STEP 1 Find 6:15 A .M. on the number line. Count on from 6:15 A.M. to 6:55 A.M. Draw marks and record the times on the number line. Then draw and label the jumps.
STEP 2 Add to find the total minutes from 6:15 A.M. to 6:55 A.M. From 6:15 A .M. to 6:55 A.M. is 40 minutes.
So, the second train will leave the station in 40 minutes

Measure Time Intervals – Page No. 578

Lesson Check

Question 1.
Marcus began playing basketball at 3:30 P.M. and stopped playing at 3:55 P.M. For how many minutes did he play basketball?
Options:
a. 25 minutes
b. 30 minutes
c. 55 minutes
d. 85 minutes

Answer: a. 25 minutes

Explanation:
STEP 1 Find 3:30 P.M. on the number line. Count on from 3:30 P.M. to 3:55 P.M. Draw marks and record the times on the number line. Then draw and label the jumps.
STEP 2 Add to find the total minutes from3:30 P.M. to3:55 P.M. From 3:30 P.M. to 3:55 P.M. is 25 minutes.
So, the elapsed time is 25 minutes
Marcus played basketball for 25 minutes.

Question 2.
The school play started at 8:15 P.M. and ended at 8:56 P.M. How long was the school play?
Options:
a. 15 minutes
b. 31 minutes
c. 41 minutes
d. 56 minutes

Answer: c. 41 minutes

Explanation:
STEP 1 Find 8:15 P.M. on the number line. Count on from 8:15 P.M. to 8:56 P.M. Draw marks and record the times on the number line. Then draw and label the jumps.
STEP 2 Add to find the total minutes from 8:15 P.M. to 8:56 P.M. From 8:15 P.M. to 8:56 P.M. is 41  minutes.
So, the elapsed time is 41 minutes
Therefore, the school play was for 41 minutes.

Spiral Review

Question 3.
Each car has 4 wheels. How many wheels will 7 cars have?
Options:
a. 11
b. 24
c. 27
d. 28

Answer: d. 28

Explanation:
STEP 1 Draw 4 counters in each group.
STEP 2 Skip count to find how many wheels in all. Skip count by 4s until you say 7 numbers (groups)
There are 7 groups with 4 wheels in each group. So, there are 28 wheels in all.

Question 4.
Which number completes the equations?
3 × ■ = 27 27 ÷ 3 = ■
Options:
a. 6
b. 7
c. 8
d. 9

Answer: d. 9

Explanation:
Count how many rows of 27 counters there are. There are  3 rows of 27 counters. The unknown factor is 9 (c0lumns). n = 9
3 × 9 = 27
STEP 1 Draw 27 counters.
STEP 2 Make a group of 3 counters by drawing a circle around them. Continue circling groups of 3 until all 27 counters are in groups.
There are 9 groups of 27 counters.

Question 5.
There are 20 napkins in each package. Kelli bought 8 packages for her party. How many napkins did Kelli buy in all?
Options:
a. 28
b. 40
c. 160
d. 180

Answer: c. 160

Explanation:
Step1 let there be 20 napkins in each package.
Step2 the number of packages required is 8
Step3 total number of napkins will be the number of packages multiplied by the number of napkins in each package.
8 × 20=160

Question 6.
Mr. Martin drove 290 miles last week. This week he drove 125 miles more than last week. How many miles did Mr. Martin drive this week?
Options:
a. 125 miles
b. 165 miles
c. 315 miles
d. 415 miles

Answer: d. 165 miles

Explanation:
Total number of miles drove by Mr Martin in this week = 290+125

Use Time Intervals – Page No. 583

Find the starting time.

Question 1.
Ending time: 4:29 P.M.
Elapsed time: 55 minutes
Go Math Grade 3 Answer Key Chapter 10 Time, Length, Liquid Volume, and Mass Use Time Intervals img 22

Answer: 3:34 P.M.

Explanation:
STEP 1 Find the starting time on the number line
STEP 2 Count back on the number line to subtract the elapsed time. Draw and label the jumps to show the minutes.
STEP 3 Write the times below the number line.
You jumped back to 55 minutes
So, the starting time is 3:34 P.M.

Question 2.
Ending time: 10:08 A.M.
Elapsed time: 30 minutes
Go Math Grade 3 Answer Key Chapter 10 Time, Length, Liquid Volume, and Mass Use Time Intervals img 23
______ : ______ ______ (A.M./P.M.)

Find the ending time.

Answer: 9:38 A.M.

Explanation:
STEP 1 Find the starting time on the clock.
STEP 2 Count back by fives for the elapsed time of 3o minutes. Write the missing counting numbers next to the clock.
So, the starting time is 9:38 A.M.

Question 3.
Starting time: 2:15 A.M.
Elapsed time: 45 minutes
Go Math Grade 3 Answer Key Chapter 10 Time, Length, Liquid Volume, and Mass Use Time Intervals img 24
______ : ______ ______ (A.M./P.M.)

Answer: 3:00 A.M.

Explanation:
STEP 1 Find the ending time on the number line.
STEP 2 Count forward on the number line to add the elapsed time. Draw and label the jumps to show the minutes.
STEP 3 Write the times below the number line.
The jumps end at 3:00 A.M.
So, the ending time is 3:00 A.M.

Question 4.
Starting time: 6:57 P.M.
Elapsed time: 47 minutes
Go Math Grade 3 Answer Key Chapter 10 Time, Length, Liquid Volume, and Mass Use Time Intervals img 25
______ : ______ ______ (A.M./P.M.)

Answer: 7:44 P.M.

Explanation:
STEP 1 Find the ending time on the clock.
STEP 2 Count on by fives and ones for the elapsed time of 47 minutes. Write the missing counting numbers next to the clock.
So, the ending time is 7:44 P.M.

Problem Solving

Question 5.
Jenny spent 35 minutes doing research on the Internet. She finished at 7:10 P.M. At what time did Jenny start her research?
______ : ______ ______ (A.M./P.M.)

Answer: 6:35 P.M.

Explanation:
STEP 1 Find the starting time on the clock.
STEP 2 Count back by fives for the elapsed time of 35 minutes. Write the missing counting numbers next to the clock.
So, the starting time is 6:35 A.M.

Question 6.
Clark left for school at 7:43 A.M. He got to school 36 minutes later. At what time did Clark get to school?
______ : ______ ______ (A.M./P.M.)

Answer: 8:19 A.M.

Explanation:
STEP 1 Find the ending time on the clock.
STEP 2 Count on by fives and ones for the elapsed time of 36 minutes. Write the missing counting numbers next to the clock.
So, the ending time is 8:19 A.M.

Use Time Intervals – Page No. 584

Lesson Check

Question 1.
Cody and his friends started playing a game at 6:30 P.M. It took them 37 minutes to finish the game. At what time did they finish?
Options:
a. 5:07 P.M.
b. 5:53 P.M.
c. 6:53 P.M.
d. 7:07 P.M.

Answer: d. 7:07 P.M.

Explanation:
STEP 1 Find the ending time on the clock.
STEP 2 Count on by fives and ones for the elapsed time of 37 minutes. Write the missing counting numbers next to the clock.
So, the ending time is 7:07 P.M.

Question 2.
Delia worked for 45 minutes on her oil painting. She took a break at 10:35 A.M. At what time did Delia start working on the painting?
Options:
a. 9:40 A.M.
b. 9:50 A.M.
c. 11:20 A.M.
d. 11:30 A.M.

Answer: b. 9:50 A.M.

Explanation:
STEP 1 Find the starting time on the clock.
STEP 2 Count back by fives for the elapsed time of 45 minutes. Write the missing counting numbers next to the clock.
So, the starting time is 9:50 A.M.

Spiral Review

Question 3.
Sierra has 30 collector’s pins. She wants to put an equal number of pins in each of 5 boxes. How many pins should she put in each box?
Go Math Grade 3 Answer Key Chapter 10 Time, Length, Liquid Volume, and Mass Use Time Intervals img 26
Options:
a. 4
b. 5
c. 6
d. 8

Answer: c. 6

Explanation:
Use 30 counters.
Take 5 boxes.
Place 1 counter at a time in each box until all 30 counters are used.
Place the rest of the counters till all the 30 counters (pins) are completed.
Total number of counters in each box is 6.

Question 4.
What time is shown on the clock?
Go Math Grade 3 Answer Key Chapter 10 Time, Length, Liquid Volume, and Mass Use Time Intervals img 27
Options:
a. 1:24
b. 2:24
c. 4:12
d. 5:12

Answer: b. 2:24

Explanation:
Count on by fives and ones from the 12 on the clock to where the minute hand is pointing. Write the missing counting numbers next to the clock.
Write: 2:24
Read:
• twenty-four minutes after two

Question 5.
Ricardo has 32 books to put on 4 shelves. He puts the same number of books on each shelf. How many books does Ricardo put on each shelf?
Options:
a. 6
b. 7
c. 8
d. 9

Answer: c. 8

Explanation:
Use 32 counters. (books)
The number of shelves is 4.
Place 1 counter at a time in each shelf until all 32 counters are used.
Place the rest of the counters till all the 32 counters (books) are completed.
Total number of counters in each shelf is 8.

Question 6.
Jon started playing a computer game at 5:35 P.M. He finished the game at 5:52 P.M. How long did Jon play the game?
Options:
a. 17 minutes
b. 23 minutes
c. 25 minutes
d. 27 minutes

Answer: a. 17 minutes

Explanation:
STEP 1 Find the ending time on the clock.
STEP 2 Count the minutes by counting on by fives and ones to 5:52 P.M. Write the missing counting numbers next to
the clock.
So, Jon has spent 17 minutes on the computer playing a game.

Lesson 5 – Page No. 587

Share and Show

Question 1.
Patty went to the shopping mall at 11:30 a.m. She shopped for 25 minutes. She spent 40 minutes eating lunch.
Then she met a friend at a movie.
At what time did Patty meet her friend?
_______ : ________

Answer: 12:35 P.M.

Explanation:
The times afternoon and before midnight is written with P.M.
Count (25+40) minutes after 11:30 A.M.
Therefore the time at which Patty met her friend is 11:30 A.M.+1:05 hrs = 12:35 P.M.

Question 2.
What if Patty goes to the mall at 11:30 A.M. and meets a friend at a movie at 1:15 P.M.?
Patty wants to shop and have 45 minutes for lunch before meeting her friend.
How much time can Patty spend shopping?
_______ minutes

Answer: 60 minutes

Explanation:
The times afternoon and before midnight is written with P.M.
Count 45 minutes before 1:15 P.M. (for lunch) which is equal to 12:30 P.M.
Difference between 12:30 P.M. and 11:30 A.M. gives the time spent by Patty in shopping.
STEP 1 Find 12:30 P.M. on the number line. Count on from 12:30 P.M. to 11:30 A.M. Draw marks and record the times on the number line. Then draw and label the jumps.
STEP 2 Add to find the total minutes from 12:30 P.M. to 11:30 A.M. From 12:30 P.M. to 11:30 A.M. is 60 minutes.
So, the time spent on shopping is 60 minutes

Question 3.
Avery got on the bus at 1:10 p.m. The trip took 90 minutes. Then she walked for 32 minutes to get home. At what time did Avery arrive at home?
_______ : ________ P.M.

Answer: 3:12 P.M.

Explanation:
STEP 1 Find 1:10 P.M. on the number line. Count on from 1:10 P.M.
STEP2 Time spent in the trip= 90 minutes
Time Avery walked to get home = 32 minutes
STEP3 Then draw and label the jumps on the number line.
Add the total minutes; 90+32=122 minutes
So the time at which Avery arrive at home is calculated by the cumulative jumps on the number line which is equal to 3:12 P.M.
Question 4.
Kyle and Josh have a total of 64 CDs. Kyle has 12 more CDs than Josh. How many CDs does each boy have?
Josh _______ CDs
Kyle _______ CDs

Answer: Josh has 26 CDs
Kyle has 38 CDs

Explanation:
STEP 1 There are 64 counters. (CDs)
STEP2 Kyle has 12 CDs more than Josh. Place these 12 in a separate group. The leftover number of CDs is 64-12=52
STEP 3 Make 2 group and start placing the counters one after the other in each group until all the 52 counters are grouped
There are 2 groups of 26 counters.
Josh has 26 CDs while Kyle has 26+12 = 38 CDs

Lesson 5 – Page No. 588

Question 5.
Jamal spent 60 minutes using the computer. He spent a half hour of the time playing games and the rest of the time researching his report. How many minutes did Jamal spend researching his report?
_______ minutes

Answer: 30 minutes

Explanation: Jamal has spent half hour=30 minutes playing games and the rest of the time researching for his report.
STEP1 There are 60 counters(minutes).
STEP2 Remove the 30 counters which were used to play games
STEP3 The remaining number of counters in the box is 30
So the time spent on the report =30 minutes

Question 6.
When Caleb got home from school, he worked on his science project for 20 minutes. Then he studied for a test for 30 minutes. He finished at 4:35 p.m. At what time did Caleb get home from school?
_______ : _______ P.M.

Answer: 3:45 P.M

Explanation:
STEP 1 Find the time on the number line when Caleb finished his homework.
STEP 2 Count back on the number line to subtract the elapsed time. Draw and label the jumps to show the minutes.
STEP 3 Write the times below the number line.
You jumped back to 30+20=50 minutes
Therefore Caleb got home from school at 3:45 P.M.

Question 7.
Miguel played video games each day for a week. On Monday, he scored 83 points. His score went up 5 points each day. On what day did Miguel score 103 points? Explain how you found your answer
_____________

Answer: 5 days

Explanation:
STEP1 Miguel scored 83 points on Monday. His score went up 5 points each day.
STEP2 Add 5n counters to the points to reach 103
STEP3 Adding 5 points 5 times gives us the score to be 103 (83+5*5)
STEP4 Therefore Miguel takes 5 days to score 103 points

Question 8.
When Laura arrived at the library, she spent 40 minutes reading a book. Then she spent 15 minutes reading a magazine. She left the library at 4:15 p.m. Circle the time that makes the sentence true.
Laura arrived at the library at:
_______ : _______ P.M.

Answer: 3:20 P.M.

Explanation:
STEP 1 Find the time on the number line when Laura arrived at the library.
STEP 2 Count back on the number line to subtract the elapsed time. Draw and label the jumps to show the minutes.
STEP 3 Write the times below the number line.
You jumped back to 40+15=55 minutes
Therefore Laura arrived at the library at 3:20 P.M.

Problem Solving Time Intervals – Page No. 589

Solve each problem. Show your work.

Question 1.
Hannah wants to meet her friends downtown. Before leaving home, she does chores for 60 minutes and eats lunch for 20 minutes. The walk downtown takes 15 minutes. Hannah starts her chores at 11:45 A.M. At what time does she meet her friends?
Go Math Grade 3 Answer Key Chapter 10 Time, Length, Liquid Volume, and Mass Problem Solving Time Intervals img 28

Answer: 1:20 P.M.

Explanation:
STEP 1 Find the ending time on the clock, the time at which Hannah meets her friends.
STEP 2 Count back by fives for the elapsed time of 95 minutes. Write the missing counting numbers next to the clock.
So, the ending time is 1:20 P.M.

Question 2.
Katie practiced the flute for 45 minutes. Then she ate a snack for 15 minutes. Next, she watched television for 30 minutes, until 6:00 P.M. At what time did Katie start practicing the flute?
_______ : ________ (A.M./P.M.)

Answer: 4:30 P.M.

Explanation:
STEP 1 Find the starting time on the clock.
STEP 2 Count back by fives for the elapsed time of 90 minutes. Write the missing counting numbers next to the clock.
So, the starting time is 4:30 P.M.

Question 3.
Nick gets out of school at 2:25 P.M. He has a 15-minute ride home on the bus. Next, he goes on a 30-minute bike ride. Then he spends 55 minutes doing homework. At what time does Nick finish his homework?
_______ : ________ (A.M./P.M.)

Answer: 4:05 P.M.

Explanation:
STEP 1 Find the ending time on the clock, the time at which Nick finish his work.
STEP 2 Count back by fives for the elapsed time of 100 minutes. Write the missing counting numbers next to the clock.
So, the ending time is 4:05 P.M.

Question 4.
The third-grade class is going on a field trip by bus to the museum. The bus leaves the school at 9:45 A.M. The bus ride takes 47 minutes. At what time does the bus arrive at the museum?
_______ : ________ (A.M./P.M.)

Answer: 10:32 A.M.

Explanation:
STEP 1 Find the ending time on the clock, the time at which the students arrive at the museum.
STEP 2 Count back by fives for the elapsed time of 47 minutes. Write the missing counting numbers next to the clock.
So, the ending time is 10:32 A.M.

Problem Solving Time Intervals – Page No. 590

Lesson Check

Question 1.
Gloria went to the mall and spent 50 minutes shopping. Then she had lunch for 30 minutes. If Gloria arrived at the mall at 11:00 A.M., at what time did she finish lunch?
Options:
a. 11:30 A.M.
b. 11:50 A.M.
c. 12:20 P.M.
d. 12:30 P.M.

Answer: c. 12:20 P.M.

Explanation:
STEP 1 Find the ending time on the clock, the time at which Gloria finish her lunch.
STEP 2 Count back by fives for the elapsed time of 80 minutes. Write the missing counting numbers next to the clock.
So, the ending time is 12:20 P.M.

Question 2.
The ball game begins at 2:00 P.M. It takes Ying 30 minutes to get to the ballpark. At what time should Ying leave home to get to the game 30 minutes before it starts?
Options:
a. 12:30 P.M.
b. 1:00 P.M.
c. 1:30 P.M.
d. 3:00 P.M.

Answer: b. 1:00 P.M.

Explanation:
STEP 1 Find the starting time on the clock.
STEP 2 Count back by fives for the elapsed time of 60 minutes. Write the missing counting numbers next to the clock.
So, the starting time is 1:00 P.M.

Spiral Review

Question 3.
Which lists the fractions in order from least to greatest?
Options:
a. \(\frac{2}{8}\), \(\frac{2}{4}\), \(\frac{2}{6}\)
b. \(\frac{2}{4}\), \(\frac{2}{8}\), \(\frac{2}{6}\)
c. \(\frac{2}{8}\), \(\frac{2}{6}\), \(\frac{2}{4}\)
d. \(\frac{2}{4}\), \(\frac{2}{6}\), \(\frac{2}{8}\)

Answer: c. \(\frac{2}{8}\), \(\frac{2}{6}\), \(\frac{2}{4}\)

Explanation:
When the numerators are the same, think about the  pieces to compare and order fractions. So, the order from least to greatest  is \(\frac{2}{8}\), \(\frac{2}{6}\), \(\frac{2}{4}\)

Question 4.
Find the unknown factor.
6 × ■ = 36
Options:
a. 4
b. 6
c. 7
d. 8

Answer: b. 6

Explanation:
Count how many rows of 36 counters there are.
There are  6 rows of 36 counters. The unknown factor is 6 (c0lumns). n = 6
6× 6 = 36

Question 5.
There were 405 books on the library shelf. Some books were checked out. Now there are 215 books left on the shelf. How many books were checked out?
Options:
a. 620
b. 220
c. 210
d. 190

Answer: d. 190

Explanation:
STEP1 Let there be 405 books in a shelf of Library.
STEP2 Some books were checked out.
STEP3 Number of books left in the shelf are 215.
STEP4 Take a box filled with 405 counters.
STEP5 Place 215 counters in another box from the initial 405 counters.
STEP6 The number of counters left in the first box is equal to the books checked out.
405-215=190

Question 6.
Savannah has 48 photos. She places 8 photos on each page of her photo album. How many pages in the album does she use?
Options:
a. 5
b. 6
c. 7
d. 9

Answer: b. 6

Explanation:
STEP 1 Draw 48 counters.
STEP 2 Make a group of 8 counters by drawing a circle around them. Continue circling groups of 8 until all 48 counters are in groups.
There are 6 groups of 48 counters.

Mid -Chapter Checkpoint – Page No. 591

Vocabulary

Choose the best term from the box.
Go Math Grade 3 Answer Key Chapter 10 Time, Length, Liquid Volume, and Mass Mid -Chapter Checkpoint img 29

Question 1.
In one __________, the minute hand moves from one mark to the next on a clock.

Answer: Minute

Explanation: The minute hand moves after every 60 seconds = 1 Minute to the next mark on the clock.

Question 2.
The times afternoon and before midnight are written with __________ .
__________

Answer: P.M.

Explanation:
The times afternoon and before midnight are written with P.M.

Concepts and Skills

Write the time for the activity. Use A.M. or P.M.

Question 3.
play ball
Go Math Grade 3 Answer Key Chapter 10 Time, Length, Liquid Volume, and Mass Mid -Chapter Checkpoint img 30
______ : ______ ______ (A.M./P.M.)

Answer: 4:30 P.M.

Explanation:
The times afternoon and before midnight is written with P.M. Since we play ball in the evening, the time is considered to be P.M.

Question 4.
eat breakfast
Go Math Grade 3 Answer Key Chapter 10 Time, Length, Liquid Volume, and Mass Mid -Chapter Checkpoint img 31
______ : ______ ______ (A.M./P.M.)

Answer: 7:06 A.M.

Explanation:
The times after midnight and before noon is written with A.M. Since we have our breakfast in the morning, the time is considered to be  A.M.

Question 5.
do homework
Go Math Grade 3 Answer Key Chapter 10 Time, Length, Liquid Volume, and Mass Mid -Chapter Checkpoint img 32
______ : ______ ______ (A.M./P.M.)

Answer: 5:45 P.M.

Explanation:
The times afternoon and before midnight is written with P.M. Since we do our homework in the evening, the time is considered to be P.M.

Question 6.
sleep
Go Math Grade 3 Answer Key Chapter 10 Time, Length, Liquid Volume, and Mass Mid -Chapter Checkpoint img 33
______ : ______ ______ (A.M./P.M.)

Answer: 11:06 P.M.

Explanation:
The times afternoon and before midnight is written with P.M. Since we go to sleep in the night hours, the time is considered to be P.M.

Find the elapsed time.

Question 7.
Start: 10:05 A.M. End: 10:50 A.M.
Go Math Grade 3 Answer Key Chapter 10 Time, Length, Liquid Volume, and Mass Mid -Chapter Checkpoint img 34
_______ minutes

Answer: 45 minutes

Explanation:
STEP 1 Find10:05 A.M. on the number line. Count on from 10:05 A.M. to 10:50 A.M. Draw marks and record the times on the number line. Then draw and label the jumps.
STEP 2 Add to find the total minutes from 10:05 A.M. to 10:50 A.M. From 10:05 A.M. to 10:50 A.M. is 45 minutes.
So, the elapsed time is 45 minutes

Question 8.
Start: 5:30 P.M.
End: 5:49 P.M.
Go Math Grade 3 Answer Key Chapter 10 Time, Length, Liquid Volume, and Mass Mid -Chapter Checkpoint img 35
_______ minutes

Answer: 19 minutes

Explanation:
STEP 1 Find the starting time on the clock that is 5:30 P.M.
STEP 2 Count the minutes by counting on by fives and ones to 5:49 P.M. Write the missing counting numbers next to the clock.
So, the elapsed time is 19 minutes.

Find the starting time or the ending time.

Question 9.
Elapsed time: 50 minutes
Ending time: 9:05 A.M.
Go Math Grade 3 Answer Key Chapter 10 Time, Length, Liquid Volume, and Mass Mid -Chapter Checkpoint img 36
Starting time: ______ : ______ A.M.

Answer: 8:15 A.M.

Explanation:
STEP 1 Find the starting time on the number line
STEP 2 Count back on the number line to subtract the elapsed time. Draw and label the jumps to show the minutes.
STEP 3 Write the times below the number line.
You jumped back to 50 minutes
So, the starting time is 8:15 A.M.

Question 10.
Starting time: 2:46 P.M.
Elapsed time: 15 minutes
Go Math Grade 3 Answer Key Chapter 10 Time, Length, Liquid Volume, and Mass Mid -Chapter Checkpoint img 37
Ending time: ______ : ______ P.M.

Answer: 3:01 P.M.

Explanation:
STEP 1 Find the ending time on the clock
STEP 2 Count back by fives for the elapsed time of 15 minutes. Write the missing counting numbers next to the clock.
So, the ending time is 3:01 P.M.

Mid -Chapter Checkpoint – Page No. 592

Question 11.
Veronica started walking to school at 7:45 A.M. She 0 arrived at school 23 minutes later. At what time did Veronica arrive at school?
______ : ______ ______

Answer: 8:08 A.M.

Explanation:
STEP 1 Find the ending time on the number line.
STEP 2 Count forward on the number line to add the elapsed time. Draw and label the jumps to show the minutes.
STEP 3 Write the times below the number line.
The jumps end at 8:08 A.M.
So, the ending time is 8:08 A.M.

Question 12.
The clock shows the time the art class ends. At what time does it end? If the class started 37 minutes before the time shown, at what time did the class start?
Go Math Grade 3 Answer Key Chapter 10 Time, Length, Liquid Volume, and Mass Mid -Chapter Checkpoint img 38
Type below:
____________

Answer: Time at which the class ends= 1:57 P.M.
Time at which the class starts= 1:20 P.M.

Explanation:
Count by fives and ones from the 12 on the clock back to where the minute hand is pointing. Write the missing counting numbers next to the clock.
Write: 1:57 P.M.
Read:
Three minutes before two
STEP 1 Find the starting time on the clock.
STEP 2 Count back by fives for the elapsed time of 37 minutes. Write the missing counting numbers next to the clock.
So, the starting time is 1:20 P.M.

Question 13.
Matt went to his friend’s house. He arrived at 5:10 P.M. He left at 5:37 P.M. How long was Matt at his friend’s house?
______ minutes

Answer: 27 minutes

Explanation:
STEP 1 Find 5:10 P.M. on the number line. Count on from 5:10 P.M. to 5:37 P.M. Draw marks and record the times on the number line. Then draw and label the jumps.
STEP 2 Add to find the total minutes from 5:10 P.M. to 5:37 P.M. From 5:10 P.M. to 5:37 P.M. is 27 minutes.
So, the elapsed time is 27 minutes

Question 14.
Brenda’s train leaves at 7:30 A.M. She needs to arrive 10 minutes early to buy her ticket. It takes her 20 minutes to get to the train station. At what time should Brenda leave her house?
______ : ______ A.M.

Answer: 7:00 A.M.

Explanation:
STEP 1 Find the starting time on the number line
STEP 2 Count back on the number line to subtract the elapsed time. Draw and label the jumps to show the minutes.
STEP 3 Write the times below the number line.
You jumped back to 30 minutes
So, the starting time is 7:00 A.M.

Question 15.
Write the time you get home from school.
Type below:
____________

Answer: 4:00 P.M

Explanation:
The times afternoon and before midnight is written with P.M. Since we get home from school in the evening, the time is considered to be P.M.

Measure Length – Page No. 597

Measure the length to the nearest half inch.

Question 1.
Go Math Grade 3 Answer Key Chapter 10 Time, Length, Liquid Volume, and Mass Measure Length img 39

Answer: 1 1/2 inches

Explanation:
Line up the left end of the glue stick with the zero mark on the ruler.
The right end of the glue stick is between the half-inch marks for 1 and 2.
The mark that is closest to the right end of the glue stick is for one and a half inches.
So, the length of the glue stick to the nearest half-inch is 1 1/2 inches.

Question 2.
Go Math Grade 3 Answer Key Chapter 10 Time, Length, Liquid Volume, and Mass Measure Length img 40
______ inches

Answer: 3

Explanation:
Line up the left end of the glue stick with the zero mark on the ruler.
The right end of the glue stick is between the half-inch marks for 1 and 3.
The mark that is closest to the right end of the glue stick is for three inches.
So, the length of the glue stick to the nearest half-inch is 3 inches.

Question 3.
Go Math Grade 3 Answer Key Chapter 10 Time, Length, Liquid Volume, and Mass Measure Length img 41
______ \(\frac{□}{□}\) inches

Answer: 4 1/2 inches

Explanation:
Line up the left end of the glue stick with the zero mark on the ruler.
The right end of the glue stick is between the half-inch marks for4 and 5.
The mark that is closest to the right end of the glue stick is for four and a half  inches.
So, the length of the glue stick to the nearest half-inch is 4 1/2 inches.

Measure the length to the nearest fourth inch.

Question 4.
Go Math Grade 3 Answer Key Chapter 10 Time, Length, Liquid Volume, and Mass Measure Length img 42
______ \(\frac{□}{□}\) inches

Answer: 1 1/4 inches

Explanation:
Line up the left end of the paper clip with the zero mark on the ruler.
The right end of the paper clip is between the fourth-inch marks for 1 and2.
The mark that is closest to the right end of the paper clip is for 1 1/4 inches.
So, the length of the paper clip to the nearest fourth inch is 1 1/4 inches.

Question 5.
Go Math Grade 3 Answer Key Chapter 10 Time, Length, Liquid Volume, and Mass Measure Length img 43
______ \(\frac{□}{□}\) inches

Answer: 2 3/4 inches

Explanation:
Line up the left end of the paper clip with the zero mark on the ruler.
The right end of the paper clip is between the fourth-inch marks for 2 and 3.
The mark that is closest to the right end of the paper clip is for 2 3/4 inches.
So, the length of the paper clip to the nearest fourth inch is 2 3/4 inches.

Question 6.
Go Math Grade 3 Answer Key Chapter 10 Time, Length, Liquid Volume, and Mass Measure Length img 44
\(\frac{□}{□}\) inch

Answer: 3/4 inches

Explanation:
Line up the left end of the paper clip with the zero mark on the ruler.
The right end of the paper clip is between the fourth-inch marks for 0 and1.
The mark that is closest to the right end of the paper clip is for 3/4 inches.
So, the length of the paper clip to the nearest fourth inch is 3/4 inches.

Question 7.
Go Math Grade 3 Answer Key Chapter 10 Time, Length, Liquid Volume, and Mass Measure Length img 45

______ inches

Answer: 2 inches

Explanation:
Line up the left end of the paper clip with the zero mark on the ruler.
The right end of the paper clip is between the fourth-inch marks for 1 and2.
The mark that is closest to the right end of the paper clip is for 2 inches.
So, the length of the paper clip to the nearest fourth inch is 2 inches.

Problem Solving

Use a separate sheet of paper for 8.

Question 8.
Draw 8 lines that are between 1 inch and 3 inches long. Measure each line to the nearest fourth inch, and make a line plot.

Answer:

Explanation:

Question 9.
The tail on Alex’s dog is 5\(\frac{1}{4}\) inches long. This length is between which two inch-marks on a ruler?
Between ______ and ______ inch-marks

Answer: Between 5 and 6 inches

Explanation:
Line up the left end of the paper clip with the zero mark on the ruler.
The right end of the paper clip is between the fourth-inch marks for 5 and 6.
The mark that is closest to the right end of the paper clip is for 5 1/4 inches.
So, the length of the paper clip to the nearest fourth inch is 5 1/4 inches.

Measure Length – Page No. 598

Lesson Check

Question 1.
What is the length of the eraser to the nearest half inch?
Go Math Grade 3 Answer Key Chapter 10 Time, Length, Liquid Volume, and Mass Measure Length img 46
Options:
a. \(\frac{1}{2}\) inch
b. 1 inch
c. 1 \(\frac{1}{2}\) inch
d. 2 inch

Answer: c. 1 \(\frac{1}{2}\) inch

Explanation:
Line up the left end of the glue stick with the zero mark on the ruler.
The right end of the glue stick is between the half-inch marks for 1 and 2.
The mark that is closest to the right end of the glue stick is for one and a half inches.
So, the length of the glue stick to the nearest half-inch is 1 1/2 inches.

Question 2.
What is the length of the leaf to the nearest fourth inch?
Go Math Grade 3 Answer Key Chapter 10 Time, Length, Liquid Volume, and Mass Measure Length img 47
Options:
a. 1 \(\frac{1}{2}\) inches
b. 1 \(\frac{3}{4}\) inches
c. 2 inches
d. 2 \(\frac{1}{4}\) inches

Answer: c. 2 inches

Explanation:
Line up the left end of the paper clip with the zero mark on the ruler.
The right end of the paper clip is between the fourth-inch marks for 1 and 2.
The mark that is closest to the right end of the paper clip is for 2 inches.
So, the length of the paper clip to the nearest fourth inch is 2 inches.

Spiral Review

Question 3.
Which equation is NOT included in the same set of related facts as 6 × 8 = 48?
Options:
a. 8 × 6 = 48
b. 8 × 8 = 64
c. 48 ÷ 6 = 8
d. 48 × 8 = 6

Answer: b. 8 × 8 = 64

Explanation:
STEP 1 Use 8 tiles to make an array with 6 equal rows.
Draw the rest of the tiles.
There are 8 tiles in each row.
Write a division equation for the array using the total number of tiles as the dividend and the number of rows as the divisor.
48÷ 6= 8
So, 48 ÷ 8 = 6, 8 × 6 = 48,
and 6 ×8  = 48 are related facts.

Question 4.
Brooke says there are 49 days until July 4. There are 7 days in a week. In how many weeks will it be July 4?
Options:
a. 9 weeks
b. 8 weeks
c. 7 weeks
d. 6 weeks

Answer: c. 7 weeks

Explanation:
Draw 1 tile in each of 7 rows.
Continue drawing 1 tile in each of the 7 rows until all 49 tiles are drawn.
Count the number of tiles in each row.
There are 7 tiles in each row.
So, there are 7 weeks till July 4.

Question 5.
It is 20 minutes before 8:00 in the morning. Which is the correct way to write that time?
Options:
a. 7:40 A.M.
b. 7:40 P.M.
c. 8:20 A.M.
d. 8:40 A.M.

Answer: a. 7:40 A.M.

Explanation:
Count by fives and ones from the 12 on the clock back to where the minute hand is pointing. Write the missing counting numbers next to the clock.
Write: 7:40 A.M.
Read:
Twenty minutes before eight

Question 6.
Marcy played the piano for 45 minutes. She stopped playing at 4:15 P.M. At what time did she start playing the piano?
Options:
a. 3:00 P.M.
b. 3:30 P.M.
c. 4:45 P.M.
d. 5:00 P.M.

Answer: b. 3:30 P.M.

Explanation:
STEP 1 Find the starting time on the number line
STEP 2 Count back on the number line to subtract the elapsed time. Draw and label the jumps to show the minutes.
STEP 3 Write the times below the number line.
You jumped back to 45 minutes
So, the starting time is 3:30 P.M.

Estimate and Measure Liquid Volume – Page No. 603

Estimate how much liquid volume there will be when the container is filled. Write more than 1 liter, about 1 liter, or less than 1 liter.

Question 1.
large milk container
Go Math Grade 3 Answer Key Chapter 10 Time, Length, Liquid Volume, and Mass Estimate and Measure Liquid Volume img 48

Answer: more than one liter

Explanation:
Pour 1 liter of milk into one of the large containers. Repeat until the container is full. Record the number of liters you poured. This proves that the container can hold more than one liter.

Question 2.
small milk container
Go Math Grade 3 Answer Key Chapter 10 Time, Length, Liquid Volume, and Mass Estimate and Measure Liquid Volume img 49
____________

Answer: less than one liter

Explanation:
Pour 1 liter of milk into one of the small containers. There is some quantity of milk leftover in the pack of one liter, this proves that the container can hold less than one liter.

Question 3.
water bottle
Go Math Grade 3 Answer Key Chapter 10 Time, Length, Liquid Volume, and Mass Estimate and Measure Liquid Volume img 50
____________

Answer: about one liter

Explanation:
A water bottle holds about 1 liter.

Question 4.
spoonful of water
Go Math Grade 3 Answer Key Chapter 10 Time, Length, Liquid Volume, and Mass Estimate and Measure Liquid Volume img 51
____________

Answer: less than one liter

Explanation:
Pour 100 ml of water into a spoon. The excess water flow out, this proves that the spoon can hold less than one liter.

Question 5.
bathtub filled halfway
Go Math Grade 3 Answer Key Chapter 10 Time, Length, Liquid Volume, and Mass Estimate and Measure Liquid Volume img 52
____________

Answer: more than one liter

Explanation:
Pour 1 liter of water into the bathtub. Repeat until the tub is full. Record the number of liters you poured.This proves that the container can hold more than one liter.

Question 6.
filled eyedropper
Go Math Grade 3 Answer Key Chapter 10 Time, Length, Liquid Volume, and Mass Estimate and Measure Liquid Volume img 53
____________

Answer: less than one liter

Explanation:
Place the eyedropper in a bottle of the quantity, one liter. The eyedropper picks only few drops of the water, this proves that the eyedropper can hold less than one liter.

Problem Solving

Use the pictures for 7–8. Alan pours water into four glasses that are the same size.
Go Math Grade 3 Answer Key Chapter 10 Time, Length, Liquid Volume, and Mass Estimate and Measure Liquid Volume img 54

Question 7.
Which glass has the most amount of water?
_______

Answer: Glass D

Explanation:
It can be observed that glass D is almost completely filled. So, glass D most amount of water.

Question 8.
Which glass has the least amount of water?
_______

Answer: Glass A

Explanation:
It can be observed that glass A contains the least amount of water. So, glass A has the least amount of water.

Estimate and Measure Liquid Volume – Page No. 604

Lesson Check

Question 1.
Felicia filled the bathroom sink with water. About how much water does she put in the sink?
Options:
a. about 1 liter
b. more than 1 liter
c. a little less than 1 liter
d. much less than 1 liter

Answer: b. more than 1 liter

Explanation:
Pour one liter of water into the sink. By repeating the same process we can say that, a sink can hold more than one liter of water. Hence it contains more than one liter of water.

Question 2.
Kyle needed about 1 liter of water to fill a container. Which container did Kyle most likely fill?
Options:
a. a small glass
b. a spoon
c. a large pail
d. a vase

Answer: d. a vase

Explanation:
A vase holds about 1 liter while a small glass, a spoon can only hold small quantity of water. A large pail holds water more than one liter.

Spiral Review

Question 3.
Cecil had 6 ice cubes. He put 1 ice cube in each glass. In how many glasses did Cecil put ice cubes?
Options:
a. 6
b. 5
c. 1
d. 0

Answer: a. 6

Explanation:
If there is the same number of ice cubes and glasses, then 1 ice cube goes in each glass. Since each ice cube is to be placed in each glass.
6÷ 1 = 6
Number of glasses=6

Question 4.
Juan has 12 muffins. He puts \(\frac{1}{4}\) of the muffins in a bag. How many muffins does Juan put in the bag?
Go Math Grade 3 Answer Key Chapter 10 Time, Length, Liquid Volume, and Mass Estimate and Measure Liquid Volume img 55
Options:
a. 2
b. 3
c. 4
d. 5

Answer: b. 3

Explanation:
Put 12 counters(muffins) on the table.
Since you want to find 1/4 of the group, there should be equal groups of muffins.
Circle one of the groups to show 1/4th part of muffins.Then count the number of counters in that group.
There are 3 counters in 1 group. 1/4 of 12 = 3
So,  Juan has put 3 muffins in his bag.

Question 5.
Which is one way to read the time shown on the clock?
Go Math Grade 3 Answer Key Chapter 10 Time, Length, Liquid Volume, and Mass Estimate and Measure Liquid Volume img 56
Options:
a. 4 minutes before 7
b. 26 minutes before 11
c. 54 minutes after 6
d. 56 minutes after 7

Answer: a. 4 minutes before 7

Explanation:
Count by fives and ones from the 12 on the clock back to where the minute hand is pointing. Write the missing counting numbers next to the clock.
Write: 6:56
Read:
four minutes before seven

Question 6.
Julianne drew the line segment below. Use your ruler to measure the segment to the nearest fourth inch.
Go Math Grade 3 Answer Key Chapter 10 Time, Length, Liquid Volume, and Mass Estimate and Measure Liquid Volume img 57
Options:
a. \(\frac{3}{4}\) inch
b. 1 \(\frac{1}{4}\) inches
c. 1 \(\frac{1}{2}\) inches
d. 1 \(\frac{3}{4}\) inches

Answer: d. 1 \(\frac{3}{4}\) inches

Explanation:
Line up the left end of the paper clip with the zero mark on the ruler.
The right end of the paper clip is between the fourth-inch marks for 1 and2.
The mark that is closest to the right end of the paper clip is for 1 \(\frac{3}{4}\) inches.
So, the length of the paper clip to the nearest fourth inch is 1 \(\frac{3}{4}\) inches.

Estimate and Measure Liquid Volume – Page No. 609

Choose the unit you would use to measure the mass. Write gram or kilogram.

Question 1.
CD
Go Math Grade 3 Answer Key Chapter 10 Time, Length, Liquid Volume, and Mass Estimate and Measure Mass img 58
gram

Answer: Gram

Explanation:
The gram (g) is the basic metric unit for measuring mass or the amount of matter in an object. Mass can also be measured by using the metric unit kilogram (kg).
A small CD has a mass measured in grams.

Question 2.
boy
Go Math Grade 3 Answer Key Chapter 10 Time, Length, Liquid Volume, and Mass Estimate and Measure Mass img 59
________

Answer: Kilogram

Explanation:
A boy’s mass must be measured in kilograms, the metric unit. Since gram is the basic unit of mass and this measuring mass is used to measure small items such as CDs, paper pins, ice cream sticks etc.

Question 3.
bag of sugar
Go Math Grade 3 Answer Key Chapter 10 Time, Length, Liquid Volume, and Mass Estimate and Measure Mass img 60
________

Answer: Kilogram

Explanation:
A bag contains more than or equal to 1000 grams of measuring mass.
1000grams=1kilogram
Hence we can say that the sugar is measured in kilograms.

Question 4.
lion
Go Math Grade 3 Answer Key Chapter 10 Time, Length, Liquid Volume, and Mass Estimate and Measure Mass img 61
________

Answer: Kilogram

Explanation: Lion is a huge animal with muscles and hard bones. Hence, Lion is measured in kilograms.

Question 5.
paper clip
Go Math Grade 3 Answer Key Chapter 10 Time, Length, Liquid Volume, and Mass Estimate and Measure Mass img 62
________

Answer: Gram

Explanation:
A paper clip is light measured clip its mass is about 1 gram.

Question 6.
empty plastic bottle
Go Math Grade 3 Answer Key Chapter 10 Time, Length, Liquid Volume, and Mass Estimate and Measure Mass img 63
________

Answer: Gram

Explanation:
The amount of water in the bottle justifies the mass/volume of the bottle. Since the bottle is empty it is only measured in grams.

Compare the masses of the objects. Write is less than,
is the same as, or is more than.

Question 7.
Go Math Grade 3 Answer Key Chapter 10 Time, Length, Liquid Volume, and Mass Estimate and Measure Mass img 64
The mass of the candle ________ the mass of the light bulb.

Answer: is more than

Explanation:
Both candle and bulb measure in grams comparatively candle seems to have more mass than bulb.
Therefore we can say that mass of candle is more than a mass of bulb.

Question 8.
Go Math Grade 3 Answer Key Chapter 10 Time, Length, Liquid Volume, and Mass Estimate and Measure Mass img 65
The mass of the watch ________ the mass of the necklace.

Answer: is the same as

Explanation:
Both the watch and the necklace seem to have same amount of mass in grams.

Problem Solving

Question 9.
A red ball has a mass that is less than 1 kilogram. A blue ball has a mass of 1 kilogram. Is the mass of the blue ball more than or less than the mass of the red ball?
The mass of the blue ball is ________ the mass of the red ball

Answer: more than

Explanation:
Blue ball has a mass of 1 kilogram which more than that of the red ball because it has a mass less than 1 kilogram.

Question 10.
Brock’s dog is a collie. To find the mass of his dog, should Brock use grams or kilograms?
__________

Answer: kilograms

Explanation:

Estimate and Measure Liquid Volume – Page No. 610

Lesson Check

Question 1.
Which unit of measure would you use to measure the mass of a grape?
Options:
a. gram
b. inch
c. kilogram
d. meter

Answer: a. gram

Explanation:
A bunch of grapes are measured in kilograms while a single grape is measured in grams.

Question 2.
Elsie wants to find the mass of her pony. Which unit should she use?
Options:
a. gram
b. liter
c. kilogram
d. centimeter

Answer: c. kilogram

Explanation:
The mass of the pony is calculated in kilogram because the cluster of grams is called as kilogram.

Spiral Review

Question 3.
Marsie blew up 24 balloons. She tied the balloons together in groups of 4. How many groups did Marsie make?
Options:
a. 5
b. 6
c. 7
d. 8

Answer: b. 6

Explanation:
STEP 1 Let 24 counters be ballons.
STEP 2 Make a group of 4 counters tied together into a group. Continue making groups of 4 until all 24 counters are in groups.
There are 6 groups of 24 counters.

Question 4.
Clark used the order of operations to find the unknown number in 15 − 12 ÷ 3 = n. What is the value of the unknown number?
Options:
a. 1
b. 6
c. 9
d. 11

Answer: d. 11

Explanation:
STEP 1 Let there be 12 counters in a box.
STEP 2 Make a group of 3 counters by drawing a circle around them. Continue circling groups of 3 until all 12 counters are in groups.
There are 4 groups of 12 counters.
STEP3 There are 15 counters in a box
STEP4 Remove 4 counters from the box, then only 11 counters are left in the box.

Use the pictures for 5–6. Ralph pours juice into four bottles that are the same size.
Go Math Grade 3 Answer Key Chapter 10 Time, Length, Liquid Volume, and Mass Estimate and Measure Mass img 66

Question 5.
Which bottle has the most amount of juice?
Options:
a. Bottle A
b. Bottle B
c. Bottle C
d. Bottle D

Answer: a. Bottle A

Explanation:
Comparatively in all the four bottles bottle A is almost completely filled. Therefore bottle A has the most amount of juice.

Question 6.
Which bottle has the least amount of juice?
Options:
a. Bottle A
b. Bottle B
c. Bottle C
d. Bottle D

Answer: d. Bottle D

Explanation:
Comparatively in all the four bottles bottle D has the least amount of juice. Therefore bottle A has the least amount of juice.

Solve Problems About Liquid Volume and Mass – Page No. 615

Write an equation and solve the problem.

Question 1.
Luis was served 145 grams of meat and 217 grams of vegetables at a meal. What was the total mass of the meat and the vegetables?
Think: Add to find how much in all.
145 + 217 = _______ grams

Answer: 145 + 217 = 362 grams

Explanation:
STEP 1 Place 145 grams mass of meat.
STEP 2 Place 217 grams mass of vegetables at a meal .
STEP 3 To find the total mass of the meat and the vegetables is 362 grams.
So, the total mass of the meat and the vegetables is 362 grams.

Question 2.
The gas tank of a riding mower holds 5 liters of gas. How many 5-liter gas tanks can you fill from a full 20-liter gas can?
_______ ÷ _______ = _______ 5-liter gas tanks

Answer: 20 ÷ 5 = 4 gas tanks

Explanation:
STEP1 A single gas tank of a riding mower holds 5 liters of gas.
STEP2 Quantity of gas given to fill in the tanks is 20 liters.
STEP3 To calculate the number of tanks, we should make groups of 5 liters of gas such that the total quantity of 20 liters is completed
STEP4 The number of gas tanks is the number of counters in which the 20 liter of gas is filled.
Therefore the number of tanks are 4

Question 3.
To make a lemon-lime drink, Mac mixed 4 liters of lemonade with 2 liters of limeade. How much lemon-lime drink did Mac make?
_______ + _______ = _______ liters lemon-lime

Answer: 4 + 2 = 6 liters

Explanation:
STEP 1 Place 4 liters of lemonade for preparing the lemon-lime drink.
STEP 2 Place 2 liters of limeade for preparing the lemon-lime drink.
STEP 3Mass of lemon-lime drink made by Mac is 4+2=6 liters.
So, the total mass of the lemon-lime drink made is 6 liters.

Question 4.
A nickel has a mass of 5 grams. There are 40 nickels in a roll of nickels. What is the mass of a roll of nickels?
_______ × _______ = _______grams

Answer: 5 × 40 = 200 grams

Explanation:
STEP1 Given that there are 40 nickels in a roll of nickels.
STEP2 Each nickel has a mass of 5 grams.
STEP3 Place the counters(nickels) in rows.
STEP4 Find the mass of a roll of nickels.
STEP5 Mass of a roll of nickels=5×40
STEP6 Place 5 boxes each containing 40 counters. Add the counters in the boxes, total number of counters is equal to 200.

Question 5.
Four families share a basket of 16 kilograms of apples equally. How many kilograms of apples does each family get?
_______ ÷ _______ = _______ kilograms

Answer: 20 ÷ 5 = 4

Explanation:
STEP1 Total number of apples =16
STEP2 Number of families =4
STEP3 Number of apples each family shared=16÷4
=4 apples

Question 6.
For a party, Julia made 12 liters of fruit punch. There were 3 liters of fruit punch left after the party. How much fruit punch did the people drink at the party?
_______ – _______ = _______ liters

Answer: 12 – 3 = 9 liters

Explanation:
STEP1 Quantity of fruit punch made by Julia=12 liters
STEP2 Quantity of fruit punch left after the party=3 liters
STEP3 Quantity of fruit punch the people drank in the party=12-3 = 9 liters

Problem Solving

Question 7.
Zoe’s fish tank holds 27 liters of water. She uses a 3-liter container to fill the tank. How many times does she have to fill the 3-liter container in order to fill her fish tank?
_______ times

Answer: 9 times

Explanation:
STEP1 Quantity of water Zoe’s fish tank holds=27 liters
STEP2  Zoe has a 3-liter container to fill the tank.
STEP3 Total number of times  she has to fill the 3-liter container in order to fill her fish tank=27÷3=9 times

Question 8.
Adrian’s backpack has a mass of 15 kilograms. Theresa’s backpack has a mass of 8 kilograms. What is the total mass of both backpacks?
15 + 8 = _______ kilograms

Answer: 23 kilograms

Explanation:
STEP1 Mass of Adrian’s backpack = 15 kilograms
STEP2 Mass of Theresa’s backpack= 8 kilograms
STEP3  Total mass of both backpacks=15+8=23 kilograms

Solve Problems About Liquid Volume and Mass – Page No. 616

Lesson Check

Question 1.
Mickey’s beagle has a mass of 15 kilograms. His dachshund has a mass of 13 kilograms. What is the combined mass of the two dogs?
Options:
a. 2 kilograms
b. 18 kilograms
c. 23 kilograms
d. 28 kilograms

Answer: d. 28 kilograms

Explanation:
Mickey has 2 dogs beagle and dachshund
STEP1 Mass of Beagle=15 kilograms
STEP2 Mass of dachshund= 13 kilograms
STEP3 Combined mass of the two dogs= 15+13=28 kilograms

Question 2.
Lois put 8 liters of water in a bucket for her pony. At the end of the day, there were 2 liters of water left. How much water did the pony drink?
Options:
a. 4 liters
b. 6 liters
c. 10 liters
d. 16 liters

Answer: b. 6 liters

Explanation:
STEP1 Quantity of the water put in the bucket by Lois=8 liters
STEP2 Quantity of the water left in the bucket=2 liters
STEP3 Quantity of the water drank by pony= 8-2 = 6 liters

Spiral Review

Question 3.
Josiah has 3 packs of toy animals. Each pack has the same number of animals. Josiah gives 6 animals to his sister Stephanie. Then Josiah has 9 animals left. How many animals were in each pack?
Options:
a. 1
b. 3
c. 5
d. 6

Answer: c. 5

Explanation:
STEP1 Number of packs Josiah has =3
STEP2 Number of animals given by Josiah to his sister =6
STEP3 Total number of animals = 6+9=15
According to the problem,
Each pack has the same number of animals.
3× ■ = 15
Count how many rows of 15 counters there are. There are  3 rows of 15 counters. The unknown factor is 5 (c0lumns). n = 5
3 × 5 = 15

Question 4.
Tom jogged \(\frac{3}{10}\) mile, Betsy jogged \(\frac{5}{10}\) mile, and Sue jogged \(\frac{2}{10}\) mile. Who jogged a longer distance than \(\frac{4}{10}\) mile?
Options:
a. Betsy
b. Sue
c. Tom
d. None

Answer: a. Betsy

Explanation:
When the numerators are the same, think about the of the pieces to compare and order fractions. So, the order from greatest to least gives, the greatest fraction and the person who jogged a longer distance than \(\frac{4}{10}\) mile.
\(\frac{5}{10}\)>\(\frac{3}{10}\)>\(\frac{2}{10}\)
Therefore Betsy jogged a longer distance than \(\frac{4}{10}\) mile.

Question 5.
Bob started mowing at 9:55 A.M. It took him 25 minutes to mow the front yard and 45 minutes to mow the back yard. At what time did Bob finish mowing?
Options:
a. 10:20 A.M.
b. 10:55 A.M.
c. 11:05 A.M.
d. 11:20 A.M.

Answer: c. 11:05 A.M.

Explanation:
STEP1 Time Bob spent in mowing=25+45=70 minutes
STEP2 Finding the time at which Bob finished mowing
STEP3 Find the ending time on the number line.
STEP4 Count forward on the number line to add the elapsed time. Draw and label the jumps to show the minutes.
STEP5 Write the times below the number line.
The jumps end at 11:05 A.M.
So, the ending time is 11:05 A.M.

Question 6.
Juliana wants to find the mass of a watermelon. Which unit should she use?
Options:
a. gram
b. kilogram
c. liter
d. meter

Answer: b. kilogram

Explanation:
Mass is measured in grams and kilograms small quantities like paper clips, pins are measured in grams while large quantities are measured in kilograms. Therefore watermelon is measured in kilogram.

Review/Test – Page No. 617

Question 1.
Yul and Sarah’s art class started at 11:25 a.m. The class lasted 30 minutes. Yul left when the class was done. Sarah stayed an extra 5 minutes to talk with the teacher and then left.
Write the time that each student left. Explain how you found each time.
Yul: _______ A.M.
Sarah: ______ P.M.

Answer: Yul: 11:55 A.M.
Sarah: 12:05 P.M.

Explanation:
Yul:
STEP 1 Find the ending time on the number line.
STEP 2 Count forward on the number line to add the elapsed time, 30 minutes. Draw and label the jumps to show the minutes.
STEP 3 Write the times below the number line.
The jumps end at 11:55 A.M.
So, the ending time is 11:55 A.M.
Sarah:
STEP 1 Find the starting time on the number line
STEP 2 Count back on the number line to subtract the elapsed time, 30+5=35 minutes. Draw and label the jumps to show the minutes.
STEP 3 Write the times below the number line.
The jumps end at  12:05 P.M.
So, the starting time is 12:05 P.M.

Question 2.
Julio measured an object that he found. It was about 3/4 inch wide.
For numbers 2a–2d, choose Yes or No to tell whether the object could be the one Julio measured.
a. Go Math Grade 3 Answer Key Chapter 10 Time, Length, Liquid Volume, and Mass Review/Test img 67
i. yes
ii. no

Answer: ii. no

Question 2.
b. Go Math Grade 3 Answer Key Chapter 10 Time, Length, Liquid Volume, and Mass Review/Test img 68
i. yes
ii. no

Answer: i. yes

Explanation:
The mark that is closest to the right end of the stamp is for 1 inch. So, the length of the stamp to the nearest fourth inch is 3/4 inches. Therefore the stamp is about 3/4 inch wide.

Question 2.
c. Go Math Grade 3 Answer Key Chapter 10 Time, Length, Liquid Volume, and Mass Review/Test img 69
i. yes
ii. no

Answer: ii. no

Question 2.
d. Go Math Grade 3 Answer Key Chapter 10 Time, Length, Liquid Volume, and Mass Review/Test img 70
i. yes
ii. no

Answer: ii. no

Review/Test – Page No. 618

Question 3.
Dina started swimming at 3:38 p.m. She swam until 4:15 p.m. How long did Dina swim?
________ minutes

Answer: 37 minutes

Explanation:
STEP 1 Find 3:38 P.M. on the number line. Count on from 3:38 P.M. to 4:15 P.M. Draw marks and record the times on the number line. Then draw and label the jumps.
STEP 2 Add to find the total minutes from 3:38 P.M. to 4:15 p.M. From 3:38 P.M. to 4:15 P.M. is 37 minutes.
So, the elapsed time is 37 minutes

Question 4.
Rita’s class begins social studies at ten minutes before one in the afternoon. At what time does Rita’s class begin social studies? Circle a time that makes the sentence true.
Rita’s class begins social studies at Go Math Grade 3 Answer Key Chapter 10 Time, Length, Liquid Volume, and Mass Review/Test img 71
__________

Answer: 12:50 P.M.

Explanation:
Rita’s class begins social studies at ten minutes before one in the afternoon. So, the time at which the class begins is 10 minutes subtracted from 1:00 P.M. is 12:50 P.M.

Question 5.
Select the objects with a mass greater than 1 kilogram.
Mark all that apply.
Options:
a. bicycle
b. pen
c. eraser
d. math book

Answer: a. bicycle, has a mass greater than 1 kilogram.

Explanation:
Mass is measured in grams and kilograms small quantities like paper clips, pins are measured in grams while large quantities are measured in kilograms. Therefore mass of bicycle is measured in kilograms.

Question 6.
A chicken dish needs to bake in the oven for 35 minutes. The dish needs to cool for at least 8 minutes before serving. Scott puts the chicken dish in the oven at 5:14 p.m. For numbers 6a–6d, select True or False for each statement.
a. Scott can serve the dish at 5:51 p.m.
i. True
ii. False

Answer: ii. False

Question 6.
b. Scott can serve the dish at 5:58 p.m.
i. True
ii. False

Answer: ii. False

Question 6.
c. Scott should take the dish out of the oven at 5:51 a.m
i. True
ii. False

Answer: ii. False

Question 6.
d. Scott should take the dish out of the oven at 5:49 p.m.
i. True
ii. False

Answer: i. True

Explanation:
STEP 1 Find the ending time on the number line.
STEP 2 Count forward on the number line to add the elapsed time, 35 minutes ie. the time when the dish is kept in the oven to bake. Draw and label the jumps to show the minutes.
STEP 3 Write the times below the number line.
The jumps end at 5:49 P.M.
So, the ending time is 5:49 P.M.

Review/Test – Page No. 619

Question 7.
Anthony’s family went out to dinner. They left at the time shown on the clock. They returned home at 6:52 p.m.
Go Math Grade 3 Answer Key Chapter 10 Time, Length, Liquid Volume, and Mass Review/Test img 72
Part A
How long was Anthony’s family gone?
_____ hour _____ minutes
_____ hour _____ minutes

Answer: 5 hours 05 minutes
6 hours 52 minutes

Explanation:
Anthony’s family left for dinner at 5 hours 05 minutes
Anthony’s family returned back home at 6 hours 52 minutes

Question 7.
Part B
Explain how you found your answer.
Type below:
__________

Answer: 107 minutes

Explanation:
STEP 1 Find 5:05 P.M. on the number line. Count on from 5:05 P.M. to 6:52 P.M. Draw marks and record the times on the number line. Then draw and label the jumps.
STEP 2 Add to find the total minutes from 5:05 P.M. to 6:52 p.M. From 5:05 P.M. to 6:52 P.M. is 107 minutes.
So, the elapsed time is 107 minutes

Question 8.
Tran checked the time on his watch after he finished his daily run.
Go Math Grade 3 Answer Key Chapter 10 Time, Length, Liquid Volume, and Mass Review/Test img 73
Select the time that Tran finished running. Mark all that apply.
Options:
a. 14 minutes before nine
b. eight forty-six
c. quarter to nine
d. nine forty-six

Answer: a. 14 minutes before nine
b. eight forty-six

Explanation:
a. Count by fives and ones from the 12 on the clock back to where the minute hand is pointing. Write the missing counting numbers next to the clock.
b. Count on by fives and ones from the 12 on the clock to where the minute hand is pointing. Write the missing counting numbers next to the clock.
Write: 8 Hour 46 minutes

Question 9.
Cara uses a balance scale to compare mass.
Go Math Grade 3 Answer Key Chapter 10 Time, Length, Liquid Volume, and Mass Review/Test img 75
Circle a symbol that makes the comparison true.
The mass of the blocks img 73 the mass of the erasers.
________

Answer: mass of the blocks>mass of the erasers

Explanation:
Mass of the blocks is measured in kilograms while mass of the erasers is measured in grams.

Review/Test – Page No. 620

Question 10.
A large bottle of water holds about 2 liters. For numbers 10a–10e, choose Yes or No to tell whether the container will hold all of the water.
a. kitchen sink
i. yes
ii. no

Answer: i. yes

Explanation: A kitchen sink can hold about 2 liters. Due to the larger surface area, the kitchen sink can hold about 2 liters.

Question 10.
b. water glass
i. yes
ii. no

Answer: ii. no

Question 10.
c. ice cube tray
i. yes
ii. no

Answer: ii. no

Question 10.
d. large soup pot
i. yes
ii. no

Answer: ii. no

Question 10.
e. lunchbox thermos
i. yes
ii. no

Answer: i. yes

Explanation: Lunchbox thermos can hold about only 2 liters due to its surface area, and height.

Question 11.
Select the items that would be best measured in grams.
Mark all that apply.
Options:
a. watermelon
b. lettuce leaf
c. grape
d. onion

Answer: b. lettuce leaf
c. grape

Explanation: A single leaf is measured in grams.
Mass of a single grape is measured in grams.

Question 12.
Samir made a list of what he did on Tuesday. Write the letter for each activity next to the time he did it.

A. Get out of bed. 8:05 a.m.(b)
B. Walk to school. 6:25 p.m.(e)
C. Eat lunch. 3:50 p.m.(d)
D. Go to guitar lesson after school. 11:48 a.m.(c)
E. Eat dinner at home. 6:25 a.m.(a)

Explanation:
The times after midnight and before noon is written with A.M. The times afternoon and before midnight is written with P.M.Samir gets out of the bed at 6:25 a.m. in the morning and walk to school by 8:05 a.m. in the morning. Samir finishes his lunch in the school by 11:48 a.m. After school hours Samir goes to guitar lessons at 3:50 p.m. After attending his classes Samir at 6:25 p.m. eats his dinner at home.

Review/Test – Page No. 621

Question 13.
Amy has 30 grams of flour. She puts 4 grams of flour in each pot of chowder that she makes. She puts 5 grams of flour in each pot of potato soup that she makes. She makes 4 pots of chowder. Does Amy have enough flour left over to make 3 pots of potato soup?
______

Answer: No

Explanation:
Mass of flour Amy has= 30grams
Mass of flour in each pot of chowder= 4grams
Mass of flour in each pot of potato soup= 5grams
According to the problem,
Amy makes 4 pots of chowder.
Mass of flour used to make 4 pots of chowder= 4×4= 16grams
Leftover mass of flour=30-16=14 grams
Mass of flour used to make 3 pots of potato soup=5×3=15 grams
But, Amy only has 14 grams. So, we can say that Amy doesn’t have enough flour leftover to make 3 pots of potato soup.

Question 14.
Use an inch ruler to measure.
Go Math Grade 3 Answer Key Chapter 10 Time, Length, Liquid Volume, and Mass Review/Test img 76
Part A
What is the length of the leaf to the nearest fourth inch?
Type below:
__________

Answer: 2 1/4 inches

Explanation:
Line up the left end of the leaf with the zero mark on the ruler.
The right end of the leaf is between the fourth-inch marks for 2 and 3.
The mark that is closest to the right end of the leaf is for 1/4 inches.
So, the length of the leaf to the nearest fourth inch is 2 1/4 inches.

Question 14.
Part B
Explain what happens if you line up the left side of the object with the 1 on the ruler.
Type below:
__________

Answer: 1 1/4

Explanation:
Line up the left end of the object with the one mark on the ruler.
The right end of the paper clip is between the fourth-inch marks for 2 and 3.
The mark that is closest to the right end of the paper clip is for 1/4 inches.
So, the length of the paper clip to the nearest fourth inch is 1 1/4 inches.

Question 15.
Mrs. Park takes the 9:38 a.m. train to the city. The trip takes 3 hours and 20 minutes. What time does Mrs. Park arrive in the city?
______ : _______ _______

Answer: 12:58 P.M.

Explanation:
STEP 1 Find the ending time on the clock, the time at which Mrs. Park arrive in the city.
STEP 2 Count forward by hours and then by fives for the elapsed time of 3 hours 20 minutes. Write the missing counting numbers next to the clock.
So, the ending time is 12:58 P.M.

Question 16.
Hector buys two bags of gravel for his driveway. He buys a total of 35 kilograms of gravel. Select the bags he buys.
Go Math Grade 3 Answer Key Chapter 10 Time, Length, Liquid Volume, and Mass Review/Test img 77

Answer: 17 kg and 18 kg

Explanation:
Hector buys two bags of gravel for his delivery.
Hector buys a total of 35 kilograms of gravel.
According to the problem,
The bags selected by Hector are 17 kg and 18 kg because 17+18=35 kg

Review/Test – Page No. 622

Question 17.
Ashley measures the shells she collects. She records the measurements in a chart.
Go Math Grade 3 Answer Key Chapter 10 Time, Length, Liquid Volume, and Mass Review/Test img 78
Part A
Ashley found a razor clam shell this long. Use an inch ruler to measure. Record the measurement in the chart.
Go Math Grade 3 Answer Key Chapter 10 Time, Length, Liquid Volume, and Mass Review/Test img 79
_____ \(\frac{□}{□}\) inches

Answer: 4 inches

Question 17.
Part B
Complete the line plot to show the data in the chart. How many shells are longer than 2 inches? Tell how you know.
Go Math Grade 3 Answer Key Chapter 10 Time, Length, Liquid Volume, and Mass Review/Test img 80
_______ shells

Answer: 4

Explanation:
Calculate the number of shells which are longer than 2 inches.
By observing the number line we find 2. From 2 draw curves to the next values in such a way that the other values are greater than 2.
We need to draw the curves till 4.
Count the curves drawn from 2 to 4 the number of curves=number of shells (which are measured to the nearest half-inch)=4

Question 18.
Lucy fills a bathroom sink with water. Is the amount of water more than 1 liter, about 1 liter, or less than 1 liter?
Explain how you know.
________

Answer: More than one liter

Explanation:
Volume is measured in milliliters and liters. Small quantities like syrup, water in glass etc. are measured in milliliters while the volume of water in the bathroom sink, oil can are measured in liters. Therefore, the bathroom sink with water can hold more than one liter of water.

Summary

We believe the information shed regarding Go Math Grade 3 Answer Key Chapter 10 Time, Length, Liquid Volume, and Mass has helped you a lot. If you need any further help you can test your knowledge using the Go Math Grade 3 Answer Key Chapter 10 Time, Length, Liquid Volume, and Mass Extra Practice. Bookmark our site to seek information on different Classes Go Math Answer Key.

Go Math Grade 3 Answer Key Chapter 3 Understand Multiplication

go-math-grade-3-chapter-3-understand-multiplication-answer-key

Practice using Go Math Grade 3 Answer Key Chapter 3 Understand Multiplication and learn various question types on Multiplication. Solve as many questions as possible in the 3rd Grade HMH Go Math Ch 3 Understand Multiplication Answer Key and become pro in the chapter. Thus, you can answer any kind of question framed on the concept and clear your tests or assessments with higher scores.

HMH Go Math Grade 3 Answer Key Chapter 3 Understand Multiplication

To help you out with chapter 3 Understand Multiplication we have jotted down all the exercise problems in your Go Math Textbook. Simply tap on the respective chapter you wish to prepare and learn the fundamentals included within it easily. Go Math Grade 3 Answer Key Ch 3 Understand Multiplication is provided in a comprehensive manner for better understanding.

Lesson 1: Count Equal Groups

Lesson 2: Algebra • Relate Addition and Multiplication

Lesson 3: Skip Count on a Number Line

Mid-Chapter Checkpoint

Lesson 4: Problem Solving • Model Multiplication

Lesson 5: Model with Arrays

Lesson 6: Algebra • Commutative Property of Multiplication

Lesson 7: Algebra • Multiply with 1 and 0

Chapter 3 Review/Test

Count Equal Groups Page No 143

Draw equal groups. Skip count to find how many.

Question 1.
2 groups of 2 4
Go Math Grade 3 Answer Key Chapter 3 Understand Multiplication Count Equal Groups img 1

Answer: 4

Explanation:
There are two groups of 2
There are 2 two’s.
We skip count by 2’s. (2, 4)
So, there are 4 in all.

Question 2.
3 groups of 6 ________

Answer: 18

Explanation:
Draw 6 counters in each group.
There are 3 equal groups.
skip count by six until you say 3 numbers.
There are 3 equal groups with 6 counters in each group.
So, there are 18 counters in all.

Question 3.
5 groups of 3 ________

Answer: 15

Explanation:
Draw 3 counters in each group.
There are 5 equal groups.
Now, skip count by 3’s until you say 5 numbers.
There are 5 equal groups with 3 counters in each group.
So, there are 15 counters in all.

Question 4.
4 groups of 5 ________

Answer: 20

Explanation:
Draw 5 counters in each group.
There are 4 equal groups.
Skip count by 5’s until you say 4 numbers.
There are 4 equal groups with 5 counters in each group.
So, there are 20 counters in all.

Count equal groups to find how many.

Question 5.
Go Math Grade 3 Answer Key Chapter 3 Understand Multiplication Count Equal Groups img 2
______ groups of _______ , _______ in all

Answer: 4, 3, 12

Explanation:
There are three counters in each group.
There are 4 equal groups.
We skip count by 3’s until you say 4 numbers. (3,6,9,12)
So, there are 4 groups of 3, 12 in all.

Question 6.
Go Math Grade 3 Answer Key Chapter 3 Understand Multiplication Count Equal Groups img 3
______ groups of _______ , _______ in all

Answer: 4, 4, 16

Explanation:
There are 4 counters in each group
There are 4 equal groups.
We skip count by 4’s until you say 4 numbers. (4,8,12,16)
There are 4 groups of 4, 16 in all

Problem Solving

Question 7.
Marcia puts 2 slices of cheese on each sandwich. She makes 4 cheese sandwiches. How many slices of cheese does Marcia use in all?
_________ slice of cheese

Answer: 8

Explanation:
Draw 2 counters (cheese) in each group.
There are 4 equal groups (sandwiches).
We skip count by 2’s until you say 4 numbers (4,8)
There are 8 slices of cheese.

Question 8.
Tomas works in a cafeteria kitchen. He puts 3 cherry tomatoes on each of 5 salads. How many tomatoes does he use?
_________ tomatoes

Answer: 15

Explanation:
Draw 3 counters (tomatoes) in each group.
There are 5 equal groups (salads).
We skip count by 3’s until you say 5 numbers (3,6,9,12,15)
There are 15 tomatoes in all.

Count Equal Groups Lesson Check Page No 144

Question 1.
Jen makes 3 bracelets. Each bracelet has 3 beads. How many beads does Jen use?
Go Math Grade 3 Answer Key Chapter 3 Understand Multiplication Count Equal Groups img 4
Options:
a. 12
b. 9
c. 6
d. 3

Answer: b

Explanation:
There are 3 beads in each group.
There are 3 equal groups (bracelets).
Now, skip count by 3’s until you say three numbers (3,6,9)
Jen use a total of 9 beads.

Question 2.
Ian has 5 cards to mail. Each card needs 2 stamps. How many stamps does Ian need?
Go Math Grade 3 Answer Key Chapter 3 Understand Multiplication Count Equal Groups img 5
Options:
a. 2
b. 5
c. 10
d. 15

Answer: c

Explanation:
There are 5 equal groups (cards) with two counters (stamps) in each group.
Now, skip count by 2’s until you say five numbers (2,4,6,8,10)
Ian need a total of 1o stamps.

Spiral Review

Question 3.
There were 384 people at a play on Friday night. There were 512 people at the play on Saturday night. Which is the best estimate of the total number of people who attended the play on both nights?
Options:
a. 900
b. 800
c. 700
d. 500

Answer: a

Explanation:
Step 1: Round each number to nearest hundred.
384 —-> 400
512  —-> 500
Step 2: Add the rounded numbers
400 + 500 = 900
The best estimate of the total number of people who attended the play on both nights are 900.

Question 4.
Walking the Dog Pet Store has 438 leashes in stock. They sell 79 leashes during a one-day sale. How many leashes are left in stock after the sale?
Options:
a. 459
b. 441
c. 369
d. 359

Answer: d

Explanation: Use place value to subtract
Subtract 438 – 79
Estimate 450 – 100 =

Step 1
Subtract the ones.
8 < 9, so regroup.
3 tens 8 ones = 2 tens ____ ones
GO Math Grade 3 Count Equal Groups Spiral Review Page No 144 Answer 1 - i
On subtracting the ones place decimals, we get 9

Step 2
Subtract the tens.
2 < 3, so regroup.
4 hundreds 2 tens = 3 hundreds _____ tens
GO Math Grade 3 Count Equal Groups Spiral Review Page No 144 Answer 1 - ii
On subtracting the ones and tens place decimals, we get 59

Step 3:
Subtract the hundreds
Then, add to check your answer.
GO Math Grade 3 Count Equal Groups Spiral Review Page No 144 Answer 1 - iii  GO Math Grade 3 Count Equal Groups Spiral Review Page No 144 Answer 1 - iv

Question 5.
The Lakeside Tour bus traveled 490 miles on Saturday and 225 miles on Sunday. About how many more miles did it travel on Saturday?
Options:
a. 500 miles
b. 400 miles
c. 300 miles
d. 100 miles

Answer: c

Explanation:
Use compatible numbers
Step 1: Round each number to nearest hundred.
490 —-> 500
225 —-> 200
Step 2: Subtract the rounded numbers.

(i) Working each column from right to left
GO Math Grade 3 Count Equal Groups Spiral Review Page No 144 Answer 2 - i
(ii) 0 minus 0 is 0
GO Math Grade 3 Count Equal Groups Spiral Review Page No 144 Answer 2 - ii
(iii) 0 minus 0 is 0
GO Math Grade 3 Count Equal Groups Spiral Review Page No 144 Answer 2 - iii
(iv) 5 minus 2 is 3
GO Math Grade 3 Count Equal Groups Spiral Review Page No 144 Answer 2 - iv

Question 6.
During one week at Jackson School, 210 students buy milk and 196 students buy juice. How many drinks are sold that week?
Options:
a. 496
b. 406
c. 396
d. 306

Answer: b

Explanation:
Use place value to add two addends.
Add 210 + 196
Estimate 200 + 200

Step 1:
Add the ones.

GO Math Grade 3 Count Equal Groups Spiral Review Page No 144 Answer 3 - i

Step 2:

Add the tens.
GO Math Grade 3 Count Equal Groups Spiral Review Page No 144 Answer 3 - ii

Step 3:
Add the hundreds. Regroup the tens as hundreds.
GO Math Grade 3 Count Equal Groups Spiral Review Page No 144 Answer 3 - iii

Relate Addition and Multiplication Page No 149

Draw a quick picture to show the equal groups. Then write related addition and multiplication sentences.

Question 1.
3 groups of 5
Go Math Grade 3 Answer Key Chapter 3 Understand Multiplication Relate Addition and Multiplication img 6
5 + 5 + 5 = 15
3 × 5 = 15

Answer: 15

Explanation:
Addition Sentence:
There are 5 counters in each group.
There are 3 groups.
Now, add equal groups
5 + 5 + 5 = 15
Multiplication Sentence:
Since there are same number of counters in each circle, you can multiply to find how many in all.
Use multiplication method to find the total counters in equal groups.
Factor x Factor = Product
5    x     3       =  15
GO Math Grade 3 Understand Multiplication Relate Addition and Multiplication Page No 149 Answer 1 - i

Question 2.
3 groups of 4
_____ + _____ + _____ = ______
_____ × _____ = ______

Answer: 4 + 4 + 4  = 12
3 x 4 = 12

Explanation: Addition Sentence
Draw 4 counters in each group.
There are total 3 groups.
Now, the addition sentence is 4 + 4 + 4 = 12

Multiplication sentence
Draw 4 counters in each circle or group.
Since there are same number of counters in each group, multiply groups and counters to find how many there are altogether.
3 x 4 = 12
factor x factor = product

Question 3.
4 groups of 3
_____ + _____ + _____ + _____ = ______
_____ × _____ = ______

Answer: 3 + 3 + 3 + 3 = 12
4 x 3 = 12

Explanation:
Addition Sentence
Draw 3 counters in each group.
There are total 4 groups.
Now, the addition sentence is 3 + 3 + 3 + 3 = 12

Multiplication sentence
Draw 3 counters in each circle or group.
Since there are same number of counters in each group, multiply counters and groups to find how many there are altogether.
4 x 3 = 12
factor x factor = product

Question 4.
5 groups of 2
_____ + _____ + _____ + _____ + _____ = ______
_____ × _____ = ______

Answer: 2 + 2 + 2 + 2 + 2 = 10
5 x 2 = 10

Explanation:
Addition Sentence
Draw 2 counters in each group.
There are total 5 groups.
Now, the addition sentence is 2 + 2 + 2 + 2 + 2 = 12

Multiplication sentence
Draw 2 counters in each circle or group.
Since there are same number of counters in each group, multiply counters and groups to find how many there are altogether.
5 x 2 = 12
factor x factor = product

Complete. Write a multiplication sentence.

Question 5.
7 + 7 + 7 = _____
_____ × _____ = ______

Answer: 7 + 7 + 7 = 21
3 x 7 = 21

Explanation:
7 + 7 + 7 = 21
This is addition sentence, from this we can find the total no of groups and counters.
From above sentence, we can tell that there are 3 groups and 7 counters in each group.
Multiplication Sentence
There are 3 groups and 7 counters in each group.
Since there are same number of counters in each group, multiply counters and groups to find how many there are altogether.
3  x  7 =  21

Question 6.
3 + 3 + 3 = _____
_____ × _____ = ______

Answer: 3 + 3 + 3 = 9
3 x 3 = 9

Explanation:
3 + 3 + 3 = 9
This is addition sentence, from this we can find the total no of groups and counters.
From above sentence, we can tell that there are 3 groups and 3 counters in each group.
Multiplication Sentence
There are 3 groups and 3 counters in each group.
Since there are same number of counters in each group, multiply counters and groups to find how many there are altogether.
3  x  3 =  9

Problem Solving

Question 7.
There are 6 jars of pickles in a box. Ed has 3 boxes of pickles. How many jars of pickles does he have in all? Write a multiplication sentence to find the answer.
_____ × _____ = ______ jars

Answer: 3 x 6 = 18 jars

Explanation:
Draw 3 boxes as Pickle boxes.
Draw 6 counters in each box to show jars.
Now, find the number of counters (jars).
Since, there are same number of jars in each pickle box, you can multiply to find how many in all.
3 groups of 6 = 3 x 6 = 18

Question 8.
Each day, Jani rides her bike 5 miles. How many miles does Jani ride in all in 4 days? Write a multiplication sentence to find the answer.
_____ × _____ = ______ miles

Answer: 4 x 5 = 20 miles

Explanation:
Draw 4 circles to show 4 days.
Draw 5 counters to show miles.
Now, find the total no of miles ride by Jani in 4 days.
Since, Jani rides same number of miles each day, you can multiply to find how many in all.
4 groups of 5 = 4 x 5 = 20

Relate Addition and Multiplication Lesson Check Page No 150

Question 1.
Which is another way to show
3 + 3 + 3 + 3 + 3 + 3?
Options:
a. 5 × 3
b. 4 × 3
c. 8 × 3
d. 6 × 3

Answer: d

Explanation:
The given question is in the form of addition sentence. Now, we can find the no of counters and groups using it.
Draw 6 circles to show equal groups.
Then, draw 3 counters in each circle.
When you combine equal groups, you can multiply to find how many in all.
No. of equal groups x no of counters = total no of counters
6 x 3 = 18

Question 2.
Use the model. How many counters are there in all?
Go Math Grade 3 Answer Key Chapter 3 Understand Multiplication Relate Addition and Multiplication img 7
Options:
a. 8
b. 10
c. 12
d. 14

Answer: b

Explanation:
From the above figure, we can see there are two equal groups.
There are 5 counters in each group.
Now, multiplication sentence to find the number of counters.
No. of equal groups x No. of counters in each group = Total no. of counters
2 x 5 = 10

Spiral Review

Question 3.
A school gave 884 pencils to students on the first day of school. What is 884 rounded to the nearest hundred?
Options:
a. 800
b. 880
c. 890
d. 900

Answer: d

Explanation:
We need to round 884 to nearest hundred.
Now, look at the digit to the right which is 8.

GO Math Grade 3 Understand Multiplication Spiral Review Page No 150 Answer 5 - i
8 > 5
So, the hundreds digit increases by one.
Write 9 as the hundreds digit.
Write zeros as the tens and ones digits.
So, 884 rounded to nearest hundred is 900

Question 4.
Find the difference.
6 3 2
– 2 7 4
———
Options:
a. 906
b. 442
c. 358
d. 354

Answer: c

Explanation:
Step-1:
Working each column from right to left.
GO Math Grade 3 Understand Multiplication Lesson Check Page No 150 Answer 6 - i
Step-2:
Subtract the ones. Since, 2 < 4 , you must regroup.
3 tens 2 ones = 2 tens ______ ones.
GO Math Grade 3 Understand Multiplication Lesson Check Page No 150 Answer 6 - ii

Step-3:
Subtract the tens. Since 2 < 7, you must regroup.
6 hundreds 2 tens = 5 hundreds ______ ones.
GO Math Grade 3 Understand Multiplication Lesson Check Page No 150 Answer 6 - iii
Step-4:
Subtract the hundreds.
GO Math Grade 3 Understand Multiplication Lesson Check Page No 150 Answer 6 - iv
Step-5:
Add to check answer.
GO Math Grade 3 Understand Multiplication Lesson Check Page No 150 Answer 6 - v

Question 5.
The line plot below shows how many points Trevor scored in 20 games.
Go Math Grade 3 Answer Key Chapter 3 Understand Multiplication Relate Addition and Multiplication img 8
In how many games did Trevor score at least 18 points?
Options:
a. 3
b. 5
c. 6
d. 10

Answer: d

Explanation:
The numbers in the line plot shows the points scored by Trevor.
Each in the figure stands for 1 game.
Find 18 points scored on the number line.
In the above line plot, we can see the three s above 18, four s above 19, 3 s above 20.
So, We can say Trevor scored at least 18 points in 10 games.

Question 5.
In how many games did Trevor score 18 points or fewer?
Options:
a. 7
b. 10
c. 13
d. 15

Answer: c

Explanation:
The numbers in the line plot shows the points scored by Trevor.
Each in the figure stands for 1 game.
Find 18 and below 18 points scored on the number line.
In the above line plot, we can see the three s above 18, five s above 17, two s above 16, three s above 15 .
So, We can say Trevor scored at least 18 points in 13 games.

Question 6.
Darrien read 97 pages last week. Evan read 84 pages last week. How many pages in all did the boys read?
Options:
a. 13
b. 171
c. 181
d. 271

Answer: c

Explanation:
Add 97 and 84
Step 1:
Add ones. Regroup the ones as tens and ones.
GO Math Grade 3 Understand Multiplication Spiral Review Page No 150 Answer 7 - i
Step 2:
Add tens.
GO Math Grade 3 Understand Multiplication Spiral Review Page No 150 Answer 7 - ii
Step 3:
Add hundreds.
GO Math Grade 3 Understand Multiplication Spiral Review Page No 150 Answer 7 - iii

Skip Count on a Number Line Page No 155

Draw jumps on the number line to show equal groups. Find the product.

Question 1.
6 groups of 3
Go Math Grade 3 Answer Key Chapter 3 Understand Multiplication Skip Count on a Number Line img 9
6 × 3 = 18

Answer: 18

Explanation:
1 jump on the number line is considered as 1 group.
There are 6 jumps on the number line. So, there are 6 groups.
The length of each jump is 3.
Begin at 0. Skip count by 3’s.
Multiply 6 x 3 = 18

Question 2.
3 groups of 5
Go Math Grade 3 Answer Key Chapter 3 Understand Multiplication Skip Count on a Number Line img 10
3 × 5 = _______

Answer: 15

Explanation:
GO Math Grade 3 Understand Multiplication Skip Count on a Number Line Page No 155 Answer 8 - i
1 jump on the number line is considered as 1 group.
There are 5 jumps on the number line. So, there are 5 groups.
The length of each jump is 3.
Begin at 0. Skip count by 3’s.
Multiply 3 x 5 = 15.

Write the multiplication sentence the number line shows.

Question 3.
2 groups of 6
Go Math Grade 3 Answer Key Chapter 3 Understand Multiplication Skip Count on a Number Line img 11
______ × _____ = _______

Answer: 12

Explanation:
1 jump on the number is considered as 1 group.
There are 2 jumps on number line. So, there are 2 groups.
The length of each jump is 2.
Begin at 0. Skip count by 6’s.
Multiply 2 x 6 = 12.

Problem Solving

Question 4.
Allie is baking muffins for students in her class. There are 6 muffins in each baking tray. She bakes 5 trays of muffins. How many muffins is she baking in all?
________ muffins

Answer: 30 Muffins

Explanation:
There are 5 baking trays with 6 muffins in each tray.
Consider 1 tray as 1 jump and 1 muffin as length of each jump.
Now, use a number line to find how many muffins allie baked in all.
Begin at 0. Skip count by 6s by drawing jumps on the number line.
No. of jumps made = 5
Length of each jump = 6
Multiply. 5 x 6 = 30
So, Allie bakes 30 muffins in all.

Question 5.
A snack package has 4 cheese sticks. How many cheese sticks are in 4 packages?
________ cheese sticks

Answer: 16 Cheese sticks

Explanation:
There are 4 packages with 4 cheese sticks in each package.
Consider 1 package as 1 jump and 1 cheese stick as length of each jump.
Now, use a number line to find how many cheese sticks are in 4 packages.
Begin at 0. Skip count by 4s by drawing jumps on the number line.
No. of jumps made = 4
Length of each jump = 4
Multiply. 4 x 4 = 16
So, there are 16 cheese sticks in 4 packages.

Skip Count on a Number Line Lesson Check Page No 156

Question 1.
Louise skip counts by 4 on a number line to find 5 x 4. How many jumps should she draw on the number line?
Options:
a. 3
b. 4
c. 5
d. 9

Answer: c

Explanation:
Given no. of skip counts on a number line = 4
Product = no. of jumps x length of each jump
Given product = 5 x 4
From the given data, we can say that no of jumps = 5
So, no of jumps drawn on the number line = 5.

Question 2.
Theo needs 4 boards that are each 3 feet long to make bookshelves. How many feet of boards does he need altogether?
Options:
a. 12 feet
b. 7 feet
c. 4 feet
d. 3 feet

Answer: a

Explanation:
Given no. of boards = 4
Length of each board = 3 feet
Begin at 0. Skip count by 3’s.
Product = 4 x 3 = 12
So, Theo needs 12 feet of boards altogether.

Spiral Review

Question 3.
Estimate the sum.
5 1 8
+ 2 5 1
Options:
a. 200
b. 700
c. 800
d. 900

Answer: c

Explanation:
Use Compatible numbers.
518 —-> 500
251 —-> 300
Perform addition
GO Math Grade 3 Understand Multiplication Spiral Review Page No 156 Answer 9 - i

Question 4.
Which number would you put in a frequency table to show Go Math Grade 3 Answer Key Chapter 3 Understand Multiplication Skip Count on a Number Line img 12?
Options:
a. 5
b. 6
c. 7
d. 8

Answer: d

Explanation:
GO Math Grade 3 Understand Multiplication Spiral Review Page No 156 Answer 11 - i

Question 5.
A manager at a shoe store received an order for 346 pairs of shoes. What is 346 rounded to the nearest hundred?
Options:
a. 400
b. 350
c. 340
d. 300

Answer: d

Explanation:
Round 346 to nearest hundred.
GO Math Grade 3 Understand Multiplication Spiral Review Page No 156 Answer 12 - i
Since 4 < 5, the digit in the rounding place stays same. i.e, 3
Now, write zeros to the right of the rounding place (tens and ones place)
So, 346 rounded to nearest hundred is 300.

Question 6.
Toby is making a picture graph. Each picture of a book is equal to 2 books he has read. The row for Month 1 has 3 pictures of books. How many books did Toby read during Month 1?
Options:
a. 2
b. 3
c. 6
d. 8

Answer: c

Explanation:
Given, each picture  = 2 books
Row for month 1 = 3 pictures of books = 3 x 2 = 6
So, Toby read 6 books during month 1.

Mid-Chapter Checkpoint Page No 157

Choose the best term from the box.
Go Math Grade 3 Answer Key Chapter 3 Understand Multiplication Mid -Chapter Checkpoint img 13

Question 1.
When you combine equal groups, you can _______________ to find how many in all.
_________

Answer: Multiply

Explanation:
When equal groups are combined together, multiplication operation is performed to find the total.

Question 2.
The answer in a multiplication problem is called the _________________ .
_________

Answer: Product

Explanation:
The Product is the answer to a multiplication problem.

Question 3.
The numbers you multiply are called the ______________ .
_________

Answer: factors

Explanation:
The numbers which are multiplied are called factors.

Concepts and Skills

Count equal groups to find how many.

Question 4.
Go Math Grade 3 Answer Key Chapter 3 Understand Multiplication Mid -Chapter Checkpoint img 14
_______ groups of _______ ; _______ in all

Answer:
3 groups of 3; 9 in all

Explanation:
From the figure, we can say that there are 3 equal groups.
Each group has three counters.
So, the total no of counters are 9 in all.

Question 5.
Go Math Grade 3 Answer Key Chapter 3 Understand Multiplication Mid -Chapter Checkpoint img 15
_______ groups of _______ ; _______ in all

Answer:
4 groups of 5; 20 in all

Explanation:
From the figure, we can say that there are 4 equal groups.
Each group has five counters.
So, the total no of counters are 20 in all.

Question 6.
Go Math Grade 3 Answer Key Chapter 3 Understand Multiplication Mid -Chapter Checkpoint img 16
_______ groups of _______ ; _______ in all

Answer:
2 groups of 10; 20 in all

Explanation:
From the figure, we can say that there are 2 equal groups.
Each group has 10 counters.
So, the total no of counters are 20 in all.

Write related addition and multiplication sentences.

Question 7.
3 groups of 9
_______ + _______ + _______ = _______ ; _______ × _______ = _______

Answer: 9 + 9 + 9 = 27; 3 x 9 = 27

Explanation:
Draw 3 circles as groups.
Draw 9 counters in each circle.
Now, find the number of counters.
Addition Sentence: 3 groups of 9 = 9 + 9 + 9 = 27
Multiplication Sentence: 3 groups of 9 = 3 x 9 = 27

Question 8.
5 groups of 7
Type below:
________
Answer: 7 + 7 + 7 + 7 + 7 = 35; 5 x 7 = 35

Explanation:
Draw 5 circles as groups.
Draw 7 counters in each circle.
Now, find the number of counters.
Addition Sentence: 5 groups of 7 = 7 + 7 + 7 + 7 + 7 = 35
Multiplication Sentence: 5 groups of 7 = 5 x 7 = 35

Draw jumps on the number line to show equal groups.
Find the product.

Question 9.
6 groups of 3
Go Math Grade 3 Answer Key Chapter 3 Understand Multiplication Mid -Chapter Checkpoint img 17
_______ × _______ = _______

Answer: 6 x 3 = 18

Explanation:
Given, 6 groups of 3
No. of jumps = 6
Length of each jump = 3
Product = No. of jumps x length of each jump = 6 x 3 = 18

Mid-Chapter Checkpoint Page No 158

Question 10.
Beth’s mother cut some melons into equal slices. She put 4 slices each on 8 plates. Write a multiplication sentence to show the total number of melon slices she put on the plates.
Type below:
_________

Answer:  8 x 4 = 32

Explanation:
Given data,
There are 8 groups which is shown as plates.
Each group has 4 counters which is shown as slices.
Since, there are equal no. of slices in each plate, we can multiply to find the total number of melon slices.
Now, we can write multiplication sentence as 8 x 4 = 32.

Question 11.
Avery had 125 animal stickers. She gave 5 animal stickers to each of her 10 friends. How many animal stickers did she have left? What number sentences did you use to solve?
_________ stickers left

Answer: Multiplication and Subtraction Sentence, 75

Explanation:
Given, Total no. of animal stickers = 125
She gave 5 animal stickers to each of her 10 friends.
Use multiplicative sentence to find no. of stickers given to her friends.
Product = 5 x 10= 50
Total no. of stickers given to her friends = 50
Now, Total no. of stickers she left with = 125 – 50 = 75.

Question 12.
Matt made 2 equal groups of marbles. Write a multiplication sentence to show the total number of marbles.
Go Math Grade 3 Answer Key Chapter 3 Understand Multiplication Mid -Chapter Checkpoint img 18
Type below:
_________

Answer: 2 x 8 = 16

Explanation:
From the above figure, we can see
There are 2 equal groups.
Each group has 8 marbles.

Question 13.
Lindsey has 10 inches of ribbon. She buys another 3 lengths of ribbon, each 5 inches long. How much ribbon does she have now?
__________ inches of ribbon

Answer: 25 inches of ribbon

Explanation:
Given, Lindsey has 10 inches of ribbon.
She buys another 3 lengths of ribbon, each 5 inches long.
Use multiplication sentence to find length of 5 inches = 3 groups of 5
Product = 3 x 5 = 15 inches
Now, add to find how much ribbon lindsey has in all.
15 + 10 = 25
Total length of ribbon = 25 inches.

Question 14.
Jack’s birthday is in 4 weeks. How many days is it until Jack’s birthday? Describe how you could use a number line to solve
__________ days

Answer:  28

Explanation:
Given, Jack’s birthday is in 4 weeks.
Since each week has 7 days, length of each jump = 7
Use number line to find the no. of days.
No. of jumps = 4
Now, begin at o. Skip count by 7’s by drawing jumps on the number line.
Multiply no. of jumps and length of each jump
4 x 7 = 28
So, jack’s birthday is in 28 days.

Problem Solving Model Multiplication Page No 163

Draw a diagram to solve each problem.

Question 1.
Robert put some toy blocks into 3 rows. There are 5 blocks in each row. How many blocks are there in all?
15 blocks

Answer: 15 blocks

Explanation:
Given, Robert put some toy blocks into 3 rows.
Each row contain 5 blocks.
Now, use bar model to find the no. of blocks in all.
Write 5 in each box to show 5 blocks in each of the 3 rows.
Since, there are equal groups, we can multiply to find the number of blocks in all.
3 x 5 = 15 blocks
So, there are 15 blocks in all.

Question 2.
Mr. Fernandez is putting tiles on his kitchen floor. There are 2 rows with 9 tiles in each row. How many tiles are there in all?
___________ tiles

Answer: 18 tiles

Explanation:
Given, Mr. Fernandez is putting tiles on his kitchen floor.
There are 2 rows.
Each row contain 9 tiles.
Now, use bar model to find the no. of tiles in all.
Write 9 in each box to show 9 tiles in each of the 2 rows.
Since, there are equal groups, we can multiply to find the number of tiles in all.
2 x 9 = 18 tiles
So, there are 18 tiles in all.

Question 3.
In Jillian’s garden, there are 3 rows of carrots, 2 rows of string beans, and 1 row of peas. There are 8 plants in each row. How many plants are there in all?
___________ plants

Answer: 48 plants

Explanation:
Given, there are 3 rows of carrots, 2 rows of string beans, and 1 rows of peas.
Total no. of rows = 6
There are 8 plants in each row.
Now, use bar model to find the no. of plants in all.
Write 8 in each box to show 8 plants in each of the 6 rows.
Since, there are equal groups, we can multiply to find the number of plants in all.
6 x 8 = 48 plants
So, there are 48 plants in all.

Question 4.
In Sorhab’s classroom, there are 3 rows with 7 desks in each row. How many desks are there in all?
_________ desks

Answer: 21 desks

Explanation:
Given, there are 3 rows.
Each row contain 7 desks.
Now, use bar model to find the no. of desks in all.
Write 7 in each box to show 7 desks in each of the 3 rows.
Since, there are equal groups, we can multiply to find the number of desks in all.
3 x 7 = 21 desks
So, there are  21 desks in all.

Question 5.
Maya visits the movie rental store. On one wall, there are 6 DVDs on each of 5 shelves. On another wall, there are 4 DVDs on each of 4 shelves. How many DVDs are there in all?
___________ DVDs

Answer: 46 DVD’s

Explanation:
Given, there are 2 walls.
On one wall, there are 5 shelves.
Each shelf has 6 DVDs.
Now, use bar model to find the no. of DVDs on one wall.
Write 6 in each box to show 6 DVDs on each of the 5 shelves.
Since, there are equal groups, we can multiply to find the number of DVDs on one wall.
5 x 6 = 30 DVDs
So, there are 30 DVD’s on one wall.
On another wall, there are 4 shelves.
Each shelf has 4 DVDs.
Now, use bar model to find the no. of DVDs on another wall.
Write 4 in each box to show 4 DVDs on each of the 4 shelves.
Since, there are equal groups, we can multiply to find the number of DVDs on another wall.
4 x 4 = 16 DVDs
So, there are 16 DVD’s on another wall.
Now, find the total no. of DVD’s on both all the walls.
Add. 30 + 16 = 46 DVDs.
So, there are 46 DVD’s in all.

Question 6.
The media center at Josh’s school has a computer area. The first 4 rows have 6 computers each. The fifth row has 4 computers. How many computers are there in all?
___________ computers

Answer: 28 computers

Explanation:
Given, there are 4 rows.
Each row contain 6 computers.
Now, use bar model to find the no. of computers in all.
Write 6 in each box to show 6 computers in each of the 4 rows.
Since, there are equal groups, we can multiply to find the number of computers in all.
4 x 6 = 24 computers
So, there are 24 computers in all.
There is another fifth row with 4 computers.
Now, use bar model to find the no. of computers in all.
No. of computers in fifth row = 4 x 1 = 4 computers.
Add the computers in all rows to find how many in all.
24 + 4 = 28
So, there are 28 computers in all.

Model Multiplication Lesson Check Page No 164

Question 1.
There are 5 shelves of video games in a video store. There are 6 video games on each shelf. How many video games are there in all?
Options:
a. 35
b. 30
c. 20
d. 11

Answer: b

Explanation:
Given, there are 5 shelves.
Each shelf contain 6 video games.
Now, use bar model to find the no. of video games in all.
Write 6 in each box to show 6 video games in each of the 5 shelves.
Since, there are equal groups, we can multiply to find the number of shelves in all.
5 x 6 = 30 shelves
So, there are 30 shelves in all.

Question 2.
Ken watches a marching band. He sees 2 rows of flute players. Six people are in each row. He sees 8 trombone players. How many flute or trombone players does Ken see?
Options:
a. 2
b. 6
c. 16
d. 20

Answer: d

Explanation:
Given, there are 6 flute players in each row.
There are 2 rows.
Now, use bar model to find the no. of flute players in all.
Write 6 in each box to show 6 flute players in each of the 2 rows.
Since, there are equal groups, we can multiply to find the number of flute players in all.
2 x 6 = 12 flute players
Given, there are 8 trombone players.
Now, add flute and trombone players
12 + 8 = 20
So, there are 20 flute or trombone players.

Spiral Review

Question 3.
What is the sum of 438 and 382?
Options:
a. 720
b. 810
c. 820
d. 910

Answer: c

Explanation:
Step 1:
Add ones. Regroup ones as tens and ones.
GO Math Grade 3 Understand Multiplication Spiral Review Page No 164 Answer 13 - i
Step 2:
Add tens. Regroup tens as hundreds and tens.
GO Math Grade 3 Understand Multiplication Spiral Review Page No 164 Answer 13 - ii
Step 3:
Add the hundreds.
GO Math Grade 3 Understand Multiplication Spiral Review Page No 164 Answer 13 - iii

Question 4.
Estimate the sum.
6 2 2
+ 8 4
———
Options:
a. 500
b. 600
c. 700
d. 800

Answer: c

Explanation:
Use compatible numbers.
622 —-> 600
84  —–> 100
GO Math Grade 3 Understand Multiplication Spiral Review Page No 164 Answer 14 - i

Question 5.
Francine uses 167 silver balloons and 182 gold balloons for her store party. How many silver and gold balloons in all does Francine use?
Options:
a. 15
b. 345
c. 349
d. 359

Answer: c

Explanation:
Given, Silver balloons = 167
Gold balloons = 182
Add Silver and gold balloons to find total in all.
Step 1: Add ones.
GO Math Grade 3 Understand Multiplication Spiral Review Page No 164 Answer 15 - i
Step 2: Add tens. Regroup tens as hundreds and tens.
GO Math Grade 3 Understand Multiplication Spiral Review Page No 164 Answer 15 - ii
Step 3: Add hundreds.
GO Math Grade 3 Understand Multiplication Spiral Review Page No 164 Answer 15 - iii
So, total no. of silver and gold balloons in all does Francine use = 349

Question 6.
Yoshi is making a picture graph. Each picture of a soccer ball stands for two goals he scored for his team. The row for January has 9 soccer balls. How many goals did Yoshi score during January?
Options:
a. 18
b. 16
c. 11
d. 9

Answer: a

Explanation:
Given, each picture = 2 goals
Row for January = 9 soccer balls
Consider each picture as 9 soccer balls.
Now, find the goals did Yoshi score during January.
9 x 2 = 18
So, 18 goals did Yoshi score during January.

Model with Arrays Page No 169

Write a multiplication sentence for the array.

Question 1.
Go Math Grade 3 Answer Key Chapter 3 Understand Multiplication Model with Arrays img 19
3 × 7 = 21

Answer: 3 x 7 = 21

Explanation:
In the above figure, we can see there are same no. of tiles in each row.
There are 3 rows with 7 tiles in each row.
Now, multiplication sentence for array can be written as number of rows x no. of tiles in each row.
Multiply. 3 x 7 = 21

Question 2.
Go Math Grade 3 Answer Key Chapter 3 Understand Multiplication Model with Arrays img 20
2 × 5 = _______

Answer: 10

Explanation:
In the above figure, we can see there are same no. of tiles in each row.
There are 2 rows with 5 tiles in each row.
Now, multiplication sentence for array can be written as number of rows x no. of tiles in each row.
Multiply. 2 x 5 = 10

Draw an array to find the product.

Question 3.
4 × 2 = _______

Answer: 8

Explanation:
Make an array by placing the same no. of tiles in each row.
From the given data, make an array with 4 rows of 2 tiles.
Now, draw an array.
In the above figure, we can see there are same no. of tiles in each row.
There are 4 rows with 2 tiles in each row.
Now, multiplication sentence for array can be written as number of rows x no. of tiles in each row.
Multiply. 4 x 2 = 8

Question 4.
4 × 4 = _______

Answer: 16

Explanation:
Make an array by placing the same no. of tiles in each row.
From the given data, make an array with 4 rows of 4 tiles.
Now, draw an array.
In the above figure, we can see there are same no. of tiles in each row.
There are 4 rows with 4 tiles in each row.
Now, multiplication sentence for array can be written as number of rows x no. of tiles in each row.
Multiply. 4 x 4 = 16

Question 5.
3 × 2 = _________

Answer: 6

Explanation:
Make an array by placing the same no. of tiles in each row.
From the given data, make an array with 3 rows of 2 tiles.
Now, draw an array.
In the above figure, we can see there are same no. of tiles in each row.
There are 3 rows with 2 tiles in each row.
Now, multiplication sentence for array can be written as number of rows x no. of tiles in each row.
Multiply. 3 x 2 = 6

Question 6.
2 × 8 = _______

Answer: 16

Explanation:
Make an array by placing the same no. of tiles in each row.
From the given data, make an array with 2 rows of 8 tiles.
Now, draw an array.
In the above figure, we can see there are same no. of tiles in each row.
There are 2 rows with 8 tiles in each row.
Now, multiplication sentence for array can be written as number of rows x no. of tiles in each row.
Multiply. 2 x 8 = 16

Problem Solving

Question 7.
Lenny is moving tables in the school cafeteria. He places all the tables in a 7 × 4 array. How many tables are in the cafeteria?
_________ tables

Answer: 28 tables

Explanation:
Given array = 7 x 4
Now, make an array with 7 rows of 4 tiles.
Draw an array and find the no. of tables.
Multiply. 7 x 4 = 28
So, there are 28 tables in the cafeteria.

Question 8.
Ms. DiMeo directs the school choir. She has the singers stand in 3 rows. There are 8 singers in each row. How many singers are there in all?
_________ singers

Answer: 24 singers

Explanation:
Given, no. of singers in each row = 8
No. of rows = 3
Now, make an array with 3 rows of 8 singers.
Find the no. of singers by multiplying no. of rows with singers.
Multiply. 3 x 8 = 24 singers.
So, there are 24 singers in all.

Model with Arrays Lesson Check Page No 170

Question 1.
What multiplication sentence does this array show?
Go Math Grade 3 Answer Key Chapter 3 Understand Multiplication Model with Arrays img 21
Options:
a. 2 × 3 = 6
b. 6 × 3 = 18
c. 3 × 4 = 12
d. 3 × 5 = 15

Answer: d

Explanation:
From the above figure, we can see that the array consists of 3 rows and 5 tiles.
Now, the multiplications sentence is 3 x 5 = 15

Question 2.
What multiplication sentence does this array show?
Go Math Grade 3 Answer Key Chapter 3 Understand Multiplication Model with Arrays img 22
Options:
a. 3 × 9 = 27
b. 3 × 8 = 24
c. 3 × 7 = 21
d. 4 × 5 = 20

Answer: a

Explanation:
From the above figure, we can see that the array consists of 3 rows and 9 tiles.
Now, the multiplications sentence is 3 x 9 = 27

Spiral Review

Question 3.
Use the table to find who traveled 700 miles farther than Paul during summer vacation.
Go Math Grade 3 Answer Key Chapter 3 Understand Multiplication Model with Arrays img 23
Options:
a. Andrew
b. Bonnie
c. Susan
d. Tara

Answer: d

Explanation:
From the table, we can say that paul travelled 233 miles.
Now, use break apart strategy to find sums.
Paul –>      233 = 200 + 30 + 3
Andrew –> 380 = 300 + 80+ 0
Bonnie –>  790 = 700 + 90+ 0
Tara –>      933 = 900 + 30+ 3
Susan –>   853 = 800 + 50+ 3
From the above sums, we can say that Tara travelled farther miles than paul.

Question 4.
Use the bar graph to find what hair color most students have.
Go Math Grade 3 Answer Key Chapter 3 Understand Multiplication Model with Arrays img 24
Options:
a. Brown
b. Black
c. Blond
d. Red

Answer: a

Explanation:
The title shows hair color for students.
The length of bar tells the no. of students had each color.
Now, Find out the which color maximum no. of students have.
We can see that the brown color is the highest.

Question 5.
Spencer ordered 235 cans of tomatoes to make salsa for the festival. What is 235 rounded to the nearest ten?
Options:
a. 200
b. 230
c. 240
d. 300

Answer: c

Explanation:
Round 235 to nearest ten.
The digit to the right of rounding place is 5.
So, the digit in the rounding place is increased by one.
Write zero to the round of rounding digit.
Now, it becomes 240.

Question 6.
Which bar would be the longest on a bar graph of the data?
Go Math Grade 3 Answer Key Chapter 3 Understand Multiplication Model with Arrays img 25
Options:
a. Cheese
b. Pepperoni
c. Vegetable
d. Sausage

Answer: a

Explanation:
Make a bar graph.
Step 1:
Write a title at the top to tell what the graph is about. Label the side of the graph to tell about the bars. Label the bottom of the graph to explain what the numbers tell.
Step 2:
Choose numbers for the bottom of the graph so that most of the bars will end on a line. Since the least number is 1 and the greatest number is 5, make the scale 0-5.
Step 3: Draw and shade a bar to show the number for each pizza topping.
From the bar graph, we can say that the cheese bar would be the longest on a bar graph of the data.

Commutative Property of Multiplication Page No 175

Write a multiplication sentence for the model. Then use the Commutative Property of Multiplication to write a related multiplication sentence.

Question 1.
Go Math Grade 3 Answer Key Chapter 3 Understand Multiplication Commutative Property of Multiplication img 26
5 × 2 = 10
2 × 5 = 10

Answer: 5 × 2 = 10
2 × 5 = 10

Explanation:
From the figure, we can say that there are 5 rows.
There are two tiles in each row.
From the given array, the multiplication sentence can be written as 5 x 2 = 10
The Commutative property of multiplication have the same factors in different order.
So, by using commutative property, Multiplication sentence is written as 2 x 5 = 10.

Question 2.
Go Math Grade 3 Answer Key Chapter 3 Understand Multiplication Commutative Property of Multiplication img 27
______ × _____ = _______
______ × _____ = _______

Answer:
6 x 4 = 24
4 x 6 = 24

Explanation:
From the figure, we can say that there are 6 rows.
There are 4 tiles in each row.
From the given array, the multiplication sentence can be written as 6 x 4 = 24
The Commutative property of multiplication have the same factors in different order.
So, by using commutative property, Multiplication sentence is written as 4 x 6 = 24.

Question 3.
Go Math Grade 3 Answer Key Chapter 3 Understand Multiplication Commutative Property of Multiplication img 28
______ × _____ = _______
______ × _____ = _______

Answer:
3 x 4 = 12
4 x 3 = 12

Explanation:
From the figure, we can say that there are 3 equal groups.
There are 4 counters in each group.
From the given array, the multiplication sentence can be written as 3 x 4 = 12
The Commutative property of multiplication have the same factors in different order.
So, by using commutative property, Multiplication sentence is written as 4 x 3 = 12.

Question 4.
Go Math Grade 3 Answer Key Chapter 3 Understand Multiplication Commutative Property of Multiplication img 29
______ × _____ = _______
______ × _____ = _______

Answer:
2 x 6 = 12
6 x 2 = 12

Explanation:
From the figure, we can say that there are 2 equal groups.
There are 6 counters in each group.
From the given array, the multiplication sentence can be written as 2 x 6 = 12
The Commutative property of multiplication have the same factors in different order.
So, by using commutative property, Multiplication sentence is written as 6 x 2 = 12.

Problem Solving

Question 5.
A garden store sells trays of plants. Each tray holds 2 rows of 8 plants. How many plants are in one tray?
___________ plants

Answer:
16 plants

Explanation:
Given,there are 8 plants.
Each tray holds 2 rows of 8 plants.
Now, by using multiplicative sentence or commutative property, we can find the no. of plants in one tray.
Multiplicative Sentence : 2 x 8 = 16 plants
Commutative property of multiplication: 8 x 2 = 16
So, there are 16 plants in one tray.

Question 6.
Jeff collects toy cars. They are displayed in a case that has 4 rows. There are 6 cars in each row. How many cars does Jeff have?
________ cars

Answer:
24 cars

Explanation:
Given, toy cars are displayed in 4 rows.
There are 6 cars in each row.
Now, by using multiplicative sentence or commutative property, we can find the no. of cars in each row.
Multiplicative Sentence : 6 x 4 = 24 cars.
Commutative property of multiplication: 4 x 6 = 24 cars.

Commutative Property of Multiplication Lesson Check Page No 176

Question 1.
Which is an example of the Commutative Property of Multiplication?
Options:
a. 8 × 4 = 8 × 4
b. 4 × 2 = 2 × 4
c. 2 × 8 = 4 × 4
d. 2 + 4 = 2 × 4

Answer: b

Explanation:
The Commutative Property of Multiplication states that when you change the order
of the factors, the product stays the same. From the given options, 4 x 2 = 2 x 4 is an example of commutative property of multiplication.

Question 2.
What factor makes the number sentence true?
7 × 4 = ■ × 7
Options:
a. 2
b. 4
c. 7
d. 28

Answer: b

Explanation:
The Commutative Property of Multiplication states that when you change the order
of the factors, the product stays the same. From the question, we can see that 7 x 4 = 28;
Then 4 x 7 = 28

Spiral Review

Question 3.
Ms. Williams drove 149 miles on Thursday and 159 miles on Friday. About how many miles did she drive altogether the two days?
Options:
a. about 150 miles
b. about 200 miles
c. about 300 miles
d. about 400 miles

Answer: c

Explanation:

Given, miles driven by Ms. Williams on Thursday = 149
Miles driven by Ms. Williams on Friday = 159
By using compatible numbers, we can find the no. of miles did she drive altogether the two days.
149 —> 150
159 —> 150

GO Math Grade 3 Understand Multiplication Spiral Review Page No 176 Answer 16 - i
So, the no. of miles driven altogether is about 300 miles.

Question 4.
Inez has 699 pennies and 198 nickels. Estimate how many more pennies than nickels she has.
Options:
a. about 500
b. about 600
c. about 700
d. about 900

Answer: a

Explanation:

Given, there are 699 pennies and 198 nickels.
Now, estimate to find how many more pennies than nickels she has.
Use compatible numbers to compute it.
699 —> 700
198 —> 200
GO Math Grade 3 Understand Multiplication Spiral Review Page No 164 Answer 17 - i
So, Inez has 500 more pennies than nickels.

Question 5.
This year, the parade had 127 floats. That is 34 fewer floats than last year. How many floats were in the parade last year?
Options:
a. 161
b. 151
c. 103
d. 93

Answer: a

Explanation:
Given, no. of floats this year = 127
No. of floats last year are 34 greater than this year.
Now, estimate the no. of floats in the parade last year
Perform Addition operation
Step 1: Add ones. Regroup the ones as tens and ones.

GO Math Grade 3 Understand Multiplication Spiral Review Page No 176 Answer 18 - i

Step 2: Add tens.

GO Math Grade 3 Understand Multiplication Spiral Review Page No 176 Answer 18 - ii
Step 3: Add hundreds.

GO Math Grade 3 Understand Multiplication Spiral Review Page No 176 Answer 18 - iii

The total no. of floats in the parade last year = 161

Question 6.
Jeremy made a tally table to record how his friends voted for their favorite pet. His table shows Go Math Grade 3 Answer Key Chapter 3 Understand Multiplication Commutative Property of Multiplication img 30 next to Dog. How many friends voted for dog?
Options:
a. 6
b. 8
c. 10
d. 12

Answer: d

Explanation:
Count the tally marks. It shows 12. So, the no. of friends voted for dog = 12

Multiply with 1 and 0 Page No 181

Find the product.

Question 1.
1 × 4 = 4

Answer: 4

Explanation:
The Identity Property of Multiplication states that the product of any numberand 1 is that number.
So, 1 x 4 = 4

Question 2.
0 × 8 = _______

Answer: 0

Explanation:
The Zero Property of Multiplication states that the product of zero and any number is zero
So, 0 x 8 = 0

Question 3.
0 × 4 = _______

Answer: 0

Explanation:
The Zero Property of Multiplication states that the product of zero and any number is zero.
So, 0 x 4 = 0

Question 4.
1 × 6 = _______

Answer: 6

Explanation:
The Identity Property of Multiplication states that the product of any number and 1 is that number.
So, 1 x 6 = 6

Question 5.
3 × 0 = _______

Answer: 0

The Zero Property of Multiplication states that the product of zero and any number is zero.
So, 3 x 0 = 0

Question 6.
0 × 9 = _______

Answer: 0

The Zero Property of Multiplication states that the product of zero and any number is zero.
So, 0 x 9 = 0

Question 7.
8 × 1 = _______

Answer: 8

Explanation:
The Identity Property of Multiplication states that the product of any number and 1 is that number.
So, 8 x 1 = 6

Question 8.
1 × 2 = _______

Answer: 2

Explanation:
The Identity Property of Multiplication states that the product of any number and 1 is that number.
So, 1 x 2 = 2

Question 9.
0 × 6 = _______

Answer: 0

The Zero Property of Multiplication states that the product of zero and any number is zero.
So, 0 x 6 = 0

Question 10.
4 × 0 = _______

Answer: 0

The Zero Property of Multiplication states that the product of zero and any number is zero.
So, 4 x 0 = 0

Question 11.
7 × 1 = _______

Answer: 7

Explanation:
The Identity Property of Multiplication states that the product of any number and 1 is that number.
So, 7 x 1 = 7

Question 12.
1 × 5 = _______

Answer: 5

Explanation:
The Identity Property of Multiplication states that the product of any number and 1 is that number.
So, 1 x 5 = 5

Question 13.
3 × 1 = _______

Answer: 3

Explanation:
The Identity Property of Multiplication states that the product of any number and 1 is that number.
So, 3 x 1 = 3

Question 14.
0 × 7 = _______

Answer: 0

The Zero Property of Multiplication states that the product of zero and any number is zero.
So, 0 x 7 = 0

Question 15.
1 × 9 = _______

Answer: 9

Explanation:
The Identity Property of Multiplication states that the product of any number and 1 is that number.
So, 1 x 9 = 9

Question 16.
5 × 0 = _______

Answer: 0

The Zero Property of Multiplication states that the product of zero and any number is zero.
So, 5 x 0 = 0

Question 17.
10 × 1 = _______

Answer: 10

Explanation:
The Identity Property of Multiplication states that the product of any number and 1 is that number.
So, 10 x 1 = 10

Question 18.
2 × 0 = _______

Answer: 0

The Zero Property of Multiplication states that the product of zero and any number is zero.
So, 2 x 0 = 0

Question 19.
5 × 1 = _______

Answer: 5

Explanation:
The Identity Property of Multiplication states that the product of any number and 1 is that number.
So, 5 x 1 = 5

Question 20.
1 × 0 = _______

Answer: 0

Explanation:
The Zero Property of Multiplication states that the product of zero and any number is zero.
So, 1 x 0 = 0

Question 21.
0 × 0 = _______

Answer: 0

Explanation:
The Zero Property of Multiplication states that the product of zero and any number is zero.
So, 1 x 0 = 0

Question 22.
1 × 3 = _______

Answer: 3

Explanation:
The Identity Property of Multiplication states that the product of any number and 1 is that number.
So, 1 x 3 = 3

Question 23.
9 × 0 = _______

Answer: 0

Explanation:
The Zero Property of Multiplication states that the product of zero and any number is zero.
So, 9 x 0 = 0

Question 24.
1 × 1 = _______

Answer: 1

Explanation:
The Identity Property of Multiplication states that the product of any number and 1 is that number.
So, 1 x 1 = 1

Problem Solving

Question 25.
Peter is in the school play. His teacher gave 1 copy of the play to each of 6 students. How many copies of the play did the teacher hand out?
_________ copy

Answer: 6 copies

Explanation:
Given, No. of students = 6
Copies given to each student = 1
No. of copies of the play did teacher hand out = 1 x 6 = 6

Question 26.
There are 4 egg cartons on the table. There are 0 eggs in each carton. How many eggs are there in all?
_________ eggs

Answer: 0 eggs

Explanation:
Given, there are 4 egg cartons
There are 0 eggs in each carton
No. of eggs in all = ?
0 x 4 = 0 eggs.
No. of eggs in all = 0

Multiply with 1 and 0 Lesson Check Page No 182

Question 1.
There are 0 bicycles in each bicycle rack. If there are 8 bicycle racks, how many bicycles are there in all?
Options:
a. 80
b. 8
c. 1
d. 0

Answer: d

Explanation:
There are 0 bicycles in each bicycle rack.
There are 8 bicycle racks.
No. of bicycles in all = 8 x 0 = 0

Question 2.
What is the product?
1 × 0 = _______
Options:
a. 0
b. 1
c. 10
d. 11

Answer: a

Explanation:
The Zero Property of Multiplication states that the product of zero and any number is zero.
So, 1 x 0 = 0

Spiral Review

Question 3.
Mr. Ellis drove 197 miles on Monday and 168 miles on Tuesday. How many miles did he drive in all?
Options:
a. 29 miles
b. 255 miles
c. 365 miles
d. 400 miles

Answer: c

Explanation:
Given, no. of miles driven on Monday = 197
no. of miles driven on Tuesday = 168
Perform addition to find the no. of miles driven in all

Question 4.
What multiplication sentence does the array show?
■ ■ ■ ■ ■ ■
Options:
a. 1 × 6 = 6
b. 3 × 2 = 6
c. 2 × 6 = 12
d. 5 + 1 = 6

Answer: a

Explanation:
Given array shows 1 row with 6 tiles.
So, multiplication sentence can be written as 1 x 6 = 6

Use the bar graph for 5–6.
Go Math Grade 3 Answer Key Chapter 3 Understand Multiplication Multiply with 1 and 0 img 31

Question 5.
How many cars were washed on Friday and Saturday combined?
Options:
a. 55
b. 80
c. 90
d. 120

Answer: b
Explanation:
From the bar graph, we can see that no. of cars washed on Friday = 25
No. of cars washed on Saturday = 55
Total no. of cars washed on Friday and Saturday = 80

Question 6.
How many more cars were washed on Saturday than on Sunday?
Options:
a. 95
b. 30
c. 25
d. 15

Answer: d

Explanation:
From the bar graph, we can see that cars washed on Saturday = 55
Cars washed on Sunday = 40
55-40 = 15
15 more cars washed on Saturday than on Sunday.

Chapter 3 Review Test Page No 183

Question 1.
There are 3 boats on the lake. Six people ride in each boat. How many people ride in the boats? Draw circles to model the problem and explain how to solve it.
_________ people

Answer: 18 boats

Explanation:
Given, no. of boats on the lake = 3
No. of people ride in each boat = 6
Total no. of people ride in the boats = 3 groups of 6
3 x 6 = 18
So, 18 people ride in the boats.

Question 2.
Nadia has 4 sheets of stickers. There are 8 stickers on each sheet. She wrote this number sentence to represent the total number of stickers.

4 × 8 = 32

What is a related number sentence that also represents the total number of stickers she has?
Options:
a. 8 + 4 =■
b. 4 + 4 + 4 + 4 = ■
c. 8 × 8 = ■
d. 8 × 4 = ■

Answer: d

Explanation:
Given, there are are 4 sheets of stickers.
Each sheet has 8 stickers.
Given total no. of stickers represented as = 4 x 8 = 32
By using commutative property of multiplication, related number sentence can be represented as 8 x 4 =32

Question 3.
Lindsay went hiking for two days in Yellowstone National Park. The first jump on the number line shows how many birds she saw the first day. She saw the same number of birds the next day.
Go Math Grade 3 Answer Key Chapter 3 Understand Multiplication Review/Test img 32
Write the multiplication sentence that is shown on the number line.
______ × _______ = _______

Answer: 2 x 8 = 16

Explanation:
Given, the total no. of jumps = 2
From the figure, we can see that there are two jumps which begins at 0 and skip count by 8’s.
Product = No. of jumps x Length of each jump.
Now, multiplication sentence can be written as 2 x 8 = 16

Chapter 3 Review Test Page No 184

Question 4.
Paco drew an array to show the number of desks in his classroom. Write a multiplication sentence for the array.
Go Math Grade 3 Answer Key Chapter 3 Understand Multiplication Review/Test img 33
________ desks

Answer: 21 desks

Explanation:
In the given array, there are three rows.
Each row has 7 desks.
Now, multiplication sentence can be written as no. of rows x no. of desks in each row.
Multiply. 3 x 7 = 21 desks

Question 5.
Alondra makes 4 necklaces. She uses 5 beads on each necklace. For numbers 5a–5d, choose Yes or No to tell if the number sentence could be used to find the number of beads Alondra uses.
a. 4 × 5 = ■
i. yes
ii. no

Answer: i

Explanation: Given, there are 4 necklaces.
Each necklace uses 5 beads.
Now, number sentence (multiplication) can be written as 4 groups of 5 = 4 x 5
So, the answer is yes

Question 5.
b. 4 + 4 + 4 + 4 = ■
i. yes
ii. no

Answer: ii

Given, there are 4 necklaces.
Each necklace uses 5 beads.
Now, number sentence (addition) can be written as 4 groups of 5 = 5 + 5 + 5 + 5
So, the answer is no.

Question 5.
c. 5 + 5 + 5 + 5 = ■
i. yes
ii. no

Answer: i

Given, Given, there are 4 necklaces.
Each necklace uses 5 beads.
Now, number sentence (addition) can be written as 4 groups of 5 = 5 + 5 + 5 + 5
So, the answer is yes.

Question 5.
d. 5 + 4 = ■
i. yes
ii. no

Answer: ii

Given, there are 4 necklaces.
Each necklace uses 5 beads.
Now, number sentence (addition) can be written as 4 groups of 5 = 4 x 5
Using commutative property of multiplication, it can be written as 5 x 4, but given question is 5 + 4.
So, the answer is no.

Question 6.
John sold 3 baskets of apples at the market. Each basket contained 9 apples. How many apples did John sell? Make a bar model to solve the problem.
Go Math Grade 3 Answer Key Chapter 3 Understand Multiplication Review/Test img 34
_______ apples

Answer:27 apples

Explanation:
Given, there are 3 baskets of apples.
Each basket contains 9 apples.
Draw a bar model with 3 boxes to show 3 baskets.
Write 9 in each box to show 9 apples.
Since, there are equal groups, we can multiply to find No. of apples john sold.
3 x 9 = 27 apples.

Chapter 3 Review Test Page No 185

Question 7.
Select the number sentences that show the Commutative Property of Multiplication. Mark all that apply.
Options:
a. 3 × 2 = 2 × 3
b. 4 × 9 = 4 × 9
c. 5 × 0 = 0
d. 6 × 1 = 1 × 6
e. 7 × 2 = 14 × 1

Answer: a

Explanation:
Commutative Property of Multiplication states that when you change the order of the factors, the product stays the same.
So, 3 x 2 = 2 x 3 is the answer.

Question 8.
A waiter carried 6 baskets with 5 dinner rolls in each basket. How many dinner rolls did he carry? Show your work.
___________ dinner rolls

Answer: 30 dinner rolls

Explanation:
Given, there are 6 baskets.
Each basket has 5 dinner rolls.
No. of dinner rolls = 6 x 5 = 30 dinner rolls.

Question 9.
Sonya needs 3 equal lengths of wire to make 3 bracelets. The jump on the number line shows the length of one wire in inches. How many inches of wire will Sonya need to make the 3 bracelets?
Go Math Grade 3 Answer Key Chapter 3 Understand Multiplication Review/Test img 35
_________ inches

Answer: 18 inches

Explanation:
Given, Length of one wire = jump on the number line
From the figure, we can see that length of wire = 6 inches
3 equal lengths of wire is required for 3 bracelets.
So, no. of jumps = 3
length of each jump = 6 inches
Multiply. 3 x 6 = 18
So, sonya need 18 inches of wire to make 3 bracelets.

Question 10.
Josh has 4 dogs. Each dog gets 2 dog biscuits every day. How many biscuits will Josh need for all of his dogs for Saturday and Sunday?
__________ biscuits

Answer: 16 biscuits

Explanation:
Given, there are 4 dogs.
Each dog gets 2 biscuits every day.
No. of biscuits required for all dogs every day = 4 x 2 = 8
No. of biscuits need for all dogs for Saturday and Sunday = 8 x 2 = 16 biscuits.

Chapter 3 Review Test Page No 186

Question 11.
Jorge displayed 28 cans of paint on a shelf in his store.
Go Math Grade 3 Answer Key Chapter 3 Understand Multiplication Review/Test img 36
Select other ways Jorge could arrange the same number of cans. Mark all that apply.
Options:
a. 2 rows of 14
b. 1 row of 28
c. 6 rows of 5
d. 8 rows of 3
e. 7 rows of 4

Answer: e

Explanation:
Given, There are 28 cans of paint on a shelf in a store.
In the given array, there are 4 rows with 7 cans in each row.
So, there are 4 rows of 7.
By using commutative property of multiplication, it can be arranged in 7 rows of 4.

Question 12.Choose the number that makes the statement true. The product of any number and Go Math Grade 3 Answer Key Chapter 3 Understand Multiplication Review/Test img 37 is zero.

Answer: 0

Explanation:
The Zero Property of Multiplication states that the product of any number and zero is zero.
So, the answer is 0.

Question 13.
James made this array to show that 3 × 5 = 15.
Go Math Grade 3 Answer Key Chapter 3 Understand Multiplication Review/Test img 38
Part A
James says that 5 × 3 = 15. Is James correct? Draw an array to explain your answer.
a. yes
b. no

Answer: Yes.

Explanation: Draw an array to show 5 rows with 3 tiles in each row.
Then, Multiplication sentence can be written as no. of rows x no. of tiles in each row
5 x 3 = 15

Question 13.
Part B
Which number property supports your answer?
________

Answer: Commutative Property of Multiplication

Explanation:
Commutative Property of Multiplication supports this answer. Because, it states that the product of any tow factors in reverse order remains the same.

Chapter 3 Review Test Page No 187

Question 14.
Julio has a collection of coins. He puts the coins in 2 equal groups. There are 6 coins in each group. How many coins does Julio have? Use the number line to show your work.
Go Math Grade 3 Answer Key Chapter 3 Understand Multiplication Review/Test img 39
________ coins

Answer: 12 coins

Explanation:
There are 2 equal groups.
There are 6 coins in each group.
Find total no. of coins by using number line.
Begin at 0. Skip count by 6s by drawing jumps on the number line.
So, no. of jumps = 2
Length of each jump = 6
Multiply. 2 x 6 = 12
Total no. of coins Julio have = 12

Question 15.
Landon collects trading cards.
Part A
Yesterday, Landon sorted his trading cards into 4 groups. Each group had 7 cards. Draw a bar model to show Landon’s cards. How many cards does he have?
________ trading cards

Answer: 28 trading cards

Explanation:
No. of groups = 4
Each group has 7 cards.
Draw a bar model to show the cards.
Draw 4 boxes in the bar model.
Write 7 in each box to show 7 cards.
Since, there are equal groups, multiply to find the no. of cards.
Multiply. 4 x 7 = 28 cards.

Question 15.
Part B
Landon buys 3 more packs of trading cards today. Each pack has 8 cards. Write a multiplication sentence to show how many cards Landon buys today. Then find how many cards Landon has now. Show your work.
Type below:
_________

Answer:52 cards

Explanation:
Given, Landon buys 3 more packs of trading cards
Each pack has 8 cards.
Multiplication Sentence = 3 x 8 = 24.
Total no. of cards = 28 + 24 = 52 cards.

Question 16.
A unicycle has only 1 wheel. Write a multiplication sentence to show how many wheels there are on 9 unicycles.
_______ × _______ = _______

Answer: 9 x 1 = 9

Explanation:
Given, there are 9 unicycles.
Each unicycle has only one wheel.
So, Multiplication Sentence can be written as 9 x 1 = 9.

Question 17.
Carlos spent 5 minutes working on each of 8 math problems. He can use 8 × 5 to find the total amount of time he spent on the problems.
For numbers 17a–17d, choose Yes or No to show which are equal to 8 × 5.
a. 8 + 5
i. yes
ii. no

Answer: No

Explanation:
Given, Carlos spent 5 minutes working on each of 8 math problems.
Given Sentence = 8 x 5
As there are 8 problems with 5 minutes spent on each, the total time can be find out using multiplication sentence.
So, the answer is no.

Question 17.
b. 5 + 5 + 5 + 5 + 5
i. yes
ii. no

Answer: No

Given, Carlos spent 5 minutes working on each of 8 math problems.
Given Sentence = 8 x 5 = 40
But given option 5 + 5 + 5 + 5 + 5 = 25.
So, the answer is no

Question 17.
c. 8 + 8 + 8 + 8 + 8
i. yes
ii. no

Answer: Yes
Explanation:
Given, Carlos spent 5 minutes working on each of 8 math problems.
Given Sentence = 8 x 5 = 40
Given option = 8 + 8 + 8 + 8 + 8 = 40
So, the answer is yes

Question 17.
d. 5 + 5 + 5 + 5 + 5 + 5 + 5 + 5
i. yes
ii. no

Answer: Yes
Explanation:
Given, Carlos spent 5 minutes working on each of 8 math problems.
Given Sentence = 8 x 5 = 40
Given option = 5 + 5 + 5 + 5 + 5 + 5 + 5 + 5 = 40
So, the answer is yes.

Chapter 3 Review Test Page No 188

Question 18.
Lucy and her mother made tacos. They put 2 tacos on each of 7 plates.
Select the number sentences that show all the tacos Lucy and her mother made. Mark all that apply.
Options:
A. 2 + 2 + 2 + 2 + 2 + 2 + 2 = 14
B. 2 + 7 = 9
C. 7 + 7 = 14
D. 8 + 6 = 14
E. 2 × 7 = 14

Answer: a,e

Explanation:
Total no. of tacos = 2
Each taco has 7 plates.
Addition Sentence can be written as 2 + 2 + 2 + 2 + 2 + 2 + 2 = 14
Multiplication Sentence can be written as 2 x 7 = 14

Question 19.
Jayson is making 5 sock puppets. He glues 2 buttons on each puppet for its eyes. He glues 1 pompom on each puppet for its nose.
Part A
Write the total number of buttons and pompoms he uses. Write a multiplication sentence for each.
Eyes, Noses
_________ buttons _________ pompoms

Answer: Eyes
10 buttons,
5 x 2 = 10

Noses
5 pompoms
5 x 1 = 5

Explanation:
Given, there are 5 sock puppets
Each puppet has 2 buttons for its eyes
Each puppet has 1 pompom for its nose
Total no. of buttons
Multiplication Sentence = 5 x 2 = 10
Total no. of pompoms
Multiplication Sentence = 5 x 1 = 5

Question 19.
Part B
After making 5 puppets, Jayson has 4 buttons and 3 pompoms left. What is the greatest number of puppets he can make with those items if he wants all his puppets to look the same? Draw models and use them to explain.
_________ puppets

Answer: 2 puppets

Explanation:
Given, there are no. of puppets = 5
There are 4 buttons and 3 pompoms.
Each puppet requires 2 buttons and 1 pompom.
So, he can make 2 puppets with the left items.